Sei sulla pagina 1di 36

CASE DIGESTS IN PARTIAL FULFILLMENT OF THE REQUIREMENTS IN CRIMINAL LAW I SUBMITTED TO ATTY. TEODORO ANGEL BY: HANNIYAH P.

SEVILLA LLB I-WIGMORE

1. PESIGAN VS. ANGELES (LIMITATIONS) 129 SCRA 174 (1994) Nature: Petition to review the order of the RTC of Caloocan City, Angeles. Facts: Anselmo and Marcelino Pesigan, both carabao dealers, transported 26 carabaos and a calf from Sipocot Camarines Sur to Batangas on April 2, 1982 with the necessary permits. In spite of the permits, the carabaos were confiscated by Lt. Zenarosa and Dr. Mirancda while passing Basud, Camarines Norte. The confiscation was based on EO No. 626-A which provided that no carabaos shall be transported from one province to another and violation would result to confiscation of said animals and distribution to deserving farmers. The carabaos were distributed accordingly. The Pesigans filed for replevin and damages but was dismissed by Judge Angeles for lack of merit. Issue: W o N the Pesigans can be held liable for violating EO 626-A which was published on June 14, 1982. Held: The trial courts order of dismissal, reversed and set aside. The executive order should not be enforced against the Pesigans on April 2, 1982 because it is a penal regulation punished more than two months later in the Official Gazette dated June 14, 1982. It became effective only 15 days thereafter. The word laws in Art. 2 of Civil Code includes circulars and regulations which prescribe penalties. Publication is necessary to apprise the public of the contents of the regulations and make the penalties binding on the persons affected by it. 2. TANADA VS. TUVERA (LIMITATIONS) 136 SCRA 27 (1985) Nature and Facts: This is a case where the petitioners, Lorenzo Taada, et al., seek a writ of mandamus to compel respondents, Juan Tuvera (in his capacity as Exec Asst to the Pres), et al., to publish and/or cause the publication in the official gazette of various presidential decrees, letters of instructions, general orders, proclamation, executive orders, letter of implementation and administrative orders Issue: Whether or not these presidential decrees are already enforceable and binding even before publication (contained special provisions as to the date they are to take effect) Held: The government, as a matter of policy, refrains form prosecuting violations of criminal laws until the same shall have been published in the official gazette or in some other publication, even though some criminal laws provide that they shall take effect immediately. Without publication, the people have no means of knowing what presidential decrees have actually been promulgated. Without such notice and publication, there would be no basis for the application of the maxim ignorantia legis non excusat. 3. PEOPLE VS. NARVAEZ (PROSPECTIVITY OF CRIMINAL LAW AND SELF DEFENSE/DEFENSE OF PROPERTY) 121 SCRA 389 (1983) Nature: Appeal from the decision of the Court of First Instance of South Cotabato, Branch I convicting the accused of murder. Facts: At about 2:30 in the afternoon of August 22, 1968, Graciano Juan, Jesus Verano and Cesar Ibaez, together with the two deceased Davis Fleischer and Flaviano Rubia, were fencing the land of George Fleischer, father of deceased Davis Fleischer. This is located in the municipality of Maitum, South Cotabato. At the place of the fencing is the house and rice drier of appellant Mamerto Narvaez. At that time, appellant was taking his rest, but when he heard that the walls of his house were being chiselled, he arose and there he saw the fencing going on. If the fencing would go on, appellant would be prevented from getting into his house and the bodega of his ricemill. So he addressed the group, saying -

'Pare,if possible you stop destroying my house and if possible we will talk it over - what is good,' addressing the deceased Rubia, who is appellant's compadre. The deceased Fleischer, however, answered: 'No, gademit, proceed, go ahead.' Appellant apparently lost his equilibrium and he got his gun and shot Fleischer, hitting him. As Fleischer fell down, Rubia ran towards the jeep, and knowing there is a gun on the jeep, appellant fired at Rubia, likewise hitting him. Both Fleischer and Rubia died as a result of the. It appears, however, that this incident is intertwined with the long drawn out legal battle between the Fleischer and Co., Inc. of which deceased Fleischer was the secretary-treasurer and deceased Rubia the assistant manager, on the one hand, and the land settlers of Cotabato, among whom was the appellant. Issue: W o N Narvaez defense of property can be appreciated. Held: Narvaez that he did so in defense of his person and of his rights, and therefore he should be exempt from criminal liability. Defense of one's person or rights is treated as a justifying circumstance under Art. 11, par. 1 of the Revised Penal Code, but in order for it to be appreciated, the following requisites must occur: "First. Unlawful aggression; "Second. Reasonable necessity of the means employed to prevent or repel it; "Third. Lack of sufficient provocation on the part of the person defending himself". The aggression referred to by appellant is the angry utterance by deceased Fleischer. There is no question, therefore, that there was aggression on the part of the victims: Fleischer was ordering, and Rubia was actually participating in the fencing. This was indeed aggression, not on the person of appellant, but on his property rights. The third element of defense of property is present, i.e., lack of sufficient provocation on the part of appellant who was defending his property. As a matter of fact, there was no provocation at all on his part, since he was asleep at first and was only awakened by the noise produced by the victims and their laborers. His plea for the deceased and their men to stop and talk things over with him was no provocation at all. Be that as it may, appellant's act in killing the deceased was not justifiable, since not all the elements for justification are present, particularly the reasonable necessity of the means employed to prevent or repel such attack. He should therefore be held responsible for the death of his victims, but he could be credited with the special mitigating circumstance of incomplete defense, pursuant to paragraph 6, Article 13 of the Revised Penal Code. The SC finds the aggravating (qualifying) circumstance of evident premeditation not sufficiently established. Since in the case at bar, there was no direct evidence of the planning or preparation to kill the victims nor that the accused premeditated the killing, and clung to his premeditated act, the trial court's conclusion as to the presence of such circumstance may not be endorsed. Evident premeditation is further negated by appellant pleading with the victims to stop the fencing and destroying his house and to talk things over just before the shooting. But the trial court has properly appreciated the presence of the mitigating circumstance of voluntary surrender, it appearing that appellant surrendered to the authorities soon after the shooting. Likewise, SC also finds that passion and obfuscation attended the commission of the crime. The appellant awoke to find his house being damaged and its accessibility to the highway as well as of his rice mill bodega being closed. 4. PEOPLE VS. BERNARDO (NULLUM CRIMEN NULLA POENA SINE LEGE) 123 SCRA 365 (1983) Nature: Petition for certiorari of the decision of CFI Bulacan, Br. VI convicting the petitioners of violating PD 772 (AntiSquatting Act), sentencing them to pay P 2, 500 each with subsidiary imprisonment in case of insolvency.

Facts: ISIDRO BERNARDO, tenant of Leda Sta. Rosa in her Riceland in PLARIDEL, BULACAN from Oct. 1972- Aug. 1974, constructed a house for their dwelling. Without the knowledge of Sta. Rosa, Isidro left and transferred to San Nicolas, transferring his tenancy rights to his son, CAYETANO. Through Dr. Patricio Cruz, Sta. Rosa took possession of the whole land and filed forcible entry against the petitioners. Petitioners lost in the CFI and lower courts but still failed to vacate the land. On Aug. 22, 1974, a criminal complaint for the violation of PD 772 was filed against the Bernardos and hence were found guilty by CFI of Bulacan. Issue: Whether or not PD772 applies to pasture lands. Held: Petition granted, judgment of conviction set aside; criminal case dismissed. PD 772 is intended to apply only to urban communities, particularly illegal constructions. No person shall be brought within the terms of a penal statute who is not clearly within them; nor should any act be pronounced criminal which is not clearly made so by the statute. (There is no crime when there is no law punishing it.) 5. PASCUAL vs. BOARD OF MEDICAL EXAMINERS (STRICT CONSTRUCTION) 28 SCRA 344 (1969) Nature: Appeal from a decision of the CFI of Manila for the writ of prohibition rendered on Aug. 2, 1965 Facts: At the initial hearing for an administrative case for alleged immorality, counsel for complainants called as his first witness, the appellee, Arsenio Pascual Jr., who was the one charged with malpractice. Issue/s: W o N the Board was right to call and coerce Pascual to take the witness stand against himself? W o N the right against self-incrimination can be availed of in an administrative hearing. Held: Decision of the lower court affirmed. The constitutional guarantee against self-incrimination extends to administrative hearings which possess a criminal or penal act. The Board cannot compel the person to take the witness stand without his consent. A proceeding for malpractice possesses a criminal or penal aspect in the sense that the respondent would suffer revocation of his license as a medical practitioner. The right against self-incrimination extends not only to the right to refuse to answer questions put to the accused while on the witness stand, but also to forego testimony and remain silent and refuse to take the witness stand. That while crimes should not go unpunished, objectives should not be accomplished by means offensive to the high sense of respect accorded to the human personality. 6. PEOPLE vs. TEMBLON (DOLO) 161 SCRA 623 (1988) Nature: Appeal from the judgment of CFI of Agusan del Norte and Butuan City convicting VICENTE TEMBLOR alias Ronald for murder sentencing him to the penalty of reclusion perpetua. Facts: On Dec. 30, 1980, at around 7:30 pm in Brgy. Talo-ao, Buenavista, Agusan del Sur, TEMBLOR bought a half-pack of Hope cigarette from the store of JULIUS CAGAMPANG. While the latter was opening the pack, a gun shot was heard and CAGAMPANG fell to the floor with a gunshot wound in the head. The accused, together with another, barged into the room, demanding the firearms of the victim. Before fleeing with the victims .38 caliber gun, TEMBLOR filed tow more shots. On Aug. 1981, during the mass surrender of dissidents, TEMBLOR surrendered to Mayor Dick Carmona. On Nov. 26, 1981, he was arrested by the Buenavista Police at the public market and detained at the municipal jail where he was seen by

VICTORIA CAGAMPANG, the victims widow and was positively identified. TEMBLORs defense was an alibi: on the said date, he and his father had been in the house of SILVERIO PEROL in Brgy. Camagong. Issue: W o N motive is essential in convicting Temblor Held: Decision of lower court affirmed; civil indemnity increased to P30,000. Motive is not essential when culprit has been identified. Fact that accused has knowledge of the deceaseds firearm is sufficient enough for motive. The fleeing of the accused after killing and taking CAGAMPANGs firearm implies admission of guilt. Accuseds alibi cannot prevail over the positive identification of the witness who had no base motive to accuse him of the crime. In order for alibi to be acceptable as a defense, it is not enough for the appellant to be elsewhere when the crime was committed, but it must be proven beyond reasonable doubt that it was physically impossible for him to be at the scene of the crime. Nasipit is accessible to Talo-ao by jeep or tricycle for 15 to 20 minutes. 7. PEOPLE vs. HASSAN 157 SCRA 261 (1988) Nature: Appeal from a decision of the RTC of Zamboanga City finding the accused guilty beyond reasonable doubt of the crime of murder and sentenced to reclusion peretua. Facts: July 23, 1981, at around 7pm JOSE SAMSON was a backrider on the motorcycle of RAMON PICHEL JR. when they went to buy mangoes at the Fruit Paradise near the Barter Trade Zone in Zamboanga City. SAMSON saw a person stab PICHEL only once while he was parked 2-3 meters away. After stabbing, the suspect fled to PNB. SAMSON rushed the victim to the General Hospital where the latter died. In the hospital, the witness was interrogated as to the suspects description, who according to him was had semi-long hair, wearing white polo short-sleeved shirt, maong pants, standing 55 and with a dark complexion. According to SAMSON, he only knows the suspect by face and not by name. At Funeraria La Merced, police brought the accused alone, for identification where the witness positively identified him as the killer. Held: Decision reversed; accused is acquitted. Evidence of the prosecution does not satisfy quantum of proof beyond reasonable doubt. Value judgment must not be separated from the constitutionally guaranteed presumption of innocence. Prosecutions evidence is weak and unconvincing. Expert testimony of the medico-legal officer (DR.VALENTIN BERNALEZ) contradicted on material points of the lone witness. He found two stab wounds, (chest and at the left arm posterior), the nature of the wounds indicating they were inflicted while the suspect was in front of the victim. The investigation conducted by the police was not satisfactory. The lone presentation of HASSAN to SAMSON at the funeral parlor violated the accused rights to counsel in all the stages of investigation into the commission of a crime. Motive is essential when there is doubt as to the identity of the culprit. 8. PEOPLE vs. AH CHONG (MISTAKE OF FACT) 15 PHIL 488 (1910) Nature: Appeal from the judgment of CFI of Rizal convicting the appellant of the crime of simple homicide with extenuating circumstances sentenced to 6 years 1 day of presidio mayor. Facts: Ah Chong was employed as a cook at Fort McKinley. He was roommates with the deceased, PASCUAL GUALBERTO at officers quarters #27, about 40 meters away form the nearest building, without a lock and had only 1 door opening to the

porch and 1 window. As a safety precaution, Ah Chong and Gualberto had an understanding that if either returned late at night, he should knock and acquaint the other as to his identity. On Aug. 14, 1908 at around 10 pm, he was awakened by someone trying to force open the door. He called out twice, Who is there?, to which no answer was given. Because of the vines covering the porch, the room was very dark. While calling out a threat to the invader, he was struck above the knee by the edge of the chair which was thrown towards his direction when the door was opened forcibly. Getting the common knife under his pillow and struck wildly at the intruder which turned out to be GUALBERTO. Seeing it was his roommate, Ah Chong ran back to his room to secure bandages and called to his employees in Room #28 for help. Prior to the incident, there had been several instances of robberies inside the port. Issue/s: W o N one can be held criminally liable for doing an act that would be exempt form criminal liability had there been no mistake of fact. W o N malice or criminal intent is an essential element or ingredient of the crimes of homicide and assassination as defined and penalized in the penal code. Held: Decision reversed; accused is acquitted. The definitions of crimes and offenses as set out in the penal code rarely contain provisions expressly declaring that malice or criminal intent is an essential ingredient of the crime. Nevertheless, the provisions of Art. 1 indicate malice and criminal intent as an essential requisite. Voluntary act is a free, intelligent and intentional act, and which, without intention there can be no crime. Voluntary implies and includes the words con malice or with malice. When the act which was actually intended to be done was in itself a lawful one, and in the absence of negligence or imprudence, in general without intention, there can be no crime. *Requisites of Mistake of fact: The act would have been lawful had the facts be how the accused believed them to be; The intention would have been lawful; The mistake was not attended by any fault or negligence on the part of the accused. There must be no reasonable opportunity to ascertain the facts Excessive force negates mistake of fact (OANIS) 9. PEOPLE VS. OANIS 74 PHIL 257 (1943) Nature: Appeal from the judgment of CFI of Nueva Ecija finding the accused guilty of homicide through reckless imprudence. Facts: On December 24, 1938, Provincial Inspector, CAPT. GODOFREDO MONSOD received a telegram from MAYOR GUIDO ordering the arrest (whether dead or alive) of one ANSELMO BALAGTAS, who was an escaped convict. The said convict was informed to be with a bailarina named IRENE. MONSOD then informed the defendants and instructed them to arrest Balagtas, and if overpowered, they were to follow the instructions in the telegram. OANIS, knowing a certain IRENE, accompanied GALANTA and went to the location of IRENE at Rizal St. Upon reaching the place, they asked BRIGADA MALLARE to point where IRENEs room was. They were also informed that Irene was sleeping with her paramour. When they reached the room, seeing a man with his back towards the door, they simultaneously and successively fired at him killing him. The man killed turned out not to be Balagtas, but one SERAPIO TECSON. Issue: W o N the killing of Tecson was a mistake of fact to exempt Oanis and Galanta from criminal liability.

Held: Defendants are guilty of murder with the mitigating circumstance of incomplete requisites of performance of duty appreciated in their favor and sentenced to reclusion temporal. Ignorance of fact can be appreciated only when the mistake is committed without fault of carelessness. Defendants had ample time and opportunity to ascertain the victims identity since he was asleep. Mistake in the identity of the intended victim cannot be considered reckless imprudence. 10. PEOPLE VS. BUAN (CULPA) 22 SCRA 1383 (1968) Nature: Appeal from an order of CFI Bulacan in its Criminal Case #5423, overruling a motion to quash on the ground of double jeopardy. Facts: JOSE BUAN was driving a passenger bus of the La Mallorca Company on July 23, 1962 along McArthur Highway, Guiguinto, Bulacan when it collided with the passenger jeep of Sergio Lumidao injuring 9 passengers. (6 suffered slight physical injuries, 3 serious physical injuries and damaging the jeep to the extent of P 1, 395.00) On December 19, 1963, he was acquitted in the Justice of Peace Court for slight physical injuries through reckless imprudence. However, prior the acquittal, the provincial fiscal of Bulacan filed in the CFI a case for serious physical injuries and damage through property through reckless imprudence. Issue: W o N the 2nd case places the appellant twice in jeopardy for the same offense and is barred by the previous acquittal. Held: The order of the CFI is reversed and is ordered to quash and dismiss the charges. Once acquitted or convicted of a specific act of reckless imprudence, the accused may not be prosecuted again for the same act. The gravity of the consequences is only taken into account to determine the penalty, it does not qualify the substance of the offense. 11. PEOPLE VS. DIZON (NEGLIGENCE) 158 SCRA 127 (1988) Nature: An administrative complaint against respondent Baltazar R. Dizon for rendering a manifestly erroneous decision due to gross incompetence and gross ignorance of the law Facts: The case in which the respondent rendered a decision of acquittal involved a tourist, Lo Chi Fai, who was caught by a Customs guard at the Manila International Airport while attempting to smuggle foreign currency and foreign exchange instruments out of the country. At the time of his apprehension, he was found carrying with him foreign currency and foreign exchange instruments (380 pieces) amounting to US$355,349.57, in various currency denominations without authority as provided by law. At the time the accused was apprehended, he was able to exhibit two currency declarations, which he was supposed to have accomplished upon his arrival in Manila in previous trips. Information was filed against Lo Chi Fai with the RTC of Pasay City for violation of Sec. 6, Central Bank Circular No. 960. The respondent judge, in his decision acquitting the accused, stated: The factual issue for this Court to determine is whether or not the accused willfully violated Sec. 6 of Circular No. 960. The fact that the accused had in his possession the foreign currencies when he was about to depart from the Philippines did not by that act alone make him liable for violation of Sec. 6. What is imperative is the purpose for which the act of bringing foreign currencies out of the country was done the very intention. Issue: Whether or not the respondent judge was guilty of gross incompetence or gross ignorance of the law in rendering the decision in question

Held: The Court found the respondent judge guilty of gross incompetence, gross ignorance of the law and grave and serious misconduct. The respondent judge has shown gross incompetence or gross ignorance of the law in holding that to convict the accused for violation of Central Bank Circular No. 960; the prosecution must establish that the accused had the criminal intent to violate the law. The respondent ought to know that proof of malice or deliberate intent (mens rea) is not essential in offenses punished by special laws, which are mala prohibita. 12. PEOPLE VS. VALDEZ (PROPOSAL CONSPIRACY) 159 SCRA 153 (1988) AND

Nature:This case is before us on automatic review of the decision of the Regional Trial Court, First Judicial Region, Branch 26, San Fernando, La Union, convicting the accusedappellants Danilo Valdez and Simplicio Orodio of the crime of murder and sentencing each of them to death. Facts: The house of the Maquiling family stands on the slope of a mountain in Barangay Ambagat, Santol, La Union. At about 8:00 o'clock in the evening of 7 June 1977, the victim Eleno Maquiling, his sisters Leticia and Thelma, his mother Esmenia, and his father Juanito were all in the yard of their house. Esmenia and Juanito were under the awning of their house facing north, engaged in stringing together tobacco leaves. The victim's brother Dionisio was eating his dinner in the wall-less kitchen located on the ground floor of the house. The victim Eleno was seated with his back toward the north and plucking a guitar. The place and its surroundings were lighted by a 300 candle power petromax lamp hanging under the northern end of the awning of the house. While the Maquilings were thus seated in their yard, a relative of the family, one Carolina, arrived and asked Esmenia to accompany her to a prayer meeting. Esmenia demurred and instead asked Eleno to accompany Carolina. The victim was then just about two (2) meters away from his parents and about to stand up when suddenly a very loud gun shot rang out from the northern side of the yard and Eleno fell to the ground, crying out to his father for help. Juanito rushed to his fallen son and carried him into their house; Eleno, however, died immediately thereafter. The victim's mother Esmenia was about to succor Eleno when she instinctively looked toward the direction from whence the gunshot came and saw the two(2) accused, Danilo Valdez and Simplicio Orodio, running down the hill away from the bamboo groves on the northern side of the house. Dionisio Maquiling, brother of the victim, also testified that he too had seen Danilo with a gun and Simplicio both running away in a westernly direction. Danilo stated that he was then about seven (7) meters away from the accused-appellants. Danilo Valdez was a neighbor and a relative of the Maquilings, while Simplicio Orodio was their old acquaintance residing in Sitio Villaga, Barangay Corooy of the same town; thus, both were well-known to Esmenia and Dionisio Maquiling. On 8 June 1977, the Municipal Health, Officer of Balaoan, Dr. Monico O. Morales, conducted an autopsy which showed that the victim Eleno had sustained eight (8) gunshot (pellet) wounds on his back. The morning after the shooting, on 8 June 1977, Sgt. Segundo Tuvera of the Integrated National Police, Santol, La Union, went to the house of the Maquilings to investigate the death of Eleno. He saw a petromax lamp hanging from the awning of the northern end of the house, as well as footprints near the bamboo groves near the northern side of the house. During his investigation, neither Esmenia nor Dionisio informed Sgt. Tuvera of what they had seen. On 10 June 1977, Juanito Maquiling, the victim's father, executed a sworn statement before the police in the Santol Police Substation. Juanito admitted in his statement that he had not seen the accused-appellants on the night of the shooting. He did relate, however, that three(3) days prior to the shooting of

Eleno, Eleno had informed him that in case something untoward happened to him (Eleno), the accused-appellants Danilo Valdez and Simplicio Orodio should be held responsible, since he (Eleno) had quarreled with them concerning their stealing and robbing. Juanito, further, stated that the accused Danilo has had a personal grudge against Eleno; Danilo had mortgaged to Eleno's brother a stolen spading fork, a circumstance that Eleno discovered when the real owner of the spading fork came to talk to him. Esmenia, Eleno's mother, gave no sworn statement on that day. Ten (10) days later, on 20 June 1977, however, she made a sworn statement to the Philippine Constabulary in San Fernando, La Union. Shortly thereafter, on 23 June 1977, Dionisio Maquiling, Eleno's brother, gave his own separate sworn statement also to the Philippine Constabulary. Both Esmenia and Dionisio identified Danilo Orodio as Eleno's killers. At the trial, Esmenia Maquiling was firm and categorical in identifying the appellants as the men she saw running from the bamboo groves immediately after the shooting.. Held: There is nothing in the record to show that the prosecution witnesses were moved by any improper motive to accuse falsely the accused-appellants one a relative and the other an old acquaintance of so grave a crime as murder. The prosecution's evidence was more than adequate to sustain the finding of the trial court of a conspiracy between Danilo Valdez and Simplicio Orodio. Conspiracy being present, it does not matter that the prosecution had failed to show who as between the two actually pulled the trigger of the shotgun that killed Eleno Maquiling. 17 Both Danilo Valdez and Simplicio Orodio are liable as co-conspirators since any act of a coconspirator becomes the act of the other regardless of the precise degree of participation in the act. The trial court correctly appreciated the presence of treachery and evident premeditation. The accused had purposely sought nocturnity and hid themselves behind the bamboo groves located close by the victim's house and had fired at Eleno Maquiling suddenly, without any warning, from behind obviously to ensure the success of their deadly purpose without any risk to themselves and without any possibility of retaliation. Since both treachery and evident premeditation were present, and only one qualifying circumstance is necessary to constitute homicide into murder, evident premeditation may be considered as a generic aggravating circumstance. The circumstance of nighttime is, however, absorbed by treachery. A second aggravating circumstance that the victim who had given no provocation was slain in his dwelling was also found by the trial court. 13. PEOPLE VS. NACIONAL 248 SCRA 122 (1995) Nature: Appeal from the decision of the Regional Trial Court of Legazpi City Facts: The six accused, including the accused-appellant, were all civilian members of the barangay organization of the CPP-NPA at Daraga, Albay. Their organization had a conference for the purpose of identifying suspected informers of the military whom they perceived as posing a threat to the NPAs operations within the vicinity. They identified Quirino Lagason and Joel Lagason as military informers and were targeted for liquidation. According to Crisanto Miranda, a neighbor who accompanied the accused that day, Walter Nacional approached Quirino and said something to him. Walter then pulled out a gun from his waist and shot Quirino in the face, hitting him between the eyebrows. Quirino fell to the ground and died instantly. A few seconds later, Absalon Millamina shot Joel Lagason on the head. The group then fled towards the direction of the RCPI Relay Station. Joels mother, who was at the scene of the crime, rushed him to the hospital where he died a few hours later. The defense set up by the accused consisted of denials. Accused-appellant Javier Mirabete insisted on his claim that he was merely watching a volleyball game when the shooting happened. He denied being a member of the NPA or any rebel

organization. He likewise denied the existence of a plot and a conspiracy to kill the Lagasons. Accused-appellants claimed that he was a mere farmer, already 69 years old and had barely finished third grade in school. According to him, his advanced age made it impossible for him to join the NPA at the time of the incident. He contended that the testimonies of the witnesses identifying him with the group that killed the Lagasons were unreliable and hearsay because both witnesses never knew him. Issue: Whether or not accused-appellants contention is with merit Held: The decision appealed from was affirmed insofar as the criminal liability of accused-appellant was concerned. Evidence proved beyond doubt that accused-appellant was a civilian member of the CPP-NPA and was part of the group that deliberately planned the killing of the Lagasons. The events that led to the victims deaths also showed that the group members deliberately planned, plotted and premeditated their victims deaths. Evident premeditation exists when the execution of the criminal act was preceded by cool thought and reflection upon the resolution to carry out the criminal intent. There must be, between the reflection and execution of the crimes, a space of time sufficient for the offender to arrive at a calm judgment. It was also held that the prosecution had clearly and convincingly established the existence of a conspiracy in the planning and execution of the crimes. Conspiracy arises at the very instant the plotters agree, expressly or impliedly, to commit the felony and forthwith to actually pursue it. It hardly matters that the accused-appellant was not actually present at the specific place of the shooting. He was at the waiting shed but this was for the purpose of providing security to those who carried out the shooting. The waiting shed was located along the way to the Lagasons house, strategically at the entrance to and exit from it. A conspiracy, once established, makes each of the conspirators liable for the acts of the others. All conspirators are liable as co-principals regardless of the extent of their participation because in the contemplation of law, the act of one is the act of all. 14. PEOPLE VS. TRINIDAD 169 SCRA 51 (1989) Nature: APPEAL from the judgment of the Regional Trial Court of Bayugan, Agusan del Sur, Br. 7, convicting the accused of two counts of murder and frustrated murder. Facts: The deceased victim, Lolito Soriano, was a fish dealer based in Davao City. His helpers were Ricardo Tan, a driver, and the other deceased victim Marcial Laroa. On 19 January 1983, using a Ford Fiera, they arrived at Butuan City to sell fish. In the morning of 20 January 1983 Soriano together with Laroa and a helper of one Samuel Comendador left for Buenavista. Tan was left behind in Butuan but followed to later in the morning. While at Buenavista, accused Emeliano Trinidad, a member of the Integrated National Police, assigned at Nasipit Police Station, and residing at Baan, Butuan City, asked for a ride to Bayugan, Agusan del Sur, which is on the way to Davao City. He was in uniform and had two firearms, a carbine, and the other, a side-arm .38 caliber revolver. The four left Butuan on 20 January 1983 at about 5:20 P.M. bound for Davao City. Tan was driving, seated to his right were Soriano, then Laroa and the accused. When they reached the stretch between El Rio and Afga, the police advised them to drive slowly because, according to him, the place was dangerous. All of a sudden, Tan heard two gunshots. Soriano and Laroa slumped dead. He did not actually see the shooting of Laroa but he witnessed the shooting of Soriano, having been alerted by the sound of the first gunfire. Both were hit on the head. The accused had used his carbine in killing the two victims. Ricardo hurriedly got off the Fiera, ran towards the direction of Butuan City and hid himself in the bushes. He heard a shot emanating from the Fiera while he was hiding in the bushes, then a passenger jeep passed by, he hailed it and rode on the front seat. However, after sometime, he noticed that the accusedappellant was seated at the back. Apparently noticing him as well, the accused ordered him to get out and to approach him. Instead of following, Tan moved backward and ran around the

jeep followed by the appellant. When the jeep started to drive away, Ricardo clung to its side. The accused fired two shots, one of which hit Tan on his right thigh. As another passenger jeep passed by, he jumped from the first jeep and ran to the second. However, the passengers in the latter jeep told him to get out not wanting to get involved in the affray. Pushed out, Ricardo crawled until a member of the P.C. chanced upon him and helped him board a bus for Butuan City. Trinidad was convicted for the murder of Laroa and Soriano and for the frustrated murder of Ricardo. Issue: W o N the accused the attack on Tan constitutes frustrated or attempted murder. Held: Decision modified. Trinidad was found guilty of the two murders and attempted murder. The defense is correct in contending that in the Frustrated Murder case, TRINIDAD can only be convicted of Attempted Murder. He had commenced the commission of the felony directly by overt acts but was unable to perform all the acts of execution which would have produced it by reason of causes other than his spontaneous desistance, such as, that the jeep to which TAN was clinging was in motion, and there was a spare tire which shielded the other parts of his body. Moreover, the wound on his thigh was not fatal and the doctrinal rule is that where the wound inflicted on the victim is not sufficient to cause his death, the crime is only attempted Murder, the accused not having performed all the acts of execution that would have brought about death. 15. PEOPLE VS. VELASCO 73 SCRA 574 (1976) Nature: Appeal from the decision of the Regional Trial Court Facts: On Nov. 2, 1967, the offended party, Estelita Lopez, five years old, accompanied by her cousin Nenita Lopez, was at the North Cemetery, Manila. The defendant, Ricardo Velasco, called them, gave Nenita a five-centavo coin and asked her to buy cigarettes for him. After she left, the accused held Estelita by the hand and brought her to an alley. Once in a hidden place between the tombs, he kissed her on the lips, took off her panties and placed himself on top of the girl while she was lying down on the ground face up and tried to insert his sexual organ into that of the victim. The girl shouted in pain, Aray, aray! Arsenio Perez, who happened to see the accused holding the hand of the girl while walking along 24th street in the cemetery as well as when they turned into the alley and who at first thought that the accused was a relative of the girl, upon hearing the shouts of the girl and because of the shouts believed that something bad was being done to the girl, proceeded to the place where the shouts came and upon standing on top of one of the tombs he saw, a short distance away, the accused on top of the girl, with his pants and drawers lowered down to his knees, and the dress of the girl raised up and the buttocks of the accused making upward and downward movements. He tried to seek for help and upon seeing Jose Castro on the 24th street told him, Mang Pepe, Mang Pepe, the daughter of Mang Pidiong is being raped! Castro proceeded to the place pointed by Perez with the latter following him, and while standing on top of a tomb, a short distance away, Castro saw the defendant on top of the girl. The accused then stood up and raised his pants while the girl rose from the ground crying. Castro approached the defendant and the girl and asked him what happened, and he said the girl lost her way and was crying. The girl was bleeding at the same time and he noticed that she even wiped off with her dress the blood on the front part of her thighs. Issue: Whether or not the accused is guilty of consummated rape Ruling: The decision of the lower court finding the accused guilty of the crime of consummated rape was affirmed. There was no question that rape was the crime committed, but because of the tender age of the victim, penetration was impossible due to the infantile character of the vagina. However, considering the anatomical position of the

labia majora and minora, that these two external parts of the female sexual organ cover the hymen and the vaginal opening and, therefore, in order to rupture the hymen and produce the medico-legal finding that the vaginal opening was painful and sensitive to touch. The conclusion was inevitable that the sexual organ of the accused must have entered and had passed the labia majora. 16. URBANO VS. IAC 157 SCRA 1 (1998) Nature: This is a petition to review the decision of the then Intermediate Appellate Court which affirmed the decision of the Circuit Criminal Court of Dagupan City finding petitioner Filomeno Urbano guilty beyond reasonable doubt of the crime of homicide. Issue: Whether or not there was an efficient intervening cause from the time Javier was wounded until his death which would exculpate Urbano from any liability for Javiers death. Facts: (1) At about 8:00oclock in the morning of October 23, 1980, petitioner Filomeno Urbano went to his rice field at Barangay Anonang, San Fabian, Pangasinan located at 100 meters from the tobacco seedbed of Marcelo Javier . He found the place where he stored his palay flooded with water coming from the irrigation canal nearby which had overflowed. Urbano went to the elevated portion of the canal to see what happened and there he saw Marcelo Javier and Emilio Erfe cutting grass. He asked them who was responsible for the opening of the irrigation canal and Javier admitted that he was the one. Urbano then got angry and demanded that Javier pay for his soaked palay. A quarrel between them ensued. Urbano unsheathed his bolo (about 2 feet long, including the handle, by 2 inches wide) and hacked Javier hitting him on the right palm of his hand, which was used in parrying the bolo hack. Javier who was then unarmed ran away from Urbano but was overtaken by Urbano who hacked him again hitting Javier on the left leg with the back portion of said bolo, causing a swelling on said leg. When Urbano tried to hack and inflict further injury, his daughter embraced and prevented him fro hacking Javier. (2) Upon the intercession of Councilman Solis, Urbano and Javier agreed to settle their differences. Urbano promised to pay Php 700.00 for the medical expenses of Javier. Hence, on October 27, 1980, the two accompanied by Solis appeared before the San Fabian Police to formalize their amicable settlement. (3) However, at about 1:30 a.m. on November 11, 1980 Javier as rushed to the Nazareth General Hospital in very serious condition. When admitted to the hospital Javier lockjaw and was having convulsion Dr. Edmundo Exconde who personally attended to Javier found the latters serious condition was caused by tetanus toxin. Eh noticed the presence of a hearing wound in Javiers palm which could have been infected by tetanus. (4) In an information dated April 10, 1981 Filomeno Urbano was charged with crime of homicide before the then Circuit Court of Dagupan City, Third Judicial District. Upon agreement, Urbano pleaded not guilty. After trial, the trial court found Urbano guilty as charged. We was sentenced to suffer an indeterminate prison term of twelve (12) years of prision mayor, as minimum to seventeen (17) years., four (4) months and one (1) day of reclusion temporal, as maximum, together with the accessories of the law to indemnify the heirs of the victim, Marcelo Javier, in the amount of Php 12,000.00 without subsidiary imprisonment in case of insolvency, and to pay the costs. He was ordered confined at the New Bilibid Prison, in Muntinlupa, Rizal upon finality of the decision, in view of the nature of his penalty. The then Intermediate Appellate Court affirmed the conviction of Urbano on appeal but raised the award of indemnity to the heirs of the deceased to Php 30,000.00 with cost against the appellant. The appellant filed a motion for reconsideration and/or new trial. The motion for new trial was based on an affidavit of Barangay Captain Menardo Soliven. (5) The motion was denied. Hence, this petition. Ruling: The instant petition is granted, the questioned decision of the then Intermediate Appellate Court, now Court of Appeal,

is reversed and set aside. The petitioner is acquitted of the crime of homicide. Costs de oficio. Rationale: (a) The evidence on record does not clearly show that the wound inflicted by Urbano was infected with tetanus at the time of the infliction of the wound. The evidence merely confirms that the wound, which was already healing at the time Javier suffered the symptoms of the fatal ailment, somehow got infected with tetanus. However, as to when the wound was infected is not clear from the record. (b) Doubts are present. There is likelihood that the wound was but the remote cause and its subsequent infection, for failure to take necessary precautions, with tenants may have been the proximate cause of Javiers death with which the petitioner had nothing to do. 17. PEOPLE VS. ABARCA 153 SCRA 735 (1987) Nature: This is an appeal from the decision of the Regional Trial Court of Palo, Leyte, sentencing the accused-appellant Francisco Abarca to death for the complex crime of murder with double frustrated murder. Issue: The accused-appellant assigns the following errors by the court a quo: (i) in convicting the accused for the crime charged instead of entering a judgment of conviction under article 247 of the Revised Penal Code, and (ii) in finding that the killing was attended by the qualifying circumstance of treachery. Facts: (1) Khingsley Paul Koh and the wife of accused Francisco Abarca. Jenny had illicit relationship. The illicit relationship apparently began while the accused was in Manila reviewing for the 1983 bar examinations. (2) On July 15, 1984, the accused was in his residence in Tacloban, Leyte. On the morning of that date he went to the bus station to go to Dolores, Easter Samar, to fetch his daughter. However, he was not able to catch the first trip (in the morning). He went back to the station in the afternoon to take the 2:00 oclock trip but the bus engine trouble and could not leave. The accused then proceeded to the residence of his father after which he went home. He arrived at his residence at the V & G Subdivision in Tacloban City at around 6:00 oclock in the afternoon. (3) Upon reaching home the accused found his wife, Jenny and Khingsley in the act of sexual intercourse. When the wife and Koh noticed the accused, the wife pushed her paramour who got his revolver. The accused who was then peeping above the builtin cabinet in their room jumped and ran away. (4) The accused went to look for a firearm at Tacloban City. He went to the house of PC soldier, C2C Arturo Talbo, arriving there at around 6:30 p.m. He got Talbos firearm, an M -16 rifle and went back to his house at V & G Subdivision. He was not able to find his wife and Koh there. He proceeded to the mahjong session as it was the hangout of Kingsley Koh. The accused found Koh playing mahjong. He fired at Kingsley Koh three times with his rifle. Koh was hit. Arnold and Lina Amparado who were occupying a room adjacent to the room where Koh was playing mahjong were also hit by shots fired by the accused Kingsley Koh died instantaneously of cardiorespiratory arrest due to shock and hemorrhage as a result of multiple gunshot wounds in the head, trunk and abdomen. Arnold Amparado was hospitalized and operated on in the kidney to remove a bullet. His wife, Lina Amparado, was also treated in the hospital as she was hit by bullet fragments. Ruling: The decision appealed from is modified. The accusedappellant is sentenced to four months and 21 days to six months of arresto mayor. The period within which he has been in confinement shall be credited in the service of these penalties. He is furthermore ordered to indemnify Arnold and Lina Amparado in the Sum of Php 16,000.00 as and for Arnold Amparados loss of earning capacity. No special pronouncement as to costs. Rationale: (a) Art. 247 of the RPC apply in the instant case. There is no question that the accused surprised his wife and her paramour, the vacuum in this case, in the act of illicit copulation, as, a result of which he went out to kill the deceased in a fit of passionate outburst. Article 247 prescribes the following

elements: (1) that a legally married person surprises his spouse in the act of committing sexual intercourse with another person, and (2) that he kills any of them or both of them in the act or immediately thereafter. These elements are present in this case. The trail court, in convicting the accused-appellant of murder, therefore erred. (b) Article 247, or the exceptional circumstances mentioned therein, amount to an exempting circumstances mentioned therein, amount to an exempting circumstance, for even where death or serious physical injuries is inflicted, the penalty is so greatly lowered as to result to no punishment at all. (c) Article 247 of the Revised Penal code does not define and provide for specific crime, but grants a privilege or benefit to the accused for the killing of another or the infliction of serious physical injuries under the circumstances. Punishment consequently is not inflicted upon the accused. He is banished, but that is intended for his protection. (d) The next question refers to the liability of the accused-appellant for the physical injuries suffered by Lina Amparado and Arnold Amparado who were caught in the crossfire as the accusedappellant shot the victim. The Solicitor General recommends a finding of double frustrated murder against the accusedappellant and being the more severe offense, proposes the imposition of reclusion perpetua in its maximum periods pursuant to Article 48 of the Revised Penal Code. This where we disagree. The accused-appellant did not have the intent to kill the Amparado couple. Although as a rule, one committing as offense is liable for all the consequences of his act, that rule presupposes that the act done amounts to a felony. Here, the accused-appellant was not committing murder when he discharges his rifle upon the deceased. Inflicting death under exceptional circumstances is not murder. 18. PEOPLE VS. ABAGON 161 SCRA 255 (1988) Nature: This is an appeal from the decision of the RTC, which found the accused guilty of the crime murder. Issue: Accused-appellant, thru their counsel, raise the following assignments of error: (i) The Honorable Regional Trial Court erred in finding that the prosecutions evidence consisting of the testimony of its witnesses sufficiently established the guilt beyond reasonable doubt of the accused for the killing of ones Celis Lupango, which killing was qualified to murder by treachery, (ii) The Honorable Regional Trial Court erred in holding that the defense of alibi on the part of the accused Mateo Abagon and of self-defense on the part of Abner Ongonion will no lie, (iii) The Honorable Trial court erred in declaring that conspiracy had been sufficiently established, and (iv) The Honorable Regional Trial Court erred in finding both accused guilty of the crime of murder considering that their guilt were not established beyond reasonable doubt. Facts: (1) About 6:00 oclock in the afternoon of April 17, 1981, at Barangay Pinamarubuhan, Mobo, Masbate, while the herein victim Celis Lupango and companions Isabelo Radaza, Jr., Benjamin Bergado and Nilo Lalaguna were inside the store of Corazon Cana to celebrate the birthday of Isabelo Radaza, Jr., two persons, later identified as Mateo Abagon and Abner Ongonion, entered and stabbed Celis Lupango. First to enter was Abner Ongonion, followed closely behind Mateo Abagon, and with a six-inches double bladed knife stabbed Celis Lupango three or four times. Mateo Abagon, in turn, with a seven-inch knife also stabbed Celis Lupango several times. (2) After Celis Lupango fell to the ground the two accused left. At this point Terisito Lupango, brother of Celis Lupango, arrived and he carried Celis Lupango, with the help of Benjamin Bergado and Nilo Lalaguna whom he found inside the store, outside the store intending to bring him to the hospital. Outside the store, the waiting Abner Ongonion, who was with Mateo Abagon, Julio Ongonion, Alejandro Ongonion, Romulo Barruga, Antonio Danao and Arnel Onarosa, he drew his firearm and fire two shots at them. Upon being fire at, Benjamin Bergado and Nilo Lalaguna ran away while Teresito Lupango sought cover. Abner Ongonion and his companions approached and they took turns in stabbing the prostate body of Celis Lupango with bolos and knives. Terisito eventually able to report the incident.

(3) The cause of death was established to be shock, secondary to massive external. Hemorrhage caused by multiple stabs and incised wounds. Testifying, Dr. Quemi admitted the possibility that the wounds were afflicted the possibility that the wounds were inflicted by one or two assailants. (4) The defense presented both accused to deny the charges. For his defense, accused Mateo Abagon claims that at the time of the incident he was in his house at the seashore of Pinamarubuhan about 100 meters away from the scene of the incident. He went out of the house only he learned of the stabbing incident when he saw many persons running towards the scene. After seeing the lifeless body of the victim, he returned home immediately. He did not see his co-accused Abner Ongonion at the scene. In fact he did not see any other person there. On the other hand, accused Abner Ongonion claims that at that particular time he left his house to fetch his mother at the Tugbo River where she washed clothes. On his way he passed by the store of Corazon Cana to buy cigarettes. At the store he was pulled inside by Celis Lupango, where the latter was drinking with others, among who was June Radaza. He was asked to drink but he refused because of a headache. Celis then asked him why are you brave and then he pulled his knife, but at as he did so the knife bumped the edge of the table and fell to the ground. As Celis recovered the knife from the ground, Ongonion was able to get hold of Celis hand and they grappled for the knife. While grappling he succeeded in thrusting the knife to the left breast of Celis and again thrust it to the stomach. After he was able to get possession of the knife he kept on stabbing Celis, being by then angry. In the meantime, as they grappled, the companions of Celis Lupango jumped out of the window, while June Radaza who was there watching ran away when he saw Celis was stabbed. Eh then went out and proceeded to the PC Headquarters at Masbate and surrendered. Benjamin Bergado and Teresito Lupango were not seen by him in the store. He then stated that his co-accused Mateo Abagon was in the store. Ruling: Wherefore the judgment appealed from is affirmed except that the penalties are modified. Appellant Mateo Abagon is sentenced to suffer an indeterminate penalty of imprisonment from twelve years and one day of prision mayor as minimum to eighteen years, eight months, and one day of reclusion temporal as maximum. Appellant Abner Ongonion is sentenced to suffer an indeterminate penalty of imprisonment from ten years and one day of prision mayor as minimum to seventeen years, four moths and one day of reclusion temporal as maximum. The two accused-appellants shall pay jointly the amount of, thirty thousand pesos to the heirs of Celis Lupango as indemnity Rationale: (i) Appellant Ongonions theory of self-defense is untenable. According to the testimonies of Bergado, Radaza, and Lupango, the attack by the assailant was unprovoked. The sudden attack on the victim with knives drawn indicates that the stabbing was unintentional. No other conclusion can likewise be surmised from their gunshots fired by the assailants at those who tried to bring Celis to the hospital, while the victims body lay helpless i=on the street, the appellants kept on stabbing the victim, thereby ensuring his death. (ii) Having admitted the killing Ongonion must clearly establish that he acted in selfdefense, the burden of proof is now shifted to him, he must, therefore, rely on the strength of his own evidence and not on the weakness of the prosecution (People vs. Sandie, 149 SCRA 240; and People vs. Regulacion) for even if the latters evidence is weak, it could not be disbelieved after the appellant admitted the killing. The number and nature of the stab wounds inflicted by more than one person beloved Ongonions theory of selfdefense. These and the testimonies of two eyewitnesses and one peace officer further serve to destroy Ongonions statement. Moreover it is a well settled rule that the findings of the fact of the trial court on the credibility of witnesses are generally accorded the highest respect by the appellee court (People vs. Traya, 147 SCRA 381) for these courts have the privileges of examining the deportment and demeanor of witnesses, and therefore, can discern if such witnesses, and therefore, can discern if such witnesses are telling the truth or not (People vs. Ramilo, 147 SCRA 102). (iii) Ongonions claim of self-defense is likewise negated by the physical evidence and other circumstances, such as his failure to present the knife upon surrender, his failure to tell the police authorities that he killed

the deceased in self-defense and the absence of any injury on the body of Ongonion while the deceased suffered eleven wounds when, according to the appellant, there was supposedly a struggle that tool place. For self-defense to prosper, the following elements should have been proven by appellant: (a) unlawful aggression, (b) reasonable necessity of the means employed to prevent or repel it; and (c) lack of provocation on the part of the one defending himself. On the contrary, all the evidence on record shows that not one of the elements of selfdefense is present. (iv) Appellant Abagaons defense on th e other hand, is alibi, an inherently weak defense especially when it can be proved that it was not physically impossible for him to be at the scene of the crime. In order to be given full faith and credit, alibi must not leave any room for doubt as to its plausibility and veracity. The appellant at the time of the crime was allegedly in a place which approximately only 100 meters away from the scene of the crime, renders his defense of alibi not credible. (v) More important, Abagon and his companion were positively identifies by eyewitnesses Bergado and Lalaguna. The records show that the appellants took turns at stabbing the victim inside and outside the store. The presence and location of the eleven stab wounds, as testified by Dr. Quemi also indicate that the same were inflicted by more than one person. (vi) The assailants acted in concerted efforts with community of criminal purpose to ensure the death of the victim is indicative of conspiracy between them. Conspiracy is established by concerted action It may be noted that even if conspiracy had not been established, the liability of the two appellants would not change for each inflicted on his own, multiple stabbing blows on the victim resulting in mortal injuries. They acted as principals by direct participation. (vii) Treachery was likewise proven by the evidence presented. The attack was immediate, sudden and unexpected. Treachery exists when the offender commits any crime against person, employing means, methods or forms in the execution, without risk to him arising from any defense which the offended party might make. 19. PEOPLE VS. IGNACIO G.R. NO. 134563 (FEB. 2000) Nature: Appeal on the May 18, 1998 decision of the Regional Trial Court of Masbate, Masbate, convicting Eulogio Ignacio of murder Facts: On January 11, 1997 (morning) in Barangay Divisoria, Municipality of Dimasalang, Masbate, Jessie Lacson and Edwin Velasco were gathering shells from the seashore. They got thirsty, went to the fishpond and get coconuts or butong. The fishpond is owned by Cielo Cortes alias Malagring. Eulogio Ignacio, Loloy, is the caretaker, which stays at the house inside the fishpond. Jessie then got 1 coconut, walked ahead of Edwin in going to the dike, to break open the coconut. Eulogio saw Jessie as Jessie reached the dike, he did not see Edwin walking behind Jessie. |Eulogio asked Jessie to put down the young coconut. Jessie did. Eulogio fired his homemade shotgun at Jessie hitting the left portion of his breast. Eulogio was meters away fro, Jessie. Edwin was meters away. Eulogio cranked his homemade shotgun aimed it at Edwin but did not fire. Edwin went to Jessies parents, then to Carlit Alcove, the Baraga Tanned. Carlit asked Eulogio to surrender which he did. Version of the Defense: Eulogio acted on the defense of property. On January 9, 1997, he was informed by his neighbor, Gil Aristotles regarding a theft incident in the fishpond. On January 11, 1997, he saw Jessie and Edwin coming out of his house carrying a basket his house was filled with 28 crabs. Upon seeing Eulogio, Jessie and Edwin fled. He ordered them to stop; they did not, so he fired his gun (without intention to kill them). He then reported incident to Kalawao Gil Aristotles. Ruling: There was no legal reason for him to shoot the victim, an unarmed minor; killing was qualified as murder because of the presence of treachery. Issue: 1st issue: Evidence of appellants guilt For jstifyi8ng circumstances to be appreciated, the accused has the burden of proving unlawful aggression on the part the victim. Eulogio was not attacked by Jessie.

2nd issue: Treachery A killing is qualified as treachery when the accused employs means without risk to him arising from the defense which the offended party might make. Victim and companion stopped after Eulogio shouted at them Jessie was already facing him when he shot him There was no proof that he deliberately and consciously adopted any means to kill did not act on mere impulse 3rd issue: Mitigating Circumstance Mitigating circumstance of because he allegedly gave himself up?

voluntary surrender

Mitigating Circumstance: 1. offender has not been actually arrested 2. offender surrender himself to a person of authority 3. surrender is voluntary =No mitigating circumstance he forced to give himself up Main issue: Whether Eulogio acted in unlawful defense of the landowners property. Held: Appeal is denied, assailed decision is affirmed. 20. PEOPLE VS. RICOHERMOSO 56 SCRA 431 (1974) Nature: Severo Padernal and Juan Padernal appealed from the decision of the Circuit Criminal Court at Lucena City, convicting them of murder, sentencing each of them to reclusion perpetua and ordering them to pay solitarily the sum of twelve thousand pesos to the heirs pf Geminiano de Leon ad to pay the costs (Criminal Case No. CCC-IX-37- Quezon or 1922-CFIGumaca). In the same decision they were convicted of lesions levees. Each one was sentences to suffer the penalty of fifteen (15 days of arresto manor and to pay the costs. Rosendo Perpean, Rito Monterey and Macario Monterey were acquitted (Criminal Case No. CCC-IX-38- Quezon or 1923-CFI-Gumaca). Issue: The only issue in this appeal, which concerns Juan Padernal, is whether he conspired with Ricohermoso and Severo Padernal to kill Geminiano de Leon. Facts: 1. At about nine oclock in the morning of January 30, 1965 Geminiano de Leon, together with his thirtythree-year old common-law wife Fabian Rosales, his twenty year old son Marian to de Leon and one Rizal Rosales, encountered Poi Ricohermoso in Barrio Tagabawa Silage, Catamaran, Quezon. 2. Geminiano owned a parcel of land in that barrio which Ricohermoso cultivated as kingpin. Geminiano asked Ricohermoso about his share of the palay harvest. He added that he should at least be allowed to taste the palay harvested from his land. Ricohermoso answered that Geminiano could go to his house anytime and he would give the latter palay. Geminiano rejoined that he could not get the palay that morning because he was on his way to Barrio Bag basin but, on his return, he would stop at Ricohermoso house and get the palay. 3. When Geminiano returned to Barrio Tagabawa Silage, he stopped at Ricohermoso place. It was about two oclock in the afternoon. Geminiano sat on a sack beside Fabian Rosales in front of the house while Marian to stood about three meters behind his father. Ricohermoso stood near the door of his house while Severo Padernal was stationed near the eaves of the house. 4. Geminiano asked Ricohermoso about the palay. The latter, no longer conciliatory and evidently hostile, answered in a defiant tone: Whatever happens, I will not give you palay. Geminiano restated: Why did you tell us to pass by your house, you were not willing to give the palay?

5. At that juncture, as if by pre-arrangement, Ricohermoso unsheathed his bolo and approached Geminiano from the left, while Severo Padernal (Ricohermoso father-inlaw) got an axe and approached and approached Geminiano from the right. The latter looked up to the sexagenarian Severo Padernal, with both hands raised and pleaded: Mama (gran dpa), why will you do this to us. We will not fight you. While Geminiano was still looking up to Severo Padernal on his right, Ricohermoso walked to Guineans left, and, when about one meter from him ,stabbed him on the nick with his bolo. Geminiano fell face downward on the ground. While in that helpless position, he was hacked on the back with an axe by Severo Padernal. 6. At the same place and time, while Severo Padernal and Ricohermoso were assaulting Geminiano de Leon, another episode was taking place. Juan Padernal (Ricohermosos brother-in-law and the son of Severo) suddenly embraced Marianito de Leon from behind , with his right armed locked around Marionettes neck and his left hand pressing Marionettes left forearm. They grappled and rolled downhill towards a camote patch. Marianito passed out. When he regained consciousness, his rifle was gone. He walked uphill, saw his mortally wounded father Geminiano in his death those, and embraced him, He carried Geminiano for a short distance. The fifty-one year old Geminiano died at two oclock on that sane day. 7. Doctor Matundan said that the first wound was fatal. It could have caused instantaneous death because it was a deep wound which pierced the carotoid artery and jugular vain. The second wound on the back could likewise have caused the victims death if it had penetrated the kidney. 8. Doctor Matundan found that Marianito de Leon sustained multiple abrasions on the nick and abdomen and a lacerated wound on the left foot which would heal from one to nine days even without medical treatment. 9. Appellants version is that in the afternoon of January 30, 1965, when Ricohermoso refused to give any palay to Geminiano de Leon, because the land tilled by the former as allegedly a public land, Geminiano approached Ricohermoso. When Geminiano unsheathed his bolo, Ricohermoso met him drew his bolo and struck Geminiano on the left side of the neck. The latter tried to parry the blow. He was wounded in the wrist. As Geminiano turned right to flee, Ricohermoso struck him again on the left side of the body, causing him to fall on the ground. Geminiano died on the spot due to the bleeding from the wound on his neck. While Geminiano was being assaulted, his son Marianito tried to shoot with his rifle but Juan Padernal disabled him and wrested the gun. Marianito suffered abrasions on the neck and other parts of the body. 10. The appellants filed their brief on February 6, 1970. Later, Severo Padernal withdrew his appeal Ruling: Wherefore, the judgment of the lower court as to appellant Juan Padernal is affirmed with costs against him. Rationale: The trial court rationalized its conclusion that there was conspiracy by stating that their conduct revealed unity of purpose and a concerted effort to encompass Geminiano death. Appellant Juan Padernal invokes the justifying circumstance of avoidance of a greater evil or injury in explaining his act of preventing Marianito de Leon from shooting Ricohermoso and Severo Padernal. His reliance on that justifying circumstance is erroneous. The act of Juan Padernal in preventing Marianito de Leon from shooting Ricohermoso and Severo Padernal, who were the aggressors, was designed to insure the killing of Geminiano de Leon without any risk to his assailants. Juan Padernal was not avoiding any evil when he sought to disable Marianito. Adrenals malicious intention was to forestall any interference in the felonious assault made by his father and brother-in-law on Geminiano. That situation is unarguably not the case envisaged in paragraph 4 of article 11. It should be recalled that, in the morning, Geminiano had an understanding with Ricohermoso that he would return in the afternoon to get his share of the palay harvest Ricohermoso gave Geminiano the impression that he was amenable to giving Geminiano his share of the harvest. However, during the interval, Ricohermoso changed his mind. Instead of remaining

steadfast to his original intention to give Geminiano palay, Ricohermoso planned with his father in law, Severo Padernal, the manner of liquidating Demonian so as to stop him from pestering Ricohermoso with demands for a share in the harvest. So, when Geminiano reappeared at Ricohermosos place in the afternoon, Severo Padernal, Ricohermoso, Juan Padernal, like actors in a well rehearsed play, performed their assigned roles with dramatic positions. Severo Padernal and Ricohermoso, one armed with an axe and the other with a bolo, in a pincer movement, confronted Geminiano de Leon. Simultaneously with that maneuver, the 35 year old Juan Padernal embraces Marianito de Leon and prevented him from firing at Severo Padernal and Ricohermoso or from helping his father. Considering the trios behavior and appellant Juan Adrenals close relationship to Ricohermoso and Severo Padernal ineluctable conclusion is that he acted in conspiracy with them. The circumstances surrounding the killing of Geminiano de Leon alevosia and treachery. His hands were raised and he was pleading for mercy with Severo Padernal, when Ricohermoso struck him on the neck with a bolo. 21. PEOPLE VS. BERONILLA 96 PHIL 566 (1955) Nature: This is an appeal by accused Manuel Beronilla, Policarpio Paculdo, Filipino Velasco, and Jacinto Adriatico from the judgment of the Court of First Instance of Abra (Criminal Case No. 70) convicting them of murder for the execution of Arsenio Borjal in the evening of April 18, 1945, in the town of La Paz , Province of Abra. Facts: Arsenio Borjal was the elected mayor of La Paz, Abra, at the outbreak of war, and continued to serve as Mayor during the Japanese occupation, until March 10, 1943, when he moved to Bangued because of an attempt upon his life by unknown persons. On December 18, 1944, appellant Manuel Beronilla was appointed Military Mayor of La Paz by Lt. Col. R. H. Arnold, regimental commander of the 15th Infantry, Philippine Army, operating as a guerrilla unit in the province of Abra. Simultaneously with his appointment as Military Mayor, Beronilla received copy of a memorandum issued by Lt. Col. Arnold to all Military Mayors in Northern Luzon, authorizing them "to appoint a jury of 12 bolomen to try persons accused of treason, espionage, or the aiding and abetting (of ) the enemy" (Exhibit 9). He also received from the Headquarters of the 15th Infantry a list of all puppet government officials of the province of Abra (which included Arsenio Borjal, puppet mayor of La Paz), with a memorandum instructing all Military Mayors to investigate said persons and gather against them complaints from people of the municipality for collaboration with the enemy (Exhibit 12-a). Sometime in March, 1945, while the operations for the liberation of the province of Abra were in progress, Arsenio Borjal returned to La Paz with his family in order to escape the bombing of Bangued. Beronilla, pursuant to his instructions, placed Borjal under custody and asked the residents of La Paz to file complaints against him. In no time, charges of espionage, aiding the enemy, and abuse of authority were filed against Borjal; a 12-man jury was appointed by Beronilla, composed of Jesus Labuguen as chairman, and Benjamin Adriatico, Andres Afos, Juanito Casal, Santiago Casal, Benjamin Abella, Servillano Afos, Mariano Ajel, Felimon Labuguen, Felix Murphy, Pedro Turqueza, and Delfin Labuguen as members; while Felix Alverne and Juan Balmaceda were named prosecutors, Policarpio Paculdo as clerk of the jury, and Lino Inovermo as counsel for the accused. Later, Atty. Jovito Barreras voluntarily appeared and served as counsel for Borjal. Sgt. Esteban Cabanos observed the proceedings for several days upon instructions of Headquarters, 15th Infantry. The trial lasted 19 days up to April 10, 1945; the jury found Borjal guilty on all accounts and imposed upon him instruction from his superiors. Mayor Beronilla forwarded the records of the case to the Headquarters of the 15th Infantry for review and on the night of the same day, April 18, 1945, Beronilla ordered the execution of Borjal. Jacinto Adriatco acted as executioner and Anotnio Palope as grave digger. Two years thereafter, Manuel Beronilla as military mayor, Policarpio Paculdo as Clerk of the jury, Felix Alverne and Juan Balmaceda as prosecutors, Jesus Labuguen, Delfin Labuguen, Filemon Labuguen, Servillano Afos, Andres Afos, Benjamin

Adriatico, Juanito Casel, Santiago Casel, Mariano Ajel, Felix Murphy, Benjamin Abella, and Pedro Turqueza as members of the jury, Jacinto Adriatico as executioner, Severo Afos as grave digger, and Father Filipino Velasco as an alleged conspirator, were indicted in the Court of First Instance of Abra for murder, for allegedly conspiring and confederating in the execution of Arsenio Borjal. Soon thereafter, the late President Manuel A. Roxas issued Executive Proclamation No. 8, granting amnesty to all persons who committed acts penalized under the Revised Penal Code in furtherance of the resistance to the enemy against persons aiding in the war efforts of the enemy. Defendant Jesus Labuguen, then a master sergeant in the Philippine Army, applied for and was granted amnesty by the Amnesty Commission, Armed Forces of the Philippines (Records, pp. 618-20). The rest of the defendant filed their application for amnesty with the Second Guerrilla Amnesty Commission, who denied their application on the ground that the crime had been inspired by purely personal motives, and remanded the case to the Court of First Instance of Abra for trial on the merits. Issue: The crucial question thus becomes whether or not this message, originally sent to Arnold's quarters in San Esteban, Ilocos Sur, was relayed by the latter to appellant Beronilla in La Paz, Abra, on the morning of April 18, 1945, together with the package of records of Borjal's trial that was admittedly returned to and received by Beronilla on that date, after review thereof by Arnold (Exhibit 8-8-a). Obviously, if the Volckmann message was known to Beronilla, his ordering the execution of Borjal on the night of April 18, 1945 can not be justified. Ruling: The judgment appealed from is reversed and the appellants are acquitted with costs de officio. Rationale: The records are ample to sustain the claim of the defense that the arrest, prosecution and trial of the late Arsenio Borjal were done pursuant to express orders of the 15th Infantry Headquarters. (Exhibit 9 and 12-a), instructing all military mayors under its jurisdiction to gather evidence against puppet officials and to appoint juries of at least 12 bolomen to try the accused and find them guilty by two thirds vote. It is to be noted that Arsenio Borjal was specifically named in the list of civilian officials to be prosecuted (Exhibit 12-b). In truth, the prosecution does not seriously dispute that the trial and sentencing of Borjal was done in accordance with instructions of superior military authorities, although it point to irregularities that were due more to ignorance of legal processes than personal animosity against Borjal. The state, however, predicates its case principally on the existence of the radiogram Exhibit H from Col. Volckmann, overall area commander, to Lt. Col. Arnold, specifically calling attention to the illegality of Borjal's conviction and sentence, and which the prosecution claims was known to the accused Beronilla. We have carefully examined the evidence on this important issue, and find no satisfactory proof that Beronilla did actually receive the radiogram Exhibit H or any copy thereof. The accused roundly denied it. The messenger, or "runner", Pedro Molina could not state what papers were enclosed in the package he delivered to Beronilla on that morning in question, nor could Francisco Bayquen (or Bayken), who claimed to have been present at the delivery of the message, state the contents thereof. The plain import of the affidavit is that the witness Rafael Balmaceda was not with Beronilla when the message arrived, otherwise Beronilla would have given him his orders direct, as he (Balmaceda) testified later at the trial. Moreover, it is difficult to believe that having learned of the contents of the Volckmann message, Balmaceda should not have relayed it to Borjal , or to some member of the latter's family, considering that they were relatives. In addition to Balmaceda was contradicted by Bayken, another prosecution witness, as to the hatching of the alleged conspiracy to kill Borjal. Balmaceda claimed that the accused-appellants decided to kill Borjal in the early evening of April 18, while Bayken testified that the agreement was made about ten o'clock in the morning, shortly after the accused had denied Borjal's petition to be allowed to hear mass. Our conclusion is that Lt. Col. Arnold, for some reason that can not now be ascertained, failed to transmit the Volckmann message to Beronilla. And this being so, the charge

of criminal conspiracy to do away with Borjal must be rejected, because the accused had no need to conspire against a man who was, to their knowledge, duly sentenced to death. 22. TABUENA VS. SANDIGANBAYAN 268 SCRA 332 (1997) Nature: This is a separate petition to review the decision of the Sandiganbayan dated October 12, 1990 convicting them of malversation under Article 217 of the Revised Penal Code as well as the Resolution dated December 20, 1991 denying reconsideration 3 criminal cases were filed against Tabuena who appears as the principal accused and one for Peralta since the total amount of 55 million pesos was taken on 3 separate dates of January, namely 10, 16, and 29. They were convicted of malversation under Article 217 of the Revised Penal Code. Facts: Then President Marcos instructed Tabuena over the phone to pay directly to the Presidents office and in cash what the MIAA owes the PNCC to which Tabuena replied. Yes, sir. I will do it. On January 8, 1986, one week after the phone conversation Tabuena received from Mrs. Gimenez, the personal secretary of Pres. Marcos, a Presidential Memorandum reiterating in black and white the verbal instruction of the President. In obedience to President Marcos verbal instruction and Memorandum, Tabuena, with the help of Dabao and Peralta caused the release of 55 million pesos of MIAA funds by means of 3 withdrawals. 1st withdrawal was made on January 10, 1986 for P25 million following a letter of even date signed by Tabuena and Dabao requesting the PNB extension office at the MIAA depository branch of MIAA funds to issue a manager check for said amount payable to Tabuena and was encashed at the PNB Villamor Branch. Subsequently, the P25 million in cash was delivered by Tabuena with the use of an armored car of the PNB to the office of Mrs. Gimenez located at Aguado St. fronting Malacanang. There was no receipt issued for the delivery of the money. Similar circumstances surrounded the second withdrawal/encashment and delivery of another P25 million made on January 16, 1986. The 3rd and last withdrawal was made on January 31, 1986 for P5 million. Peralta was Tabuenas co -signatory to the letter-request for the managers check for the amount Peralta did not go with Tabuena to deliver the money to the office of Mrs. Gimenez. It was only upon this delivery that Mrs. Gimenez issued a receipt, dated January 30, 1986, for all the amounts she received from Tabuena. Issue: 1. Whether the Sandiganbayan erred in convicting petitioners of a crime not charged in the amended informations 2. Whether the defense of Good Faith of the petitioners relieve then from the crime of malversation, and; 3. Whether the petitioners constitutional rights to due process was violated Held: 1. No. It is stated in Cabello vs. Sandiganbayan even on putative assumption that the evidence against petitioner yielded a case of malversation by negligence but the information was for intentional malversation, under the circumstances of this case his conviction under the first mode of misappropriation would still be in order. Malversation is committed either intentionally or by negligence. The dolo or the culpa present in the offense is only a modality in the perpetration of the felony. Even if the mode charged differs from the mode proved, the same offense of malversation is involved and conviction thereof is proper. Moreover, SEC. 5 Rule 116 of the Rules of court does not require that all the essential elements of the offense charged in the information be proved, it being sufficient that some of said

10

essential elements or ingredients thereof be established to constitute the crime proved. 2. Yes. Firstly, Marcos was undeniably. Tabuenas superior and as a recipient of such kind of a directive coming from the highest official of the land no less, GOOD FAITH should be read on Tabuenas compliance, without hesitation nor any question, with the MARCOS Memorandum. The superiorsubordinate relationship was clearly established and so is the lawfulness of the order contained in the MARCOS Memorandum for its purpose partial payment of the liability of one government agency (MIAA) to another (PNCC). Tabuena is therefore entitled to the justifying circumstance of Any person who acts in obedience to an order issued by a superior for some lawful purpose. Secondly, there is no denying that the disbursement did not comply with certain auditing rules and regulations. But this deviation was inevitable under the circumstances Tabuena was in because he did not have the luxury of time to observe all auditing procedures of disbursement considering the last that the MARCOS Memorandum enjoined his immediate compliance with the directive that he forwarded to the Presidents office P55 million in cash. Tabuena surely cannot escape responsibility for the emission but since he was acting in good faith, his liability should only be administrative or civil in nature and not criminal. Thirdly, the Sandiganbayan made the finding that Tabuena had already converted and misappropriated the P55 million when he delivered the same to Mrs. Gimenez and not to the PNCC however it was stated in the memorandum to pay immediately the PNCC, thru his office, the sum of 55 million pesos and that was what Tabuena precisely did when he delivered the money to the Presidents office thru his secretary Mrs. Gimenez. Therefore, Tabuenas good faith in delivering the money to the President in strict compliance with the memorandum was not at all affected even if it later turned out that PNCC never received the money. Fourthly, even assuming that the real and sole purpose behind the MARCOS Memorandum was to siphon-out public money for personal benefit of those then in power still, no criminal liability can be imputed to Tabuena. There is no showing that he had anything to do whatsoever with the execution of the MARCOS Memorandum. There is also no proof that he profited from the felonious scheme in short, no conspiracy was established between Tabuena and the real embezzlers of the P55 million. This is not a sheer case of blind and misguided obedience, but obedience in good faith of a duly executed order. 3. Yes. The cold neutrality of an impartial judge is an essential requirement for due process because it would show that the courts are impartial, and unbiased. Moreover, the trial judge has the right to question witness in view of satisfying his mind upon any material point which presents itself during the trial o a case over which he presides but it is limited only to clarifactory questions only. However, the appellate court, while going over the records, noticed that the way the Sandiganbayan actively took part in the questioning of a defense witness and of the accused themselves and basing on its manner indicated prejudgment of guilt, bias, hatred, or hostility against the said appellants. It would be noticed by the volume of questions hurled by the Sandiganbayan and more importantly, it is noticed that the questions were in the nature of cross-examinations characteristic of confrontation, probing, and insinuation. Decision: WHEREFORE, in view of the foregoing herein petitioners Luis A. Tabuena and Adolfo M. Peralta are hereby ACQUITTED of the crime of malversation as defined and penalized under Article 217 of the Revised Penal Code The Sandiganbayan Decision of October 12, 1990 and the Resolution dated December 20, 1991 are REVERSED and SET ASIDE. 23. PEOPLE VS. PUNO 105 SCRA 151 (1991) Nature: Automatic Review from the decision of Circuit Criminal Court of Pasug, Rizal. FACTS: At about two oclock in the afternoon of September 8, 1970, Ernesto Puno, 28, a jeepney driver, entered a bedroom in the house of Francisca Col (Aling Kikay), 72, a widow. The house was located in the area known as Little Baguio, Barrio

Tinajeros, Malabon, Rizal. On seeing Aling Kikay sitting in bed, Puno insulted her by saying Mangkukulam ka mambabarang. Mayroon kang bubuyog. Then, he repeatedly slapped her and struck her several times on the head with a hammer until she was dead. The assault was witnessed by Hilaria de la Cruz, 23, who was in the bedroom with the old woman, and by Lina Pajes, 27, a tenant of the adjoining room. They testified that Punos eyes were reddish. His look was baleful menacing. After the killing Puno went to the room of Lina where Hilaria had taken refuge and according to Hilaria, he made the following confession and threat: Huwag kayong magkakamaling tumawag ng pulis at sabihin ninyo na umalis kayo ng bahay at hindi ninyo alam kung sino ang pumatay sa matanda. And according to Lina, Puno said: Pinatay ko iyong matanda. Huwag kayong tumawag ng pulis. Pag tumawag kayo ng pulis kayo ang paghihigantihan ko. Puno, a native of Macabebe, Pampanga, who testified about five months after the killing, pretended that he did not remember having killed Aling Kikay. He believes that there are persons who are mangkukulam mambabarang and mambubuyog and that when one is victimized by those persons, his feet might shrink of his hands might, swell. Puno believes that a person harmed by a mambabarang might have a headache or a swelling nose and ears and can be cured only by a quack doctor (herbolaryo). Consequently, it is necessary to kill the mangkukulam and mambabarang. The trial court concluded that if Puno was a homicidal maniac who had gone berserk, he would have killed also Hilaria and Lina. The fact that he singled out Aling Kikay signified that he really disposed of her because he thought that she was a witch. RULING: WHEREFORE, the death penalty is set aside. The accused is sentenced to reclusion perpetua. The indemnity imposed by the trial court is affirmed. Costs de oficio. 24. PEOPLE VS. PAMBID G.R. NO. 124453 (MARCH 2000) Nature: Appeal from a decision of the Regional Trial Court of Quezon City Branch 81 Facts: This an appeal from the decision of the Regional Trial Court Branch 81 Quezon City, finding the accused appellant Jose C. Pambid guilty beyond reasonably doubt of two counts of statutory rape and sentencing him to suffer the penalty of reclusion perpetua with all the accessibly penalties and to indemnify the victim Maricon Delvie C. Grifalda at P50, 000 plus costs of each count of rape. Maricon told her mother on the two incidents of rape, that sometime in April and May 1993, when she was on the way home after having been told for an errand, the accused appellant, their neighbor, brought her to his house, and raped her. The accused appellant threatened to beat her if she would not comply and used a deadly weapon. During the time wherein the accused was already raping the victim, they were interrupted by the knock on the door by the mother of the accused. The other incident of rape happened on one morning when Maricon was going on the nearby store where there was no other people around. Joseph then carried her to the sofa of his house and raped her. After the accused was arrested, he pleaded not guilty and use alibi as means I escape by saying that he was not in his mothers house at that time but in Caloocan City on his fathers house. Issue: Whether or not the accused appellant is guilty of two counts of rape. Held: The Supreme Court held that on the first error assign by the accused that the victim is inconsistent of her statement. The Supreme Court held that this contention is without merit considering the revelation made by Maricon and her voluntary submission of the medical examination and willingness to undergo public trial where she was compelled to give details of the assault on her dignity. Her testimony was given respect by The Supreme Court considering his consistency and composure during the interrogation was commendable. Secondly, the defense that he was not in his mothers was not given credit. The defense of insanity that the accused was suffering schizophrenia and mind retardation was mere concoctions because he was not able to establish that indeed he was deprived of intelligence or

11

freedom of will. Lastly, the decision of the trial court in rendering the accused appellant in rendering the penalty reclusion perpetua with the use of the deadly weapon considering that it is punishable by reclusion perpetua to death, this is in consideration also on the suspension of the death penalty at the time that the crime was committed. The Supreme Court said that the trial court was wrong with this decision and since the accused did not object of charging two counts of rape in one information. The trial court failed to consider that the accused did not object to the validity of the information or raise the issue of duplicity of offense since the information does not charge him with more than one offense or occasion of rape. Decision: WHEREFORE, the decision of the RTC Branch 81. Quezon City is set aside and another one is rendered finding accused appellant Joseph Pambid guilty of one count of rape, sentencing him to suffer the penalty of reclusion perpetua and order him to pay Maricon P50, 000 as civil indemnity and the same amount on moral damages. 25. PEOPLE VS. DOQUENA 68 PHIL 580 (1939) Nature: Appeal from an order of the Court of First Instance of Pangasinan which held that the accused acted with discernment in committing the act imputed to him and ordered him sent to the Training School for Boys to remain therein until he reaches the age of majority. Facts: Between 1 and 2 oclock in the afternoon, Juan Ragojos and Epifanio Rarang were playing volleyball in the yard of the intermediate school. Valentin, who was also playing there intervened and catching the ball, tossed it to Juan Ragojos hitting him in the stomach. For his act, Juan chased Valentin upon overtaking him slapped him on the nape. Valentin then turned against Juan with a threatening attitude, so Juan struck him on the mouth with his fist. Juan then returned and continued playing with Epifanio. Valentin was offended and looked around for a stone to attack Juan. He found none so he approached a cousin named Romualdo Cecal to lend him his knife.Valentin approached Juan and challenged the latter to give him another blow with his fist to which Juan answered that he did not want to because he was bigger than Valentin. Juan ignored Valentin and continued playing. Valentin then stabbed him in the chest Issue: Whether or not Valentin acted with discernment Held: Decision is affirmed. Points to Ponder: Discernment is the mental capacity to understand the difference between right and wrong and such capacity may be known and should be determined by taking into consideration all the facts and circumstances afforded by the records in each case, the very appearance, the very attitude, the very comportment of the said minor, not only before and during the act, but also after and even during trial. 26. PEOPLE VS. LORENO 130 SCRA 311 (1984) Nature: Appeal on the conviction of the Court of First Instance of Camarines Sur charging Eustaquio Malaga and Jimmy Marantal of Robbery with Double Rape. Facts: On the evening of January 7, 1978, Barangay Captain Elias Monge was at his house with his two daughters, Monica, 14 years old and Cristina, 22 years old. His wife, Beata Monge, was still changing the diaper of baby Rachel Baybayon. The other occupants of their house that evening were Mario, 11 years old. Nilo, 13 years old and farm helper, Francisco Fable. At around 7:40pm, four men with flashlights approached their house calling Elias saying that there was a letter for the chief. Elias then invited the man with dark sweater to come inside the sala. Monica and Elias the read the letter Kami ang NPA. Elias was poked with a gun and they were all ordered to stay on the floor. Fable the recognized one of the men was Eustaquio Loreno. Loreno then tied him with rattan. Fable also recognized Jimmy Marantal as one of the lookouts.

The man in the dark sweater then dragged Monica to her room and succeeded in raping her. Below in the sala they could hear Monicas shouts for help, but Loreno pointed hid gun at them telling them not to rise off if they wanted to live. Loreno then brought Beata to the other rooms to get other contents. Then the man in the dark sweater returned and took Cristina and also raped her in one of the rooms. Moreover, Loreno entered the room and embraced her trying to kiss her and touch her private parts. The lower court found Loreno guilty of robbery with double rape sentencing him to life imprisonment. Jimmy Marantal was found guilty of robbery sentencing him to prison correccional as minimum of prison mayor. Issue: Whether or not the accused Loreno and Marantal were acting on irresistible force or uncontrollable fear Held: All facts demonstrated the voluntary participation and conspiracy of the appellants. All the conspirators are liable as co-principals regardless of the extend character of their participation because in the contemplation of law, the act of one is the act of all. Decision of the court is modified. Jimmy Marantal is sentenced to life imprisonment as well. 27. PEOPLE VS. FORONDA 222 SCRA 71 (1993) Nature:A review on the appealed decision of the Regional Trial Court of Cagayan finding the accused guilty beyond reasonable doubt for the crime of murder against the brothers Esminio and Edwin Balaan. Facts: At about 6:00am, June 11, 1986, the deceased Balaan brothers were taken by 7 armed men in fatigue with long firearms suspected to be NPA members, accompanied by the accused Rudy Fronda and Roderick Padua from the house of one Ferminio Balaan, at Brgy. Cataratan, Allacapan, Cagayan. Rudy Fronda and Roderick Padua were residents of the same places.The armed men tied the hands of the deceased at their backs, in front of their house.The armed men together with Fronda and Padua proceeded towards sitio Cataratan, Allacapan, Cagayan passing through the rice fields (taking along with them the Balaan brothers). Trial court found Fronda guilty as a principal by indispensable cooperation. (Testimony of Rudy Fronda) On the night of June 10, 1986, he was taken by the NPA from his house, accompanied by Robert Peralta, alias Ka Jun and Roderick Padua, to look for the Balaan brothers. There were around 9 NPAs with them. They found the Balaan brothers at the house of Ferminio Balaan, a brother. They tied their wrists/hands and brought them to the mountains of Sitio Tulong, Cataratan, Allacapan, Cagayan. After that, the NPA instructed them to go home, but the in afternoon of the same day, Robert Peralta, alias Ka Jun sent Elmer Martinez, Orlando Gonzales, George Peralta, and Librado Duran to get him and further he was ordered to get a spade and a crowbar. They were ordered to dig a hole in the mountain, one kilometer away from his house. Appellant interposes the exempting circumstance of uncontrollable fear (Art 12 [6] RPC) claiming that all his acts were performed under the impulse of uncontrollable fear and to save his life. Issue: W o N the accused-appellants acts make him liable for a principal by indispensable cooperation. Held: Decision modified. Accused-appellant could only be convicted as an accomplice of the crime. No incontrovertible proof was adduced by the prosecution supporting the conclusion that the appellant agreed with the members of the armed group to kill the Balaan brothers. Undoubtedly, even without appellants participation, the assailants could have easily located the Balaan brothers through the assistance of Roderick Padua. Taking account the numbers of the assailants alone, it is apparent that the armed men could have nevertheless committed the crime easily without the appellant abetting the commission thereof. As aforesaid to be considered principal by indispensable cooperation, there must be direct participation in the criminal

12

design by another act without which the crime could have not been committed. However, appellants act of joining the armed men in going to the mountains, and his failure to object to their unlawful orders, or show any reluctance in obeying the same, may be considered as circumstances evincing his concurrence with the objectives of the malefactors and had effectively supplied them with material and moral aid, thereby, making him as an accomplice. 28. PEOPLE VS. BANDIAN 63 PHIL 530 (1936) Nature: Charged with the crime of infanticide, convicted thereof and sentenced to reclusion perpetua and the corresponding accessory penalties, with the costs of the son. Josefina Bandian appealed from said sentence. Facts: At a0bout 7 oclock in the morning of January 31, 1936, Valentin Aguilar, the appellants neighbor, saw the appellant go to a thicket about four or five brazas from her house, apparently to respond to a call of nature because it was there that the people of the place used to go for that purpose. A few minutes later, he again saw her emerge from the thicket with her clothes stained with blood both in the front and back, staggering and visibly showing signs of not being able to support her. He ran to her and said, having noted that she was very weak and dizzy, he supported and helped her go up to her house and placed her in her own bed. Upon being asked before Aguilar brought her to her house, what had happened to her, the appellant merely answered that she was very dizzy. Not wishing to be alone with the appellant in such circumstances. Valentin Aguilar called Adriano Comcom, who lived nearby, to help them, and later requested him to take bamboo leaves to stop the hemorrhage which had come upon the appellant. Comcom had scarcely gone about five brazas when he saw the body of newborn baby near a path adjoining the thicket where the appellant had gone a few moments before Comcom informed Aguilar of it and the latter told him to bring the body to the appellants house. Upon being asked whether the baby which had just been shown to her was hers or not, the appellant answered in the affirmative. Upon being notified of the incident of 2 oclock in the afternoon of said day, Dr. Emilio Nepomuceno, president of the sanitary division of Talisayan, Oriental Misamis, went to the appellants house and found her lying in bed still bleeding. Her bed, the floor of her house and beneath it, directly under the bed, was full of blood. Basing his opinion upon said facts, the physician in question declared that the appellant gave birth in her house and in her own bed; that after giving birth she threw her child into the thicket to kill it for the purpose of concealing her dishonor from the man, Luis Kirol, with whom she had theretofore been living martially, because the child was not but of another man with whom she had previously had another relations. To give force to his conclusions, he testified that the appellant had admitted to turn that she had killed her child, when he went to her house at the time and on the date above-stated. Ruling: In conclusion, taking into account the foregoing facts and considerations, and granting that the appellant was aware of her involuntary childbirth in the thicket and that she later to take her child therefore, having been so prevented by reason of causes independent of her will, it should be held that the alleged errors attributed to the lower court by the appellant are and it appearing that under such circumstances said appellant has the fourth and seventh exempting circumstances in her favor, she is hereby acquitted of the crime of which she had been accused and convicted with costs de oficio, and as she is actually confined in jail in connection with the case it is ordered that she be released immediately.

indemnify the heirs of Felix Napola in the sum of twelve thousand pesos and to pay the costs. FACTS: Upon arrival of Brigido Alberto in the municipal building at around eight o'clock, he witnessed an extraordinary occurrence. He saw Policeman Ural (with whom he was already acquainted) inside the jail. Ural was boxing the detention prisoner, Felix Napola. As a consequence of the fistic blows, Napola collapsed on the floor. Ural, the tormentor, stepped on his prostrate body. Ural went out of the cell. After a short interval, he returned with a bottle. He poured its contents on Napola's recumbent body. Then, he ignited it with a match and left the cell. Napola screamed in agony. He shouted for help. Nobody came to succor him. Much perturbed by the barbarity which he had just seen, Alberto left the municipal building. Before his departure, Ural cautioned him: "You better keep quiet of what I have done" (sic). Alberto did not sleep anymore that night. From the municipal building, he went to the crossing, where the cargo trucks passed. He hitchhiked in a truck hauling iron ore and went home. Doctor Luzonia R. Bakil, the municipal health officer, certified that the thirty-year old victim, whom she treated twice, sustained second-degree burns on the arms, neck, left side of the face and one-half of the body including the back (Exh. A). She testified that his dermis and epidermis were burned. If the burns were not properly treated, death would ensue from toxemia and tetanus infection. "Without any medical intervention", the burns would cause death", she said. She explained that, because there was water in the burnt area, secondary infection would set in, or there would be complications. Napola died on August 25, 1966. The sanitary inspector issued a certificate of death indicating "burn" as the cause of death (Exh. B). ISSUE: Whether or not the accused committed the crime as guilty beyond reasonable doubt? HELD: Lack of intent to commit so grave a wrong offsets the generic aggravating, circumstance of abuse of his official position. The trial court properly imposed the penalty of reclusion perpetua which is the medium period of the penalty for murder (Arts. 64[4] and 248, Revised Penal Code). Finding no error in the trial court's judgment, the same is affirmed with costs against the appellant. RULING: This case is covered by article 4 of the Revised Penal code which provides that "criminal liability shall be incurred by any person committing a felony (delito) although the wrongful act done be different from that which he intended". The presumption is "that a person intends the ordinary consequences of his voluntary act" (Sec. 5[c], Rule 131, Rules of Court). There is a rule that "an individual who unlawfully inflicts wounds upon another person, which result in the death of the latter, is guilty of the crime of homicide, and the fact that the injured person did not receive proper medical attendance does not affect the criminal responsibility" (U.S. vs. Escalona, 12 Phil. 54). In the Escalona case, the victim was wounded on the wrist. It would not have caused death had it been properly treated. The victim died sixty days after the infliction of the wound. It was held that lack of medical care could not be attributed to the wounded man. The person who inflicted the wound was responsible for the result thereof. The crime committed by appellant Ural was murder by means of fire (incendio) (Par. 3, Art. 248, Revised Penal Code; People vs. Masin, 64 Phil. 757; U.S. vs. Burns, 41 Phil. 418, 432, 440). The trial court correctly held that the accused took advantage of his public position (Par. 1, Art. 14, Revised Penal Code). He could not have maltreated Napola if he was not a policeman on guard duty. Because of his position, he had access to the cell where Napola was confined. The prisoner was under his custody. "The policeman, who taking advantage of his public position maltreats a private citizen, merits no judicial leniency." The methods sanctioned by medieval practice are surely not appropriate for an enlightened democratic civilization. While the law protects the police officer in the proper discharge of his duties, it must at the same time just as effectively protect the individual from the abuse of the police." U.S. vs. Pabalan, 37 Phil. 352).

29. PEOPLE VS. URAL 56 SCRA 138 (1974) NATURE: An appeal of defendant Domingo Ural from the decision of Judge Vicente G. Ericta of the Court of First Instance of Zamboanga del Sur, convicting him of murder, sentencing him to reclusion perpetua, and ordering him to

13

But the trial court failed to appreciate the mitigating circumstance "that the offender had no intention to commit so grave a wrong as that committed" (Par. 3, Art. 13, Revised Penal Code). It is manifest from the proven facts that appellant Ural had no intent to kill Napola. His design was only to maltreat him may be because in his drunken condition he was making a nuisance of himself inside the detention cell. When Ural realized the fearful consequences of his felonious act, he allowed Napola to secure medical treatment at the municipal dispensary. 30. PEOPLE VS. REGATO 127 SCRA 287 (1984) Nature: For automatic review is death sentence imposed on accused-appellants Miguel Regato and Jose Salceda by the then Court of First Instance of Leyte, Branch IV, in Criminal Case No.12, entitled People vs. Miguel Regato, et al., for robbery with homicide. They were also ordered to indemnify, jointly and severally, the heirs of Victor Flores the sum of P 12,000.00; the further sum of P 8,000.00 and each to pay one third of the costs. Issue: In this appeal, appellants contend that the trial court erred (1)when it denied Salcedos motion for new trial and did not acquit him of the crime charged; (2) in convicting Regato of robbery with homicide and not with simple robbery; (3) in not considering in their favor the mitigating circumstance of lack of intent to commit so grave a wrong as that committed; (4) in considering the aggravating circumstance of nocturnity against them and (5) in failing to consider that the aggravating circumstance of craft is absorbed by the aggravating circumstance of nocturnity. Facts: About nine oclock in the evening of November 22, 1969, three persons called at the house of Victor Flores at Sitio Macaranas, Bo. Capirawa, Palo Leyte asking if they could buy cigarettes. Felicisima Flores, wife of Victor, was then maintaining a small sari-sari store inside their house. Upon hearing them, she stood up and after lighting a small kerosene lamp, opened the door of the house and extended the lamp out to recognize the persons outside. She saw accused Miguel Regato who was then at the porch and Jose Salceda. As she kept on exposing the light at them, Regato approached Felicisima and struck her hand holding the lamp, causing it to fall. Regato then pointed a gun at Felicisima who moved backwards, towards the kitchen after which she jumped out and ran to the house of Filomeno Pilmaco, a neighbor. She asked for help and was told to sty in the house while he and his companions would rush to poblacion of Palo to inform the police if the incident. After Pilmaco and his companions had left, Felicisima heard a gun explosion from the direction of their house. In the meantime, Godofredo Flores, the 12 years old son of Felicisima, who was sleeping in the sala, was awakened by the voice of the robbers. He observed that his mother was not in the house but his father was being dragged down the stairway by Rito Ramirez and Miguel Regato. He saw also appellant Salceda then lighted the lamp which was then on the floor of the sala of the house and then he brought Florencio inside the bedroom where Godofredo was then hiding. Rito Ramirez and appellant Regato in turn, brought Victor Flores inside the sala. Thereafter, Regato hit Victor Flores with the butt of his gun and said: Where is your money? Where is your money? When Victor answered that they do not have any, Rito Ramirez boxed Victor at the mouth breaking one of his teeth. While Victor was being maltreated by Rito and Regato force him to reveal where their money was, Salceda was busy ransacking a trunk inside the bedroom which contain P 870.00 in a box. Salceda took and went to the kitchen. He told Ramirez that he had the money and Ramirez hit the man as he was angry for Victor telling them earlier that he does not have money. Ramirez shot Victor Flores following which Regato, Salceda and Ramirez rushed out the house and fled. After some minutes, Felicisima Flores went back and found her husband bleeding. Things inside the bedroom and found the money inside the trunk gone. With the help of a nephew, Victor was brought to the poblacion of Palo. On the way, they met a police patrol which proceeded to the scene of the robbery. The party of Victor reached the municipal building of Palo, Leyte about midnight of November 22 and few minutes

thereafter, he gave a written statement which is now marked as Exhibit C. The following morning, Victor was admitted at the Leyte Provincial Hospital but due to severe hemorrhage, secondary to gunshot wound, he died the same day. Felicisima was formally investigated by the police to whom she gave her affidavit now marked as Exhibit F. Jose Salceda on November 26, 1969 was brought to the police department as a suspect in the case. He was identified by Felicisima Flores. Regato was likewise apprehended and a case against the three. Miguel Regato, Jose Salceda and Rito Ramirez were filed for Robbery with Homicide. The case was tried against Regato and Salceda only because Rito Ramirez remains at large. The defense is in denial and alibi. Regato claimed that on the night of November 22, 1969, he was in he Gacao, Palo, Leyte attending a novena prayers for his late father-in-law were his testimony corroborated by the defense witnesses. Salceda, on the other hand testified that in the morning of November 22, 2969 he was in Bo. Gacao, Palo to transport palay. In that evening he had a drinking spree with his group until 2 oclock in the following morning.

HELD: WHEREFORE, the judgment appealed from is AFFIRMED except as to the penalty, which is hereby modified to reclusion perpetua. RATIONALE: We find no merit in the 2nd assigned error. Appellants with Ramirez arrived together at the residence of Victor Flores and pretended to buy cigarettes. When Felicisima Flores opened the door, they went inside the house and demanded Victor Flores to bring out the money. When he refused, Ramirez and Regato maltreated him while Salceda went inside the bedroom and ransacked he trunk where the money was kept. Ramirez then inquired whether he found the money and he answered in the affirmative. It is the time that the shooting of Victor Flores tool place after the money had been taken and it was only when Flores called them robbers that Ramirez shot him. As partly stated by the lower court, It is clear that the killing was done by reason or on the occasion of the robbery, that the accused are guilty of the special complex crime of robbery with homicide. Likewise, we find no merit in the contention that there was lack of intent to commit so grave a wrong as that committed. Intention is a mental process and is an internal state of mind, the intention must be judged by his action, conduct and external acts of the accused. What men do is the best index of their intention. IN the case at bar, the aforesaid mitigating circumstance cannot be appreciated considering that the acts employed by the accused were reasonable sufficient to produce the result they actually made death of the victim. With the respect to the fourth and fifth assigned errors, nighttime and craft, the evidence is clear that the crime was committed past 9:00 in the evening which the culprits had especially sought he hiding mantle of the night I order to facilitate its commission. Craft involves intellectual trickery or cunning on the part of the accused. Herein appellants, in order to enter the house of Flores, shouted from the outside that they wanted to buy cigarettes, which induced the inmates to open the door for the, As held in People vs. Napili, 85 Phil 521, gaining entrance by pretending to buy cigarettes or drink water constitutes craft. By and large, the conspiracy among appellants and Ramirez in the commission of the crime is evident upon the facts as prove. Their acts, collectively and individually executed, have demonstrated the existence of a common design towards the accomplishment of unlawful purpose and objective. The shooting and death of Victor Flores bear a direct relation and intimate connection between the robbery and the killing which occurred during and on the occasion of the robbery. Whether the latter be prior or subsequent to the former, there is no doubt that he complex crime of robbery with homicide has been committed. On the other hand, appellants alibi must fall. In the first place, Regatos submittal that he should have been convicted of simple robbery only, instead of robbery with homicide is an admission of his presence at the scene of the

14

crime contrary to his testimony that he was in his house that evening of November 22, 1969 attending to novena prayers for his late father-in-law. Secondly, to establish an alibi, it is not enough to prove that appellants were at some other place when the crime was committed but must, likewise demonstrate that it was physically impossible for them to have been at the place of commission at such time. The distance between the place of the commission of the offense to the place where appellants were supposed to be at the time is only 1 kilometers, and these places are connected with passable roads that could have facilitated the easy negotiation by the appellants between their respective homes and the scene of the crime. Appellants evidence on this point is not sufficient to overcome the positive identification made by the prosecution witnesses Felicisima Flores and Godofredo Flores. 31. PEOPLE VS. PAGAL 79 SCRA 570 (1977) Nature: This is an appeal on the Criminal Case No. CCC-VI-5 (70) of the Circuit Criminal Court of Manila, wherein the accused, Pedro Pagal y Marcelino and Jose Torcelino y Torazo, were charged with the crime of robbery with homicide. Facts: That on or about December 26, 1969, in the City of Manila, Philippines, the said accused, conspiring and confederating together and mutually helping each other, did then and there wilfully, unlawfully and feloniously, with intent to gain, and by means of violence, take away from the person of one Gau Guan, cash amounting to P1,281.00, Philippine currency, to the damage and prejudice of the said Gau Guan in the said sum of P1,281.00. That on the occasion of the said robbery and for the purpose of enabling them to take, steal and carry away the said amount of P1,281.00, the herein accused, in pursuance of their conspiracy, did then and there wilfully, unlawfully and feloniously, with intent to kill and taking advantage of their superior strength, treacherously attack, assault and use personal violence upon the said Gau Guan, by then and there stabbing him with an icepick and clubbing him with an iron pipe on different parts of his body, thereby inflicting upon him mortal wounds which were the direct and immediate cause of his death thereafter. When the case was called for arraignment, counsel de oficio for the accused informed said court of their intention to enter a plea of guilty provided that they be allowed afterwards to prove the mitigating circumstances of sufficient provocation or threat on the part of the offended party immediately preceding the act, and that of having acted upon an impulse so powerful as to produce passion and obfuscation. Held: Since the alleged provocation which caused the obfuscation of the appellants arose from the same incident, that is, the alleged maltreatment and/or ill treatment of the appellants by the deceased, these two mitigating circumstances cannot be considered as two distinct and separate circumstances but should be treated as one. Secondly, the circumstance of passion and obfuscation cannot be mitigating in a crime which as in the case at bar is planned and calmly meditated before its execution. Finally, the appellants claim that the trial court erred in considering the aggravating circumstances of nighttime, evident premeditation, and disregard of the respect due the offended party on account of his rank and age. Although the trial court correctly considered the aggravating circumstance of nocturnity because the same was purposely and deliberately sought by the appellants to facilitate the commission of the crime, nevertheless, We disagree with its conclusion that evident premeditation and disregard of the respect due the offended party were present in the commission of the crime. Evident premeditation is inherent in the crime of robbery. However, in the crime of robbery with homicide, if there is evident premeditation to kill besides stealing, it is considered as an aggravating circumstance. In other words, evident premeditation will only be aggravating in a complex crime of robbery with homicide if it is proved that the plan is not only to rob, but also to kill. In the case at bar, a perusal of the written statements of the appellants before the police investigators show that then original plan was only to rob, and that, they killed the deceased only when the latter refused to open the "kaha de yero", and fought with them. The trial court, therefore, erred in taking into

consideration the premeditation.

aggravating

circumstance

of

evident

32. PEOPLE VS. PARANA 64 PHIL 331 (1937) NATURE: The accused Primo Parana appeals from the judgment of the Court of First Instance of Occidental Negros sentencing him, for the crime of murder committed on the person of Manuel Montinola, to the penalty of reclusion perpetua and to infirmity the heirs of the deceased in the sum of P1, 000.00 with costs. FACTS: On the morning of May 19, 1936, in the municipality of Silay, Occidental Negros, the deceased, who was taking part in a game of monte in the house of Jose Lapuos, was informed by the chauffeur Valentin Poblacion that his brother Glicerio Montinolas car which he had ordered for his trip to the municipality of Cadiz, was ready to start. Five minutes later the deceased came downstairs and upon reaching the street, he turned towards the car which was waiting for him. At that moment the chauffeur Poblacion, who saw the appellant behind the deceased in the attitude of stabbing him with a dagger shouted to warn him of the danger, and the deceaseds looking behind, really saw the appellant about to stab him. The deceased, defending himself retreated until he fell on his back into a ditch two meters wide and 1.7 meters deep. Without lessening the aggression on the appellant mounted astride of the deceased and continued to stab him with the dagger. After the appellant and the deceased had been separated, the former still asked Montelibano for the weapon taken from him but at that moment a policeman arrived and the appellant was placed under arrest. When the deceased was later removed from the ditch into which he had fallen, he was found wounded and was taken to the provincial hospital where he was treated by Dr. Ochoa, expiring six days later, as a result of general peritonitis produces by one of his wounds. The preceding nights, at about 11oclosk, monte had also been played in the house of Glicerio Montinola, brother of the deceased. The deceased took part in said game where the appellant was designated to attend to the players. One Lamay, who was also taking part in the game, gave appellant the sum of P2 to buy beer. For failure of the appellant to immediately comply with this request, a discussion ensued between him and Lamay and, as both raised their voices, they were admonished by the deceased. As the appellant disregarded said admonition, the deceased slapped him and ordered him to leave the house. The appellant left and went to Lapuos house where he lived, where the deceased took part in another game on the following day, and where said deceased came from when he was attacked. At about 7 oclock in the morning of the crime, the appellant purchased from the store of the Japanese Matzu Akisama, a hunting knife (Exhibit F) which is the same knife used by him in attacking the deceased. On the same morning, at about seven thirty, the appellant went to the house of Crispin Espacio for who he used to work to ask to wreak vengeance on somebody. Espacio advised him against it as he might again go to Bilibid prison, inasmuch as he had already served a term for the crime homicide. The appellants testimony is the only evidence in his defense. According to him, on the morning of the crime he saw the deceased taking part in the game in Lapuos house where he lived. The deceased then uttered threatening words to him which he disregarded, leaving the house and going to a nearby Chinese store. Sometime, later as he was on his way for Lapuos house, he saw the deceased coming down and approaching the latter, he spoke to him about the incident of the previous night and of their meeting a few minutes before asking said deceased to forgive and met wreak vengeance on him. The deceased, by way of an answer drew the revolver which he carried on his belt, and the appellant in the efface suck attitude, attempted to wrest the weapon from him. In the struggle the deceased fell on his back into a ditch and the appellant mounted astride of him tried to wrest the revolver from him, and at the same time drew the knife which he carried, attacking the deceased therewith. When the appellant had succeeded in taking possession of the revolver, the deceased got up and walked towards the car. At that moment

15

Liboro Montelibano appeared and the appellant turned over the knife and the revolver to him. The version of the incident given by the appellant in his testimony, without any corroboration is contradicted by the testimony of the chauffeur Poblacion and of Liboro Montelibano. Furthermore, it is improbable taking into consideration the fact that he was the offended [arty, suffering from the injustice of the offense received, provided himself with lethal weapon and approached the deceased, which circumstances do not agree with his attitude according to his testimony. HELD: The court finds the appellant guilty of the crime of murder qualified by treachery and taking into consideration the presence of one aggravating and two mitigating circumstances in the commission of the crime and applying the Indeterminate Sentence Law, Act No. 4103, he is sentenced to the penalty of from ten years of prision mayor, as the minimum, to seventeen years, four months and one day of reclusion temporal, as the maximum, affirming the appealed sentence in all other respects with the costs. RATIONALE: The court correctly found that the qualifying circumstance of treachery was present in the commission of the crime. The appellant, inspite of having seen the deceased in the upper story of Lapuos' house, did nor wish to attack the latter there undoubtedly to avid his being defended by the many players who were with him. Instead, he waited for the deceased at the nearby store until the latter came down, and attack him while he had his back turned and could not see the appellant. All these, which were the beginning of the execution of the appellants design to kill ye deceased, constitute treachery inasmuch as they tended to avoid every risk to himself arising from the defense which the deceased might make. The aggravating circumstance that the appellant is a recidivist must be taken into consideration. The mitigating circumstance that he had acted in the immediate vindication of a grave offense committed against him a few hours before, when he was slapped by the deceased in the presence of many persons, must likewise be taken into consideration. Although this offense, which engenders perturbation of mind, was not so immediate, that the influence thereof, by reason o it gravity and the circumstances under which it was inflicted, lasted until the moment the crime was continued. Lastly, the other mitigating circumstance that the appellant had voluntarily surrendered himself to the agents of the authorities must be considered. 33. PEOPLE VS. DIOKNO 63 PHIL 601 (1936) Nature: Appeal from a judgment of the CFI (Court of First Instance now Regional Trial Court) of Laguna. Facts: The deceased Yu Hiong was a vendor of sundry goods in Lucena, Tayabas. At 7 oclock in the morning of January $, 1935, Salome Diokno, to whom Yu Hiong was engaged for about a year, invited the latter to go with her. Yu Hiong accepted the invitation but he told Salome that her father was angry with him. At about 6 oclock in the afternoon of said day, Yu Hiong and Salome Diokno took an automobile and went to the house of Vicente Verina, Salomes cousin in Pagbilao. On January 5th or 6th of said year, Roman Diokno telegraphed his father Epifanio Diokno, who was in Manila, informing him that Salome had eloped with the Chinese Yu Hiong, on the morning of January 7, 1935, Epifanio and Roman Diokno went to San Pablo, in search of the elopers. Upon arriving near the house, they saw Yu Hiong coming down the stairs. When Yu Hiong saw them, he ran upstairs and they pursued him. At the moment, he was overtaken by the accused that carried knives locally known as balisong, of different size. Yu Hiong fell on his knees and implored pardon. Roman Diokno stabbed him with the knife I the back and the later in the left side. Epifanio stabbed at once. Issue: Whether or not said accused were physically stronger than the deceased? And whether or not the said accused abused such superiority?

Held: Neither does this court find the existence of the other circumstance qualifying murder, that is, evident premeditation proven beyond a reasonable doubt. In order that premeditation may be considered either as an aggravating or as an qualifying circumstance, it must be evident, that5 is the intention to kill must be manifest and it must have been planned in the mind of the offender and carefully meditated therefore, there having been neither abuse of superior strength nor evident premeditation, the crime committed by the accused is simple homicide. The presence of 5th mitigating circumstance of Article 13 of the Revised Penal Code maybe taken into consideration in favor of the two accused, because the offense did not cease while Salomes whereabouts remained unknown and her marriage to the deceased unlegalized. Therefore, there was no interruption from the time of offense was committed to the vindication thereof. The presence of the 6th mitigating circumstance of Article 13 of the RPC may also be taken into consideration in favor of the accused. The fact that the accused saw the deceased ran upstairs when he became aware of their presence, as if he refused to deal with them after having gravely offended him. The 7h mitigating circumstance of Article 13 of the RPC should also be taken in favor of the accused Epifanio Diokno. The court conclude that accused are guilty of reasonable beyond reasonable doubt of the crime of homicide, the penalty therein being reclusion temporal in its full extent, 3 mitigating circumstances must be taken into consideration in favor of the accused Epifanio and 2 mitigating circumstances in favor of the accused Roman Diokno with no aggravating circumstances. Both accused should be granted the benefits of the indeterminate sentence which prescribe a penalty the minimum of which shall be taken from the next lower to prision mayor, or prision correccional of from 6 months and 1 day to 6 years. Taking into account the circumstances of the case, the indeterminate penalty to which each of said accused must be sentenced is fixed at from 2 years and 1 day of prision correccional. Wherefore, the accused was guilty of the crime of homicide and sentence each of them to an indeterminate penalty from 2 years, 1 day of prison correccional to 8 years, 1 day of prision mayor. 34. PEOPLE VS. MUIT 117 SCRA 696 (1982) NATURE: Appeal from the Decision of the Court of First Instance of Camarines Sur, Branch I (Naga City), in Criminal Case No. R-7 (1847), convicting the accused Delfin Muit, a retired PC 2nd Lieutenant, of Murder, and sentencing him to suffer the penalty of reclusion perpetual for the gunning down of the victim, Rodolfo Torrero. FACTS: On February 26, 1976 at about 2:45 o'clock in the afternoon, while the deceased Rodolfo Torrero, his family and friends, were passing by the house of accused Delfin Muit at barrio Tamban, Tinambac, Camarines Sur, on their Nay home from a picnic, the accused invited them to his house to take a rest. At that time Muit was alone as his wife and children were not around. Once the group was inside his house, accused Muit who is a retired PC lieutenant remarked that his invitation showed that he had no ill-feeling against the Torreros and that he knew the latter had no ill-feeling also against him. As they engaged in some amenities, a group of barangay members and PC authorities in !barge of the sanitation and cleanliness program on that particular lay, which was Community Day, paid them a visit, and after a short talk, said group left. The deceased Torrero conducted the group on their way out and upon his return; accused Muit requested him to take a seat. The accused then confronted Rodolfo Torrero why the latter always visits his wife even during nighttime and why he often invites her out. Torrero replied that being the barangay zone auditor, he had to confer with the accused's wife on barangay matters as the latter was the barangay zone president. The accused then asked why Torrero even gave food and money to his (accused's) children if he had no bad intention at all on his wife. Torrero's wife answered that they did it out of pity because there were times when they would see the accused's children in need of food and money. The accused, however, angrily stood up and countered, "Why should you give when your husband

16

had also a family to support? To avoid any trouble, the deceased Torrero likewise stood up and said, 'If that is the way we talked about this will end to nothing, so it is better that I should leave', and he proceeded to move out of the house. When Torrero was already outside the house of the accused and while walking along the pathway, the accused followed him and on reaching the door the accused shouted, "Wait because we have not yet finished". At that instant, the accused raised his left hand towards Torrero and with his right hand; he pulled out his .45 caliber pistol and aimed it at the deceased. Angrily, he fired his gun at Torrero who was just 3 meters away, hitting the latter at the lower left side below the nape. On being hit by the bullet, Torrero spun from his left to the right, with his two hands inclined to the right, his face writhing in pain, his left elbow raised parallel to his armpit and his right hand placed on his breast. Upon hearing the gunshot, witness Gubatan immediately grabbed and held the accused from behind with an embrace, and said, "Manoy Delfin, why are you like that? But as soon as Gubatan embraced the accused from behind, a second shot was fired, this time hitting the elevated left hand of Torrero, with the bullet penetrating through the breast. Consequently, Torrero fell on his knees, bent forward with face downward and body in a prone position his left elbow supporting him on his left lap while his right hand extended to the ground. Witness Gubatan on the other hand tightened his grip around the accused as he tried to wrestle with him. The wife of Torrero, who was shocked by the first shot thereupon rushed towards her fallen husband. But the accused on seeing Mrs. Torrero rushed towards the deceased, aimed his gun at her. Fortunately, witness Gubatan quickly grabbed the right forearm of the accused that held the gun and jerked it upward so that the third shot was fired towards the sky, thus missing its target Id). Witness Gubatan then said, "Manoy Delfin that is enough". Gubatan thereafter moved the accused away and brought him near a coconut tree. Mrs. Torrero, on the other hand, hugged her husband and cried for help, even as blood was oozing out from the deceased's body and mouth. Shortly thereafter, Torrero died. Near the coconut tree, the accused tried to free himself from the hold of Gubatan. He even pointed his gun at Gubatan and said, "Set me free Benny or I will shoot you. When Gubatan could no longer hold the accused as the latter kept on struggling, he let him go and said, "Alright Kuya Delfin, shoot me, after all I have no fault". Slowly, the accused put down his arm, his eyes at static condition. He (appellant) then started to move away, and as he did, he made a short last look at his victim, after which, he continued on his way. After the incident, the accused proceeded to the PC detachment to surrender himself and his gun. ISSUE: Whether or not the accused can involve passion and obfuscation as mitigating circumstance? HELD: Modifying the judgment appealed from, the accused Delfin Muit, is hereby sentenced to suffer the indeterminate penalty of eight (8) years of prision mayor, as minimum, to fourteen (14) years and eight (8) months of reclusion temporal, as maximum, with the judgment being affirmed in an other respects. RATIONALE: During the trial the accused involved passion and obfuscation as mitigating circumstance. There can be no question that the accused was driven strongly buy jealousy because of rumors regarding the amorous relationship between his wife and the victim. The feeling of resentment resulting from rivalry in amorous relation with a woman is a powerful stimulant from rivalry and amorous relation with a woman is a powerful stimulant to jealously and is sufficient to produce loss of reason and self-control. In other words, it is a powerful instigation of jealously and prone to produce anger and obfuscation. 35. PEOPLE VS. AQUINO G.R. O. 128887 (JAN. 2000) NATURE: Appeal from a decision of the Regional Trial Court of Olongapo City, Branch 75

FACTS: On January 19, 1996, Roselyn Lampera, daughter of Valerio and Esmeralda Lampera was in their house, together with her mother, younger brother Daniel and younger sister. Their house is like a small cubicle without any partitions, elevated from the ground by about 2 1/2 feet. In the morning of that fateful day, Roselyn's mother, Esmeralda, was in their house taking care of Roselyn's younger sister who was sick at the time. Her younger brother, on the other hand, was playing on the ground near their house. Appellant Edgardo Aquino (who was their neighbor) arrived, looking for their father. Both Roselyn and her mother informed Edgardo that Valerio, Roselyn's father, was in Olongapo. Unsatisfied with their answer, Edgardo (who was near the door at the time) peeped in their house and when he did not see Valerio, pulled out his knife. Initially, he tried to stab Roselyn's younger brother. When Roselyn and her mother saw this, they rushed towards the younger boy in an attempt to protect him. When Edgardo saw their reaction, Edgardo stepped inside their house, eager to vent his ire on Roselyn, intending to stab her. Roselyn's mother pulled her aside, shouting. Edgardo went for her mother who tried valiantly to evade his thrust as she was then carrying Roselyn's sick younger sister. Roselyn saw Edgardo repeatedly stab her mother in the latter's stomach and chest areas. Out of fear, Roselyn managed to destroy their nipa wall and jumped out of their house. Despite her shouts for help, no help came. At about the same time also, Benjamin Costimiano, a purok leader, was in his house when he heard some kind of shouting or commotion. Being a purok leader, he went to the place of incident and saw the victim. He heard the people there say that the culprit was Edgardo Aquino. He went after Edgardo and was able to catch up with him in the house of one Francisco Franco. Benjamin asked Edgardo (who was still armed with a knife at that time) to put down the knife and the latter gave him the knife. Benjamin described the knife used as a double-bladed one, and when it was handed to him, the handle still had some blood on it. Dr. Nancy Valdez, Medico-legal Officer III of the San Marcelino District Hospital, testified that she was the one who conducted the autopsy on the cadaver of the victim. She noted four (4) stab wounds at the xiphoid processes/chest area, two (2) of which were fatal as they penetrated the thoracic cavity, causing lacerations on the anterior portion of the superior lobe of the left lung. Valerio Lampera, Esmeralda's husband, declared that the untimely death of Esmeralda caused him pain and compelled the family to incur expenses in the amount of P2, 500. Daniel Isaac, Esmeralda's 8-year-old son, was likewise psychologically and emotionally affected by the unexpected demise of his mother. He cried on the witness stand when asked of the whereabouts of his mother. EDGARDO had another story to tell. According to him, Esmeralda's husband was his business partner in the sale of fish. In the evening of 19 January 1996, he went to the house of the Lamperas to get his capital for the business. He saw Roselyn standing by the stairs of the house and asked her about the whereabouts of her father Valerio. When she informed him that Valerio was not there, he left for the store of Francisco Franco. On his way to the store, he heard shouts coming from the Lampera's house, which he mistook to be just another ordinary fight. He proceeded to Franco's store. Then Benjamin Costimiano, a purok leader, arrived at the store, carrying with him a knife which, according to him, was recovered from inside Esmeralda's house. Benjamin invited EDGARDO to go with him to the Police Department of Subic, Zambales. Upon arrival thereat Costimiano ordered the detention of EDGARDO allegedly because the latter was a suspect in the killing of Esmeralda. EDGARDO was detained for two months but was not investigated by the police. He could not remember having been brought to the office of the Provincial Prosecutor and having given a statement thereat. He insisted that he did not kill Esmeralda and that the knife presented by the prosecution was not taken from him. Besides, he had no reason to kill the wife of his business partner ISSUE: Whether or not that treachery can be appreciated as aggravating circumstance; and temporary insanity, passion and obfuscation, intoxication, and voluntary surrender as mitigating circumstances in the crime committed by the accused

17

HELD: The challenged decision of Branch 75 of the Regional Trial Court of Olongapo City in Criminal Case No. 56-96 is MODIFIED. As modified, accused-appellant EDGARDO AQUINO y PUMAWAN is found guilty beyond reasonable doubt, as principal, of the crime of homicide, defined and penalized under Article 249 of the Revised Penal Code, and is hereby sentenced to suffer an indeterminate penalty ranging from eight (8) years and one (1) day of prision mayor as minimum to seventeen (17) years and four (4) months of reclusion temporal as maximum. The awards of P50, 000 as indemnity, P50, 000 as moral damages, P30, 000 as exemplary damages, and P2, 500 as actual damages stand RATIONALE: There was treachery in view of the sudden and unexpected attack upon the unarmed victim, who had not committed the slightest provocation and who was totally unaware of EDGARDO's murderous designs. Neither the victim nor her children anticipated the attack. EDGARDO did not give any warning that he was about to start a stabbing spree. The victim, then carrying a sick child, never had the chance to defend her or to retaliate. All that she managed to do was to try to evade EDGARDO's knife blows. In this case, the victim, Esmeralda, was forewarned of the impending attack on her, since it was preceded by EDGARDO's attempts to attack her son and daughter. It cannot be said that she was in no position to defend her; for, in fact, she succeeded in repelling appellant's aggression against her children. When EDGARDO turned to her, she "tried to evade the thrust" causing her 6-year-old child whom she was carrying to be thrown away. Furthermore, there is no sufficient evidence that the appellant deliberately and consciously adopted the means of execution employed by him. What is apparent is that the killing was done impulsively or on the spur of the moment. "Temporary insanity" is not recognized in this jurisdiction and that mere abnormality of the mental faculties will not exclude imputability. In any case, EDGARDO had the burden of proving his alleged "temporary insanity," as it is a basic principle in our rules on evidence that he who alleges a fact must prove the truth thereof. However, he did not raise this argument below, and it is only now that he belatedly raises it. Anent EDCARDO's claim of the mitigating circumstance of passion or obfuscation, the same is bereft of merit because his acts did not result from an impulse arising from lawful sentiments but from a spirit of lawlessness. We disagree with the trial court in appreciating in appellant's favor the mitigating circumstance of intoxication, EDGARDO declared that he drank liquor on the day of the incident in question, and the trial court held that his intoxication was corroborated by Roselyn's testimony that EDGARDO's eyes were "red" when she saw him. For intoxication to be mitigating, the following conditions must be present: (1) the same is not habitual or is not subsequent to the plan of the commission of a felony; otherwise, it is aggravating if it is habitual and intentional; and (2) the consumption of alcoholic drinks was in such quantity as to blur the accused's reason and deprive him of a certain degree of control. In this case, EDGARDO was unable to prove both requisites. Nevertheless, we appreciate in EDGARDO's favor the mitigating circumstance of voluntary surrender. Immediately after the incident, when purok leader Benjamin Costimiano followed him in the house of Francisco Franco, EDGARDO voluntarily gave the knife to Franco and went with the latter to the Police Headquarters where he was forthwith detained. The information against him was filed much later. 36. PEOPLE VS. CAMAGUIN 229 SCRA 166 (1994) 37. PEOPLE VS. DULOS 237 SCRA 141 (1994) Nature: It is an appeal from a decision of the Regional Trial Court of Cotabato City Br. 13 which found accused Efren Dulos guilty of the crime of Murder committed with treachery and sentencing him reclusion perpetua. Facts: On the evening of march 15, 1987, professional entertainers, Susan and Alice, were sitting at the lobby of the

New Imperial Hotel in Cotabato city, waiting for prospective clients. A military police assigned at the said hotel as watchman approached them and told them that the accused-appellant had some male guests who wished to be entertained. Both parties agreed to a charge of Php 100.00 each for Susan and Alice as fee for their services. The parties then went to a disco house however Alice left early and decided to wait for Susan at the lobby of the Hotel. Susan , apparently decided to check in with one of the accused-appellants guests at the upper floor of the disco house for an additional fee of Php 500.00. After receiving the money, she changed her mind, thus, spawning a fight with her customer. After MP Gara intervene, Susan came down to the lobby of the Hotel to meet her boyfriend Paul Tamse who was waiting for her. Upon hearing the reneged deal, the irate accused-appellant confronted Susan and Paul. Accusedappellant demanded the return of his money. Susan handed the Php 100.00 to accused-appellant however she denied having received Php 500.00 unless her boyfriend would find out that she agreed to check in with a customer. She tried to grapple with the accused-appellant for the gun but was violently pushed aside. The boyfriend on the other hand pleaded for mercy by kneeling down and raising his hands up. Despite the plea, he was shot by the accused-appellant twice killing him on the spot. The accused-appellant surrendered the gun to the military authorities in Camp Siongco, Maguindanao and was not placed under custody by the military authorities as he was free to roam around as he pleased. Issue: Won a mitigating circumstance of voluntary surrender could be appreciated in this case. Ruling: No, a mitigating circumstance of voluntary surrender could not be appreciated in this case. The SC held that in order that voluntary surrender may be appreciated, it is necessary that it must be spontaneous and made in such manner that it shows the intent of the accused to surrender unconditionally to the authorities, either because he acknowledged his guilt or because he wishes to save the trouble and expenses necessarily incurred in his search and capture. In this case the elements were not present. There was no conscious effort on the part of Dulos to voluntarily surrender to the military authorities. As he himself admitted in his testimony, he was not placed under custody by the military as he was free to roam around as he pleased. Likewise, his claim that he surrendered his gun without surrendering his person to the authorities does not constitute voluntary surrender 38. PEOPLE VS. CRISOSTOMO 160 SCRA 47 (1998) Nature: It is an appeal from the decision of the Court of First Instance of Bulacan which found Eugenio Crisostomo guilty of the crime of murder and sentences him to Reclusion Perpetua, and to indemnify the heirs of the deceased in the sum fo Php 12,000.00 and to pay the costs. Facts: On December 25, 1967 between six and seven oclock in the evening at Sto. Rosario, Hagonoy, Bulacan, while Eugenio Crisostomo was passing near the house of Romeo Geronimo, he met the latter and invited him to have a drink in the place of a friend. Romeo declined the offer. Suddenly Eugenio rushed towards Romeo who was then standing near a store facing the street with his back towards Eugenio and shot him with a .22 caliber revolver at a distance of one meter. The bullet entered into his armpit and came out on the right side of the chest about one inch in the sternum. Romeo fell to the ground mortally wounded while Eugenio ran away. By-standers who were near the place came to the aide of the fallen victim and brought him to the Reyes Hospital in Hagonoy where the doctor pronounced the victim dead upon arrival. Upon arraignment wherein accused entered a plea of not guilty and again during the trial, the accused signified his intention to withdraw hi plea of not guilty to the charge of murder and to substitute it with a plea of guilty to a lesser charge of homicide and prayed that he be allowed to prove the mitigating circumstances. The same plea was made by the accused after the prosecution had rested its case but the fiscal did not agree. Thus the lower court denied the petition.

18

Issue: Won a mitigating circumstance of voluntary plea of guilt be appreciated in this case. Ruling: No, the mitigating circumstance of voluntary plea of guilt is not appreciated in this case. The SC held that he cannot be credited with the mitigating circumstance of a plea of guilty to a lesser offense of the charge of homicide. The requisites of the mitigating circumstance of voluntary plea of guilty are: (1) that the offender spontaneously confessed his guilt; (2) that the confession of guilt was made in open court, that is, before the competent court that is to try the case; and (3) that the confession of guilt was made prior to the presentation of evidence for the prosecution. In the present case the appellant offered to enter a plea of guilty to the lesser offense of homicide only after some evidence of the prosecution had been presented. He reiterated his offer after the prosecution rested his case. This is certainly not mitigating. 39. PEOPLE VS. JOSE ET AL. 37 SCRA 450 (1971) Nature: Appeal from and automatic review of a decision of the Court of first Instance of Rizal. Facts: At about 4:30 am, June 26, 1967, Miss De la Riva, was driving her car accompanied by her maid Helen Calderon. As she was approaching her house at No. 48, 12th Street, New Manila, Quezon City, a Pontiac two-door convertible car with the four accused came abreast of her car and tried to bump it. Pineda stopped the car which he was driving, jumped out of it and rushed towards her. The girl became so frightened at this turn of events that she tooted the horn of her car continuously. Undaunted, Pineda opened the door of Miss De la Riva's car and grabbed the the victims arm and dragged her inside the car. The complainant was made to sit between Jaime Jose and Edgardo Aquino at the back seat; Basilio Pineda, Jr. was at the wheel, while Rogelio Caal was seated beside him. The two men seated on each side of Miss De la Riva started to get busy with her body: Jose put one arm around the complainant and forced his lips upon hers, while Aquino placed his arms on her thighs and lifted her skirt. The car reached a dead-end street. Pineda turned the car around and headed towards Victoria Street. Then the car proceeded to Araneta Avenue, Sta. Mesa Street, Shaw Boulevard, thence to Epifanio de los Santos Avenue. When the car reached Makati, Aquino took a handkerchief from his pocket and, with the help of Jose, blindfolded Miss De la Riva. Not long after, the car came to a stop at the Swanky Hotel in Pasay City. The blindfolded lady was led out of the car to one of the rooms on the second floor of the hotel. Inside the room Miss De la Riva was made to sit on bed. Her blindfold was removed. She saw Pineda and Aquino standing in front of her, and Jose and Caal sitting beside her, all of them smiling meaningfully. Pineda ordered the victim to striptease to which the other men agreed to. After which, the four men took turns in raping her starting with Jose, Aquino, Pineda and Canal. Whenever dela Riva passed out, they would pour water on her face and slapp her to revive her. Mention must be made of the fact that while each of the four appellants was struggling with the complainant, the other three were outside the room, just behind the door, threatening the complainant with acid and telling her to give in because she could not, after all, escape, what with their presence. After the appellants had been through with the sexual carnage, they dropped her in front of the Free Press Building not far from Epifanio de los Santos Avenue near Channel 5 to make it appear, according to them, that the complainant had just come from the studio. Issue: W/N a plea of guilty is mitigating, at the same time it constitutes an admission of all the material facts alleged in the information. Held: the judgment under review is hereby modified as follows: appellants Jaime G. Jose, Basilio Pineda, Jr., and Edgardo P. Aquino are pronounced guilty of the complex crime of forcible abduction with rape, and each and every one of them is likewise convicted of three (3) other crimes of rape. As a consequence

thereof, each of them is hereby sentenced to four (4) death penalties. Ratio: Appellant Pineda claims that insofar as he is concerned there was a mistrial resulting in gross miscarriage of justice. He contends that because the charge against him and his coappellants is a capital offense and the amended complaint cited aggravating circumstances, which, if proved, would raise the penalty to death, it was the duty of the court to insist on his presence during all stages of the trial. The contention is untenable. Because of the aforesaid legal effect of Pineda's plea of guilty, it was not incumbent upon the trial court to receive his evidence, much less to require his presence in court. It would be different had appellant Pineda requested the court to allure him to prove mitigating circumstances, for then it would be the better part of discretion on the part of the trial court to grant his request. The technicalities in plain simple language of the contents of aggravating circumstances and apprised him of the penalty he would get, and we have given said accused time to think. After a while I consulted him - for three times - and his decision was still the same. Three days after the arraignment, the same counsel stated in court that he had always been averse to Pineda's idea of pleading guilty, but that he acceded to his client's wish only after the fiscal had stated that he would recommend to the court the imposition of life imprisonment on his client. 40. PEOPLE VS. VILLA G.R. 129899 (APRIL 2000) Nature: Appeal from a decision of the Regional Trial Court of Olongapo City Facts: In the early morning of 22 June 1991 Dionito Fernandez was cutting grass in his yard in New Cabalan, Olongapo City. Accused Rodolfo Villa, Jr., a member of the CAFGU and neighbor of Dionito, suddenly came out of his house with his M1 Garand rifle and shot Dionito from behind killing him instantly. Ronald Fernandez and Sheila Fernandez, children of Dionito, rushed to their father's rescue after hearing the gunshot but the accused also fired at them fatally hitting Ronald who was embracing his father, and mortally wounding Sheila on the thigh and stomach. Samuel Eclevia, another neighbor of the Fernandezes, attempted to wrestle the rifle from the accused but Samuel too was gunned down. After his rampage, Rodolfo Villa Jr. surrendered to a certain Captain Dolino of S2 OMDC (Olongapo Metropolitan District Command Issue: W/N taking advantage of his public position as a CAFGU member should be considered against accusedappellant. Held: The assailed Decision of the trial court convicting accused-appellant RODOLFO VILLA, JR. Y DELGADO of four (4) separate counts of Murder is AFFIRMED, subject to the MODIFICATION of the penalties imposed. The SC did not agree that the aggravating circumstance of "taking advantage of his public position" as a CAFGU member should be considered against accused-appellant. The mere fact that he was a member of the CAFGU and was issued an M-1 Garand rifle is not sufficient to establish that he misused his public position in the commission of the crimes. 41. PEOPLE VS. GAPASIN 231 SCRA 728 (1994) Nature: This is an appeal from the decision of the Regional Trial Court, Branch XVI, Isabela in Criminal Case No. IV-781, finding appellant guilty beyond reasonable doubt of murder qualified by treachery, with the attendance of the mitigating circumstance of voluntary surrender, and the aggravating circumstances of taking advantage of public position and evident premeditation sentencing him to suffer reclusion perpetua.

FACTS: According to prosecution witness Alberto Carrido, he and Rodrigo Ballad left the house of Enteng Teppang at about

19

2:00 P.M. of October 6, 1979 after attending the "pamisa" for the deceased father of Teppang. Jerry Calpito followed them. While they were walking along the barangay road, Calpito was shot by appellant with an armalite rifle. When Calpito fell on the ground, appellant fired more shots at him. Thereafter, accused Amor Saludares planted a .22 caliber revolver on the left hand of Calpito. Upon hearing the shots, Faustina Calpito ran to succor her fallen husband. Accused Nicanor Saludares pointed his gun at Faustina while accused Soriano fired his gun upwards. Saludares warned that he would kill any relative of Jerry Calpito who would come near him. Faustina and the other relatives of the victim scampered away as the Saludares' group chased them. Appellant invoked self-defense. He testified that he was issued a mission order on September 23, 1979 to investigate a report regarding the presence of unidentified armed men in Barrio San Jose, Roxas, Isabela. The following day, he was instructed by Sgt. Dominador Ignacio to get in touch with Nicanor Saludares who may be able to give him information on the identities of the persons with unlicensed firearms in the place. When appellant met Nicanor Saludares on September 29, 1979, he was informed that Jerry Calpito had an unlicensed firearm. The body of Calpito was autopsied by Dr. Bernardo Layugan, who found that the victim sustained four bullet wounds: (1) on the right lateral side of the arm fracturing the humerus; (2) on the right lateral side of the thorax between the 7th and 8th ribs with exit wound at the sternum; (3) on the left side of the thorax, anterior, between the 5th and 6th ribs; and (4) on the right fronto-parietal portion of the head "severing the skull and brain tissues" (Exh. "F"). Dr. Layugan opined that the victim was in a standing position when he was shot by someone positioned at his right.

attempted to draw his gun. PC soldier Virgilio Fidel immediately grabbed appellant's gun from appellant's waist and gave it to Lt. Masana. After that, Lt. Masana told the appellant to go inside the restaurant. PC soldier Virgilio Fidel followed. Lt. Masana and the appellant occupied a separate table about one and one-half (1 1/2) meters from the table of Lt. Masana's three companionsFidel, Ligsa and Mojica (p. 10, t.s.n., Nov. 22, 1971). After the two were already seated, Lt. Masana placed appellant's gun on the table. After that Lt. Masana pulled out a piece of coupon bond paper from his pocket and wrote thereon the receipt for the gun, and after signing it, he asked appellant to countersign the same, but appellant refused to do so. Instead, he asked Lt. Masana to return the gun to him. Lt. Masana rejected appellant's plea, telling the latter that they would talk the matter over in the municipal building of Indang, Cavite. When Lt. Masana was about to stand up, appellant suddenly pulled out a double-bladed dagger and with it he stabbed Lt. Masana several times, on the chest and stomach causing his death several hours thereafter. Issue: W/N the attack on the victim, who was known to the appellant as a peace officer, could be considered only as aggravating, being "in contempt of/or with insult to the public authorities", or as an "insult or in disregard of the respect due the offended party on account of his rank. Held: GUILTY BEYOND REASONABLE DOUBT OF HOMICIDE AGGRAVATED BY CONTEMPT FOR OR INSULT TO A PUBLIC AUTHORITY, THE JUDGMENT APPEALED FROM IS AFFIRMED, PENALTY MODIFIED. While the evidence definitely demonstrated that appellant knew because the victim, who was in civilian clothing, told him that he was an agent of a person in authority, he cannot be convicted of the complex crime of homicide with assault upon an agent of a person in authority, for the simple reason that the information does not allege the fact that the accused then knew that, before or at the time of the assault, the victim was an agent of a person in authority. The information simply alleges that appellant did, "attack and stab PC Lt. Guillermo Masana while the latter was in the performance of his official duties, . . . " Such an allegation .cannot be an adequate substitute for the essential averment to justify a conviction of the complex crime, which necessarily requires the imposition of the maximum period of the penalty prescribed for the graver offense. Like a qualifying circumstance, such knowledge must be expressly and specifically averred in the information; otherwise, in the absence of such allegation, the required knowledge, like a qualifying circumstance, although proven, would only be appreciated as a generic aggravating circumstance. 43. PEOPLE VS. DANIEL 86 SCRA 511 (1978) Nature: APPEAL from the judgment of the Court of First Instance of Baguio City. Facts: On September 20, 1965, at about three o'clock in the afternoon, Margarita Paleng had just arrived in the City from Tublay in a Dangwa bus. Because it was then raining and the bus was parked several meters away from the bus station, she waited inside the bus . After about three minutes of waiting, the accused came and started molesting her by inquiring her name and getting hold of her bagShe called the attention of the bus driver and the conductor about the actuation of the accused, but it seemed that the former were also afraid of him "Despite the rain, she left the bus and went to ride in a jeep parked some 100 meters away. The accused closely followed her (P. 4, id.). When the jeep started to go, the accused also rode and sat beside her "When the jeep reached Guisad, she alighted on the road but she still had to negotiate a distance of ten meters The accused also alighted and again he tried to carry her bag Although he was not allowed to carry her bag, he was adamant in following her "Reaching her boarding house, she opened the door and was about to close it when the accused dashed in and closed the door behind him When she entered her room, the accused went in, pulled a dagger eight inches long and threatened her and then raped her.

ISSUE: Whether or not the appellants are guilty beyond reasonable doubt of murder qualified by treachery, with the attendance of the mitigating circumstance of voluntary surrender, and the aggravating circumstances of taking advantage of public position and evident premeditation? HELD: Appellants are guilty beyond reasonable doubt of murder qualified by treachery, with the attendance of the mitigating circumstance of voluntary surrender, and the aggravating circumstances of taking advantage of public position and evident premeditation. Appellant's claim of self-defense is belied by the finding of the trial court that the victim was shot by someone who was standing on his right side. The fact that the prosecution witnesses are relatives of the victim does not necessarily indicate that they were biased as to impair their credibility. 42. PEOPLE VS. RODIL 109 SCRA 308 (1981) Nature: Automatic Review of the judgement of the Circuit Criminal Court of Pasig Rizal. Facts: At about 1:00 o'clock in the afternoon of April 24, 1971, the deceased, PC Lt. Guillermo Masana, together with PC soldier Virgilio Fidel, Philippine Coast Guard serviceman Ricardo Ligsa, and Patrolman Felix Mojica of Indang, Cavite, was having lunch inside a restaurant in front of the Indang market (pp. 2, 3, t.s.n., Oct. 30, 1971; pp. 10, 19, t.s.n., Nov. 22, 1971; p. 21, t.s.n., Jan. 20, 1972). While they were eating, they saw, through the glass panel of the restaurant, appellant outside the restaurant blowing his whistle. Their attention having been drawn to what appellant was doing, Lt. Masana, then in civilian clothing, accompanied by PC soldier Virgilio Fidel, went out of the restaurant, approached appellant and asked the latter, after identifying himself as a PC officer, whether the gun that was tucked in his waist had a license. Instead of answering the question of Lt. Masana, appellant moved one step backward and

20

Issue: W/N renting a bedspace in a boarding house constitute for all and purposes a dwelling. Held: The judgment of conviction of Amado Daniel for the crime of rape as charged is affirmed. Although Margarita was merely renting a bedspace in a boarding house, her room constituted for all intents and purposes a "dwelling" as the term is used in Article 14(3), Revised Penal Code. It is not necessary, under the law, that the victim owns the place where he lives or dwells. Be he a lessee a boarder or a bed-spacer, the place is his home the sanctity of which the law seeks to protect and uphold. 44. PEOPLE VS. MANDOLADO 123 SCRA 128 (1983) Nature: Appeal from the decision of the Court of First Instance of Cotabato convicting Martin Mandolado and Julian Ortillano of murder qualified with aggravating circumstances of treachery, evident premeditation and abuse of confidence/obvious ungratefulness. Facts: Sometime in October 1977, four draftees of third infantry Batallion were passengers of a bus bound for Midsayap Cotabato City. Arriving at the terminal, they decided to drink ESQ rum where Martin after going inside the market and fired his caliber machine gun. They then boarded a car and forced the driver to bring them to Midsayap Crossing and in the way, Herminigildo got his knife and tried to attack the driver. After they alighted from the jeep, the accused started firing his gun and hit the occupants of the jeep while Julian fired his armalite downwards in order to show that they were fighting with some MILF rebels. Issue: W/N abuse of confidence and obvious ungratefulness can be inferred from the mere fact that an army draftee who was allowed on board a vehicle later on fired his gun at its occupants. Held. Decision is modified as to penalty but affirmed in all other respects. There is merit in appellants' contention that there could be no abuse of confidence as the evidence on record showed the lack of confidence by the victims to the appellants, that this confidence was abused, and that the abuse of the confidence facilitated the commission of the crimes. In order that abuse of confidence be deemed as aggravating, it is necessary that "there exists a relation of trust and confidence between the accused and one against whom the crime was committed and the accused made use of such a relationship to commit the crime." (People vs. Comendador, 100 SCRA 155, 172). It is also essential that the confidence between the parties must be immediate and personal such as would give that accused some advantage or make it easier for him to commit the crime; that such confidence was a means of facilitating the commission of the crime, the culprit taking advantage of the offended party's belief that the former would not abuse said confidence (People vs. Hanasan, 29 SCRA 534). In the instant case, there is absolutely no showing of any personal or immediate relationship upon which confidence might rest between the victims and the assailants who had just met each other then. Consequently, no confidence and abuse thereof could have facilitated the crimes. Similarly, there could have been no obvious ungratefulness in the commission of the crime for the simple reason that the requisite trust of the victims upon the accused prior to the criminal act and the breach thereof as contemplated under Article 14, par. 4 of the Revised Penal Code are manifestly lacking or non-existent. In all likelihood, the accused Army men in their uniforms and holding their high-powered firearms cowed the victims into boarding their jeep for a ride at machine gun point which certainly is no source of gratefulness or appreciation. 45. PEOPLE VS. MARRA 236 SCRA 565 (1994) Nature: APPEAL from a decision of the Regional Trial Court of Dagupan City.

Facts: Jimmy Din, recounted that at around 2:00 A.M. on March 7, 1992, he and his friend, Nelson Tandoc, were conversing with each other in front of Lucky Hotel located at M. H. del Pilar Street, Dagupan City, which was owned by the witness' father and of which he was the administrator. He noticed a man pass by on the opposite side of the street. The man made a dirty sign with his finger and Din informed Tandoc thereof. The man repeated his offensive act and called them by waving his hands. Infuriated, they followed the man until the latter stopped in front of the Dunkin' Donuts store at the corner or Arellano and Fernandez streets. They demanded an explanation from the man but they were not given any. At that instant, two men arrived and one of them inquired what was going on. Tandoc informed him that they were just demanding an explanation from the man. Din was surprised when Tandoc unexpectedly slapped one of the two men. A brawl ensued, with Tandoc clashing with the two men while Din exchanged blows with the man who made the dirty finger sign. After the fisticuffs, their three opponents ran away in a westward direction. Tandoc and Din then decided to walk back to the hotel. When they were about to enter the place, they noticed that the men with whom they just had a fight were running towards them. Sensing danger, they ran inside the annex building of the hotel and immediately secured the lock of the sliding outer door. They entered a room and waited until they felt that the situation had normalized. After ten to fifteen minutes, thinking that the men were no longer in the vicinity, they left the room. Having decided to go home, Tandoc opened the sliding door, All of a sudden, Din saw appellant, who at that time was wearing a security guard's uniform, shoot Tandoc with a revolver, There was a fluorescent bulb installed at the front of the hotel which enabled Din to identify the assailant. Tandoc was shot in the middle of the chest and he fell down. Then, Din saw four to five men scamper away from the scene. Aware of his injury, Tandoc told Din, "Tol, I was shot." The latter tried to chase appellant and his companions but he failed to catch up with them. Din and his wife then brought Tandoc to the Villaflor Hospital. The victim was taken to the emergency room but he expired an hour later. Issue: W/N nocturnity was specially sought by appellant or taken advantage of by him to facilitate the commission of the crime or to ensure his immunity from capture. Held: The judgment of the court is AFFIRMED. However, the crime committed by appellant was murder qualified by treachery, the SC rejected the finding that the same was aggravated by nighttime. No evidence was presented by the prosecution to show that nocturnity was specially sought by appellant or taken advantage of by him to facilitate the commission of the crime or to ensure his immunity from capture. At any rate, whether or not such aggravating circumstance should be appreciated, the penalty to he imposed on appellant would not be affected considering the proscription against the imposition of the death penalty at the time when the offense in the instant case was committed. 46. PEOPLE VS. DESALISA (UNINHABITED PLACE) 299 SCRA 35 (1994) Nature: Appeal on the conviction of Emmanuel Desalisa of the crime of parricide. Facts: Emmanuel lived with his 18 year old wife, Norma and 2 year old daughter on a small nipa house on a hill at Pinaductan, San Juan, Bacon, Sorsogon. 2 other houses in the neighborhood, 150 meters away, cannot be seen because of the fruit tree. Paulina Dioneda, Normas mom at about 10am of Oct, 9, 1983 (day of crime), was informed by emmanuels mother that the couple had an altercation. He slapped and boxed her on the stomach. At 5pm same day, Norma complained that Emmanuel was a jealous man, even before he manhandled by Emmanuel. Vicente Dioneda, father of Norma, stated that around 6 or 7pm, Emmanuel left his child with them. The following morning, around 6 or 7am, he went to house of Emmanuel and Norma. He saw plates scattered, rope of hammock missing. He thought of feeding pig so he climbed the coconut tree. On the third step,

21

he saw the back of the body of Norma. He went down the tree, called her, he touched her, and she swayed! He realized he was hanging, her feet approximately 4 inches above the ground. He informed his wife and they went to Carlito Dichoso and fetched the authorities. He saw Emmanuel at the municipal building of Bacon on Oct. 10, he asked him why he killed her, he did not answer but just stooped down. Carlito Dichoso, neighbor, testified that around 6 or 7pm of Oct. 9, Emmanuel went to his house, it was raining. Emmanuel borrowed a flashlight because he was looking for his wife. After 2 hrs, Emmanuel returned to Carlitos house, he sat on a bench. Carlito asked him if he found his wife, he did not answer. possessed with devils. Around 5am, Emmanuel told Carlito that if there is something that happened, Manoy Carlito, what would I do? Accused was convicted based on circumstantial evidence. Issue: W o N the place can be considered uninhabited. Held: The decision appealed from is hereby modired. Aclisedappellant is found guilty beyond reasonable doubt of the complex crime of parricide with unintentional abortion and sentenced to suffer the penalty of reclusion perpetua The aggravating circumstance of uninhabited place is present. The uninhabitedness of a place is determined not by the distance of the nearest house to the scene of the crime but whether or not in the place of the commission, there was reasonable possibility of the victim receiving some help. Considering that the killing was done during nighttime and many fruit trees and shrubs obstruct the view of neighbors and passersby, there was no reasonable possibility for the victim to receive any assistance. Note that the trial court convicted accused-appellant of the crime of parricide only. This is an error. The evidence on record has shown beyond reasonable doubt that accused-appellant has committed the complex crime of parricide with unintentional abortion. The abortion was caused by the same violence that caused the death of the victim. It is unintentional because accused-appellant must have merely intended to kill the victim but not necessarily to cause an abortion. 47. PEOPLE VS. LUG-AW (TREACHERY) Nature: Appeal from a decision of the RTC of Cabarroquis, Quirino convicting Julio Lug-aw and Rogelio Bannay of murder sentencing both to reclusion perpetua or life imprisonment. Facts: On Dec. 12, 1985, Carlos Pal-oy was putting up a fence allegedly upon the instruction of the public forester to straighten out the boundary line. His daughters Sonia and Carina were with him. Pal-oy was proceeding towards the house when Sonia heard a gun report. Immediately, she went uphill and just as a second gun report resounded, she saw Rogelio Bannay and Julio Lug-aw from a distance of around four meters. She saw too that as her father was about to draw his bolo, Lug-aw shot him. Trial court appreciated both treachery and evident premeditation against the accused. Issue: W o N the trial court correctly ruled that the crime committed was murder. Held: Appellant Lug-aw is found guilty beyond reasonable doubt of the crime of homicide and Bannay is acquitted. Sonias testimony concludes that the crime committed was homicide instead of murder. The qualifying circumstances of treachery and evident premeditation had not been proven beyond reasonable doubt. The trial court drew the conclusion of the presence of treachery because the attack was sudden as Pal-loy was simply going about his task of fencing. The SC however, finds that no one witnessed the initial attack. Absent any particulars as to the manner in which the aggression commenced or how the act which resulted in the death of the victim unfolded, treachery cannot be appreciated to qualify the killing to murder. Evident premeditation cannot be appreciated because its requisites are not present.

As to Bannay, his presences at the scene of the crime, unless conspiracy is proven, do not by itself, indicate criminal culpability. Conspiracy must be proved beyond reasonable doubt. 48. PEOPLE VS. CAMILET (EVIDENT PREMEDITATION) Nature: Appeal from the judgment of the Regional Trial Court of Iloilo convicting Regino Camilet of murder and sentenced to life imprisonment. Facts: On July 2, 1982 at around 7pm, in Coyogan Sur, Leon, Iloilo City, a prayer meeting was held at the residence of Brgy. Capt. Perfecto Camancho Sr. At around 9pm, Dione Camancho, a dumb nephew of Perfecto arrived crying and making signs that he was spanked at the buttocks by someone at a certain place. Accompanied by Dione, Perfecto Camancho, Jr., Rosita Camayo and Joven Cagayoa and Perfecto, Sr. went to the place indicated by Dione. When they had walked a distance of around 150 meters, Camilet suddenly stepped from a grove of banana plants and without word or warning, stabbed Perfecto, Sr. with a onefoot long, sharp-bladed knife. Perfecto, Sr. died as a result. Prior to the incident, there was a misunderstanding between the Camanchos and the accused-appellant regarding a portion of land belonging to Camilets mother-in-law, where Camilet was working. The settlement reached apparently did not satisfy Camilet. The information alleged treachery, evident premeditation, nighttime and disregard of rank and age as aggravating circumstances, however, the trial courts decision did not indicate what circumstances qualified the killing to murder but considered disregard of rank an aggravating circumstance. Issue: W o N evident premeditation can be considered to qualify the killing into murder. Held: Judgment of trial court modified. Regino Camilet is found guilty of homicide, instead of murder. In the absence of a qualifying circumstance, the fatal stabbing of Perfecto, Sr. is a homicide, not a murder. Treachery cannot be appreciated in this case. Mere suddenness of an attack is not sufficient to constitute treachery where it does not appear that the aggressor adopted such mode of attack to facilitate the perpetration of the killing without risk to himself. Likewise, evident premeditation was not established by the prosecution. Although the facts tend to show that Camilet may have harbored ill-feelings towards the Camanchos regarding the parcel of land, there is no evidence of: (1) the time when he determined to commit the crime, (2) an act manifestly indicating that he has clung to his determination, and (3) sufficient lapse of time between determination and execution to allow him to reflect upon the consequences of his act and to allow his conscience to overcome the resolution of his will. Nighttime cannot be considered aggravating since there was no proof that it was especially sought by the accused to perpetuate the crime. Lastly, disregard of rank cannot likewise be considered because, there is no clear evidence that the accused committed the crime in disregard of the respect due to the victim. 49. PEOPLE VS. ILAOA (CRUELTY) Note: Cruelty exists when the culprit enjoys and delights in making his victim suffer slowly but gradually causing him unnecessary pain in the consummation of the act. Requisites: injury caused be deliberately increased by causing another wrong; the other wrong must be unnecessary for the execution of the purposes of the offender. Nature: Appeal from judgment of RTC of Angeles City finding RUBEN and ROGELIO ILAOA guilty of Murder with the aggravating circumstances of Evident premeditation, abuse of superior strength, and cruelty, sentencing them to life imprisonment.

22

Facts: On Nov. 5, 1987, Nestor de Loyolas body was found in a grassy portion of Tinio St., Angeles City. He was decapitated, had 43 stab wounds on the chest and slight burns all over his body. The night before, at around 11pm, de Loyola was seen drinking with Ruben Ilaoa and 3 others outside Ilaoas apartment. A few minutes later an argument arose between the two, after which Ruben and his drinking companions mauled, kicked and dragged the victim towards the accused apartment. He was heard crying aray! and pare, bakit niyo ako ginaganito, hirap na hirap na ako. At around 2am, Ruben borrowed Alex V illamils tricycle on the pretext that a neighbor needed to be brought to the hospital as she was about to give birth. However, he was seen driving alone with a sack placed in the sidecar, which looked like as if it contained a human body. When the tricycle was returned, there wee blood stains on the floor. Susan Ocampo, Rubens live-in partner, was also seen sweeping blood at the entrance of their apartment and blood was found on his shirt when police investigated him and the hair near his right forehead was partly burned. Held: Rogelio was acquitted, while Ruben was convicted for homicide only. Rogelio was acquitted because the circumstances relied upon to establish his guilt, particularly the dragging of the victims body into the house of his brother, is totally inadequate for a conviction. Rubens liability was modified to homicide only because the qualifying circumstances were not sufficiently proved. There was no evidence whatsoever that appellant was physically superior to the deceased and tat he took advantage of such superior physical strength to overcome the latter. (Mere numerical advantage is not enough to be qualified as an abuse of superior strength). Number of wounds alone is not the criterion for the appreciation of cruelty, neither can it be inferred from the mere fact that the victims dead body was dismembered. There was no showing that appellant for his pleasure and satisfaction caused the victim to suffer slowly and painfully and inflicted on him unnecessary physical and moral pain. Evident premeditation cannot be appreciated also since there is no proof that such killing was the result of meditation, calculation or resolution on the part of Ruben. 50.PEOPLE VS. MARQUEZ (CRAFT, FRAUD, DISGUISE) 117 SCRA 165 (1982) Nature: An appeal from the judgment of the Court of First Instance of Quezon finding the accused guilty of the crime of robbery with rape as defined under Article 294, paragraph 2 of the Revised Penal Code and sentenced them to life imprisonment. *note: Renato Marquez died during trial. Facts: On November 16, 1966, between seven and seven-thirty in the evening, Francisca Marquez-Tan was in their house in barrio Dahican Catanauan, Quezon together with her seven children and maid Rufina Martinez. She heard somebody call out in front of their window who identified themselves as PC soldiers looking for contraband. She replied that they did not have any contraband and that her husband, Angel Tan, was in the poblacion at that time. The men ordered her to open up otherwise they will shoot up their house. She opened up her window and Renato jumped inside. She was able to recognize Renato Marquez as the light was bright, and as he was her distant relative. Renato held her by the nape and pushed her towards the door and at gunpoint ordered her to open the same. When she opened the door, accused Samuel Jacobo and Francisco Forneste, both armed with guns, entered and ordered her to put out their contraband and when she told them that they did not have any, the intruders demanded for money. She pointed at the table, which Renato open and took P300 there from. Jacobo also pried open their aparador where he got P200. Jacobo also dispossessed her of her ring worth P15.00 and a pair of earrings worth also that much. At that instant, the other accused Forneste was upstairs guarding her children and helper.

Samuel Jacobo asked her why they have only a small amount of money when they are copra-buyers and she replied that they were just starting on their business. Whereupon, Jacobo demanded: "kuarta o buhay" so that she put out her pillow which Jacobo grabbed and ripped open and took there from P820.00. Afterwards, Samuel Jacobo raped her at gunpoint while Renato ransacked their store and took merchandise there from. After five minutes, Jacobo took her upstairs and tied both her arms and made her he face down on the floor together with her children. Subsequently, her daughter Leticia and helper Rufina Martinez were taken downstairs by Francisco Forneste. Shortly after the men left, the victims were able to free themselves and Francisca found out that the two women were raped also. Leticia Tan corroborated the foregoing testimony of her mother. The rapes, according to her were committed in the following manner: when her mother, Francisco Marquez was taken upstairs, Renato Marquez brought her downstairs, to their store. Inside the store Renato Marquez to her " to give something and if I refused I would be killed. Simultaneously Renato Marquez "poked a gun and also a balisong" at her causing her to be afraid she called for her mother but then she was told not to shout because "I am going to be killed." Thereafter, she was forcibly made to lie down and Renato Marquez committed the act on her. Leticia related that during the time that she was with Renato Marquez, Rufina Martinez was with Francisco Forneste and that immediate after the departure of Renato Marquez, Francisco Forneste and Samuel Jacobo, Rufina Martinez told her that she was also raped by Francisco Forneste. Held: The evidence adduced is not sufficient to show any conspiracy among the accused in the commission of the crime of rape against the persons of Francisca Marquez, Leticia Tan and Rufina Martinez. Therefore, the lower court was correct in concluding that the crime committed by the accused appellants was robbery with rape not robbery with multiple rapes as alleged in the information. 51. PEOPLE VS. EMPACIS 222 SCRA 59 (1993) Nature: An appeal from the decision of the Regional Trial Court of Cebu City, Branch 14 finding the accused guilty of robbery with homicide as defined and penalized under Article 294 (1) of the Revised Penal Code, and considered the attendance of the four generic aggravating circumstance of dwelling, nighttime, craft or fraud and superior strength, not offset by any mitigating or extenuating circumstance, sentencing him to death. Facts: At about 9 o'clock on the night of September 16, 1986, as Fidel Saromines and his wife, Camila, were about to close to their small store, located in their house at Kanguha, Dumanjug, Cebu, two men came and asked to buy some sardines and rice. They were Romualdo (or Maldo) Langomez and Crisologo Empacis. Camila served them and they proceeded to make a meal of the rice and sardines. After they finished eating, Romualdo told Fidel to sell him cigarettes. As Fidel was handing over the cigarettes, Romualdo announced a "hold-up" and commanded Fidel to give up his money. As it happened, Fidel then had P12, 000.00 in his house, wrapped in cellophane. This he started to give to Romualdo but as the latter was taking hold of the packet, Fidel suddenly decided to fight to keep his money. A struggle followed in the course of which Romualdo stabbed Fidel about three times. Crisologo joined in and with his own knife also stabbed Fidel. At this time, gunshots were heard outside of the house; and a neighbor of the Saromineses, Balbino Bulak, recognized one of those doing the shooting as certain Carlito Antiga. 8 A voice was heard from below saying, "Stab him!" 9 to which Langomez replied, "I already stabbed (him)." From his little sister's room, Fidel's thirteen-year old son, Peter, saw his father fighting for his life with Romualdo and Crisologo Empacis. Heeding his father's cry for help, Peter took hold of a "pinuti" (a long bolo), and struck inflicting wounds on the latters shoulder and neck. The two accused jumped out of the house and fled. Peter then turned to his wounded father, but found him already dead from his injuries.

23

Crisologo Empacis repaired to the clinic of Dr. Eustaquio Deiparine at the poblacion of Sibonga, Cebu, for treatment of the wounds inflicted on him by Peter, arriving there between 10 and 11 o'clock that same night. Dr. Deiparine asked Crisologo how he had come by these wounds. Crisologo said that at around 6 to 7 o'clock that evening, near the Papan Market, he was assaulted without warning by a young man, who injured him with a bolo. The next day police officers went to Dr. Deiparines clinic and asked for information regarding a man who might have been treated for hacking wounds. They were directed to the public market where Empacis was and he was arrested. Issue: Whether or not craft, fraud or disguise was used for the commission of the crime Held: Judgment affirmed, modification in costs. The aggravating circumstance of craft or fraud was properly appreciated against Empacis. He and Romualdo pretended to be bona fide customers of the victim's store and on his pretext gained entry into the latter's store and later, into another part of his dwelling. The SC has held stratagems and ruses of this sort to constitute the aggravating circumstance of fraud or craft, e.g.: where the accused a) pretended to be constaabulary soldiers and by that ploy gained entry into the residence of their prey whom they thereafter robbed and killed. 52. PEOPLE VS. RUELLAN (SUPERIOR STRENGTH) 231 SCRA 650 (1994) Nature: an appeal from the decision of the Regional Trial Court of Davao City, Branch 13, convicting the accused, Fordito Ruelan, of the crime of MURDER and imposing on him the penalty of "life imprisonment." Facts: On August 4, 1988, Spouses Ricardo and Rosa Jardiel hired appellant as a store helper at their store located in Bankerohan Public Market, Davao City. Appellant helped Jardiel spouses in selling and delivering rice to various customers. He stayed in the couple's residence but he had separate quarters for sleeping. On August 18, 1988, at around 4:00 a.m., Ricardo Jardiel was roused by the closing of the bedroom door and he saw his wife Rosa Jardiel leaving his room. Ricardo Jardiel stood up and followed his wife who went towards the gate of the house. Rosa Jardiel was joined by appellant since they would open the store in Bankerohan Public Market. Rosa Jardiel talked to appellant and ordered him to bring an axe which would be used in repairing some fixtures in the store. Appellant followed her order and took an axe and sack. When they were about to leave the premises, Rosa Jardiel's house dog got loose and went out towards the street. Rosa Jardiel got angry and scolded appellant while she walked ahead of him along Tulip Drive going to McArthur Highway. Appellant pleaded Rosa Jardiel to stop berating him but Rosa Jardiel did not heed to his request. Appellant got fed up and with the use of his axe, he struck Rosa Jardiel behind her right ear causing her to fall face down. Thereafter, appellant dragged Rosa Jardiel to a grassy portion at the side of the street and then immediately left the place. Issue: W o N there was abuse of superior strength. Held: Judgment modified. Accused is guilty only of homicide and ordered to suffer a penalty of six (6) years and one (1) day of prison mayor as minimum to fourteen (14) years, eight (8) months and one (1) day of reclusion temporal as maximum. To properly appreciate the aggravating circumstance of abuse of superior strength, the prosecution must prove that the assailant used purposely excessive force out of proportion to the means of defense available to the person attacked.19 In the instant case, the appellant clearly took advantage of his superior strength as the victim while the appelllant was then only 20 years old, of good statute and build and was armed with an axe with which to kill the victim. However, the aggravating circumstances of abuse superior strength cannot qualify the killing of the victim and raised it to the category of murder because the same was not alleged in the information. The rule in case like this is clear. A qualifying circumstance like abuse of

superior strength must be pleaded in considered as a generic aggravating circumstance in the imposition of the correct penalty. 53. PEOPLE VS. PADILLA 233 SCRA 46 (1994) Nature: An appeal from a decision of the then Court of First Instance of Catbalogan, Samar finding the accused guilty of murder qualified by treachery with the generic aggravating circumstance of taking advantage of his public position, but appreciating at the same time the mitigating circumstance of sufficient provocation and imposing the penalty of reclusion perpetua. Facts: On 4 May 1981, Pat. Omega was on duty at Pier 1 in Catbalogan, Samar, from eleven o'clock in the evening to seven o'clock the following morning. At past midnight, 5 May 1981, Pfc. Edino Ontuca, Officer-in-Charge of the Talalora Police Sub-Station, approached him for assistance claiming he was maltreated by strangers. Pat. Omega responded and both proceeded to where complainant was reportedly assaulted, passing by Malayan Hotel at the pier area to get Ontuca's service revolver. When they reached the corner of Rizal Avenue and Del Rosario Street, the two (2) policemen saw three (3) men in the company of a woman. Pat. Omega approached them, identified himself as a police officer, and then began to investigate the reported "castigo" or manhandling of complainant. A certain C1C Belino spoke for the group. He introduced himself first and then his companions, Maj. de la Cruz and Sgt. Padilla. Pat. Omega tapped Pfc. Ontuca's shoulder and admonished the latter saying, "Brod, let us stop; just keep silent; just go home and sleep. Pfc. Ontuca took the advice and returned to the pier with Pat. Omega. When they reached Malayan Hotel, Pfc. Ontuca stayed behind at the entrance while Pat. Omega proceeded to the Lion's Waiting Shed some fifty (50) meters away. Suddenly, there was a commotion in front of the hotel. Pfc. Ontuca was being ganged up by three (3) men. Pat. Omega then rushed towards the hotel where he saw his companion already down on his right knee with both arms stretched behind him, his left arm held by Maj. de la Cruz and his right by Sgt. Padilla, while C1C Belino held him by the waist and took his service revolver away. Pat. Omega tried to intervene but C1C Belino and Maj. de la Cruz poked their pistols at him. C1C Belino disarmed Pat. Omega and handed over his service pistol to Maj. de la Cruz who then grabbed Omega by the collar saying, "Let us go to the hospital because you are drunk." Pat. Omega denied that he was drunk. Maj. de la Cruz summoned a certain Sgt. Bongosia to accompany them to the hospital purportedly in order to have the two policemen undergo "liquor test". Sgt. Bongosia obliged and then cocked his armalite. All six (6), namely, Maj. de la Cruz, Pat. Omega, Pfc. Ontuca, C1C Belino, Sgt. Bongosia and the accused then proceeded east along Curry Avenue. Upon reaching the intersection of Rizal Avenue and San Bartolome Street, Pfc. Ontuca turned right, eastward. The accused followed by Maj. de la Cruz and Pat. Omega, pursued Pfc. Ontuca, while Sgt. Bongosia did not take the same route; he ran straight along Curry Avenue and then turned left, north, along San Francisco Street. C1C Belino dashed to the opposite direction, turning left towards Del Rosario Street. When Pfc. Ontuca reached a fruit stand beside Cinex Theater, he grabbed a girl named Lilia, an employee at the nearby Bahay Kawayan Disco situated along Callejon, a narrow street connecting Rizal Avenue. Lilia struggled and screamed for help as Pfc. Ontuca held her tightly by the waist, using her as a human shield against the accused who was pointing his pistol at him. Finally, Pfc. Ontuca and Lilia fell to the ground giving the latter a chance to escape. Left without any protection, Pfc. Ontuca squatted on the ground and reached for a piece of plywood which he held upward to cover his head. Maj. de la Cruz and Pat. Omega was just across the street standing in front of the Bonifacio Nardo Store some fifteen (15) meters away. From where they stood, they could clearly see the side view of the accused and the victim facing each other.

24

Pfc. Ontuca begged for his life. "I am not going to fight with you," he said. But the accused, showing no mercy, squeezed the trigger of his .45 cal. automatic pistol pumping a single bullet into the head of his victim who was just some three to four meters from him. The time was exactly two o'clock in the morning. After shooting Pfc. Ontuca, the accused backtracked and then returned to the fallen policeman and tauntingly kicked him saying, "Are you still alive?" The accused then went to Maj. de la Cruz and talked to him. Issue: Whether or not there was an abuse of superior strength Held: Judgment affirmed. The killing was qualified by the aggravating circumstance of abuse of superior strength which was alleged in the information and proved during the trial. Abuse of superior strength is present not only when the offenders enjoy numerical superiority, or there is a notorious inequality of forces between the victim and the aggressor, but also when the offender uses a powerful weapon which is out of proportion to the defense available to the offended party. The accused was armed with a powerful pistol, which he purposely used, gaining him an advantage over his victim who only had a piece of plywood to cover himself after he was disarmed. The accused did not abuse his public position in committing the crime. For this circumstance to be appreciated as aggravating, the public official must use his influence, prestige and ascendancy, which his office gives him in realizing his purpose. It could not be said that the accused purposely used or took advantage of his position or rank in killing the victim because he could have committed the crime just the same by using another weapon not necessarily his service firearm. 54. PEOPLE VS. VERCHEZ 233 SCRA 174 (1994) Nature: An appeal from the Decision of the Regional Trial Court, Branch 19, Bacoor, Cavite convicting the accused of murder qualified by treachery. Facts: Verchez invited Balane on August 15, 1985 to visit his brother, who was then living in Queen's Row Subdivision, Bacoor, Cavite. On their way, the two chanced upon Aldave, a "compadre" of Verchez' brother, who joined them. They arrived at their destination at about 2:30 P.M. but found that Verchez' brother was not at home. However, they saw Alfredo Mamuntag, the caretaker of the house, Alfredo's son, Hector, and Gilbert Ang, who were then visiting with Alfredo. Verchez decided to wait and drink liquor at the yard. At about 3:00 P.M., Balane drove away to buy cigarettes and "pulutan." He had not driven far when a car blocked his way, with the occupants pointing their firearms at him. Then another car arrived. One of the passengers from the second car approached Balane and frisked him. Thereafter he was dragged out of the car, handcuffed and blindfolded. After he was boxed on the face and stomach, he was pushed inside a car. Verchez saw several cars stop in front of the house. Men in civilian clothes with firearms alighted from the cars. One of the men ordered him to open the door. Suddenly, he heard a gunshot and Aldave, who was then at the back of the house, shouted that someone took a shot at him. After the two ran inside the house, they heard more gunshots. Verchez got a loaded M-16 Armalite rifle from one of the rooms and fired back at his attackers. Aldave looked around and found an Armalite rifle. He also fired back. Balane, still blindfolded and handcuffed, was ordered by Capt. Castaneda to advise his companions to surrender. Hence, he shouted, "Sumuko na kayo si Vic ito." However, someone also shouted "Huwag na kayong sumurender, papatayin nalang namin kayo." The firing continued for 15 minutes, after which the police were able to enter the house. Verchez and Aldave, together with the other occupants of the house, surrendered and were brought to Camp Crame. Verchez and Aldave claimed that at Camp Crame, they were tortured into admitting participation in several bank robberies. They were forced into signing a prepared statement confessing their illegal activities, including having engaged the police officers in a fire fight on August 15, 1985.

Issue: Whether or not there was treachery Held: In convicting appellants of murder, the trial court ruled that the killing of Sgt. Norcio was qualified by treachery as the firing of the guns was sudden and unexpected. We find, however, that treachery was not sufficiently established. For the qualifying circumstance of treachery to be present, two conditions must concur: (a) the employment of means of execution that gives the person attacked no opportunity to defend himself or to retaliate; and (b) that said means of execution was deliberately or consciously adopted (People v. Dela Cruz, 207 SCRA 632 [1992]). There is no showing that appellants deliberately and consciously adopted their mode of attack. Neither is there any showing that they planned to ambush the lawmen, much less that they knew that the lawmen were coming. What is apparent is that appellants were caught by surprise by the lawmen, hence, acting on the spur of the moment, they fired back. Absent the qualifying circumstance of treachery, appellants can only be convicted of homicide for the death of Sgt. Norcio and frustrated homicide for the wounding of Cpl. Noora. 55. PEOPLE VS. BAELLO (UNLAWFUL ENTRY) Nature: Appeal from the judgment of the RTC of Pasi convicting Baello of Robbery with Homicide and sentencing him to reclusion perpetua. Facts: On October 10, 1990, at around 4am, JOHN AMMET BAELLO together with a certain JERRY unlawfully entered, through the 2nd floor window, the house of BRGY. CAPT. EUSTAQUIO BORJA and stole 1 colored TV, 1 stereo cassette player, 1 camera plus assorted jewelry, over all amounting to P64, 000+. Borja woke up at around 5 AM to discover that the front door of their house was open and their TV was missing. He informed his wife and together they checked on their daughter, VERONICA (22) upstairs and discovered that she was dead due to multiple stab wounds. At 6 PM of the same day, the police were able to recover the TV set at the house of Eugenio Tagipa, Baellos brother-in-law. Tagipa pointed to the accused as the one responsible for placing the TV under the stairs of his house. On Oct. 13, at around 5:30 pm, accused was arrested by the Intelligence and Special Operations Unit of Pasig Police. He made an oral admission to the robbery but denied having any knowledge of the killing of Veronica as he was already downstairs stealing the TV while Jerry remained upstairs. Issue: W o N the accused is guilty of robbery with homicide even if he had no knowledge of the killing. Held: Judgment affirmed. (Art. 4- under the doctrine of proximate cause) Even though Baello had no knowledge or participation in the killing of Veronica, he is not absolved from any liability of her death. The rule is likewise settled that when homicide takes place as a consequence, or on the occasion of a robbery, all those who took part in the robbery are guilty as principals of the crime of robbery with homicide unless there is proof that the accused tried to prevent the killing. There was (implied) conspiracy between Jerry and Baello when they went together to the house to rob it These acts as a whole are more than sufficient to establish common design, joint purpose and a community of interest. (RE: UNLAWFUL ENTRY) The aggravating circumstance of unlawful entry was appreciated because the accused, using the 2nd floor window for entry, used an opening not intended for ingress or egress, therefore unlawful entry.

56. PEOPLE VS. RENEJANE (INTOXICATION) Nature: Appeal from the decision of the RTC of Cebu finding Beniano Renejane guilty beyond reasonable doubt of the crime of double murder sentencing him to reclusion perpetua in both cases.

25

Facts: On Nov. 1, 1981, Reynoso and Regino Mara-asin, Pablo Sumandig, Patrolman Mario de Jesus and his wife, Violeta, and Michael Madrigal were in the house of Artemio Ripdos in Brgy. Lamesa, balamban, Cebu. At around 5pm, BENIANO RENEJANE, NICK LABORTE, PAULINO LABORTE and PURISIMA arrived and invited the Mara-asins, Sumandig, de Jesus and Madrigal to the accused home to partake some food. From Renejanos house, they then proceeded to Nick Labortes house and then to his moms house and finally to Paulinos home. Reynoso, however, did not join them at Paulinos house but went instead to his parents house to inform them of his whereabouts. Upon reaching Paulino Labortes house (at around 11 pm), an altercation between de Jesus and Renejane broke out regarding the appellants apprehension by de Jesus for illegal possession of marijuana last Oct. 21, 1981. (Regino was suspected as the informer) While the patrolman was arguing with the accused, Paulino Laborte pulled out a knife and stabbed him. The accused stabbed the victim the 2nd time. Regino was killed too. The lower court appreciated abuse of strength, outraging the victims corpses, disregard of rank and drunkenness against the accused. Held: Decision modified with regard to penalties. In each case, a minimum of reclusion temporal in its maximum to reclusion perpetua. Only the disregard of rank can be appreciated. Drunkenness or intoxication is aggravating if it is habitual or intentional. It can neither be considered mitigating in the absence of proof that the intake of alcoholic drinks was of such quantity as to blur the appellants reason and deprive him of a certain degree of control. As to abuse of strength, it is inherent in treachery and therefore deemed absorbed. Outraging the victims corpses should not be appreciated as assailants were carried away by the intensity of their attack as attested by the nature of the wounds inflicted but had no desire to add ignominy to the offense. 57. PEOPLE VS. CAMANO 115 SCRA 258 (1982) Nature: A review of the death sentence imposed upon the accused Filomeno Camano by the Court of First Instance of Camarines Sur, for the killing of Godofredo Pascua and Mariano Buenaflor. Facts: In 1967, the two victims had a misunderstanding with the accused while fishing along Sagnay River. During this occasion it appears that the accused requested Godofredo Pascua to low his fishing boat with due motor boat owned by Mariano Buenaflor but the request was refused by both. This refusal greatly offended and embittered the accused against the victims. From this time on, the accused begrudged the two, and entertained personal resentment against them. It was noticed that defendant when intoxicated or drunk, used to challenged Mariano Buenaflor to a fight, and announce his evil intention to kill them. On February 17, 1970, after the accused had been drinking liquor, he stabbed twice the victim Godofredo Pascua with a bolo while the latter was walking alone along the barrio street almost in front of the store of one Socorro Buates. The victim, Godofredo Pascua, sustained two mortal wounds for which he died instantaneously. After hacking and stabbing to death Godofredo Pascua, the accused proceeded to the seashore of the barrio, and on finding Mariano Buenaflor leaning at the gate of the fence of his house, in a kneeling position, with both arms on top of the fence, and his head stooping down hacked the latter with the same bolo, first on the head, and after the victim fell and rolled to the ground, after said blow, he continued hacking him, until he lay prostrate on the ground, face up, when the accused gave him a final thrust of the bolo at the left side of the chest causing instant death. The trial court found the accused guilty of two counts of murder with the aggravating circumstance of evident premeditation and intoxication.

Issued: Can intoxication be an aggravating circumstance to the case? Held: The judgment is modified because of attending mitigating circumstance of intoxication. It is respectfully submitted that there was no proof that the accused was intoxicated at the time of the killing other than the bare testimony of Payago that from his house he allegedly saw the accused drinking in his house which is about 30 meters away. The prosecution did not present any police report or doctor's certification that accused was found to be intoxicated at the time of the killing. Moreover, it was not shown by competent evidence that accused purposely became drunk to facilitate the commission of the offense. "If at all, intoxication should be properly appreciated as a mitigating circumstance because it affected accused mental faculties such that it diminished his capacity to know the injustice of his acts and to comprehend fully the consequences of his acts."14 There is merit in the contention. Drunkenness or intoxication is mitigating if accidental, not habitual nor intentional, that is, not subsequent to the plan to commit the crime. It is aggravating if habitual or intentional.1 To be mitigating, it must be indubitably proved.1 A habitual drunkard is one given to intoxication by excessive use of intoxicating drinks. The habit should be actual and confirmed. It is unnecessary that it be a matter of daily occurrence. It lessens individual resistance to evil thought and undermines will-power making its victim a potential evildoer.1 The records of these cases do not show that the appellant was given to excessive use of intoxicating drinks although he used to get drunk every now and then. 58. ATTY. AQUILINA R. ARANETA VS. COURT OF APPEALS NATURE: A petition to review the decision of the then Court of Appeals finding the accused-appellant guilty of the crime of bribery. FACTS: Gertrudes Yoyongco is a widow of Antonio, an employee of the National Irrigation Administration (NIA). When Antonio died, she approached the appellant, a hearing officer of the Workmens Compensation Unit, to inquire on the procedure for filing a claim for death compensation, which upon learning prepared and filed them. After a few days, she went back to ask about the status of her claim. When she saw the appellant, she was told she needed to pay P100 for her claim to be acted upon. She then complained to her brother-in-law, Col. Yoyongco. The former then instructed Carlito Carlos to entrap the accused. Two 50 peso bills were dusted with ultra-violet powder. The two, with Balos, then went to the appellant, with Carlos posting as the nephew of the widow. When the appellant asked for the money, Balos grabbed Araneta and arrested her. ISSUE: W o N it is entrapment or instigation. Held: Decision of the lower court affirmed. Petition dismissed for lack of merit. The petitioner confuses entrapment with instigation. There is entrapment when law officers employ ruses and schemes to ensure the apprehension of the criminal while in the actual commission of the crime. There is instigation when the accused was induced to commit the crime. The difference in the nature of the two lies in the origin of criminal intent. In entrapment, the mens rea originates from the mind of the criminal. The idea and the resolve to commit the crime comes from him. In instigation, the law officer conceives the commission of the crime and suggest to the accused who adopts the idea and carries it into execution. 59. CABRERA VS. PAJARES 142 SCRA 127 (1986) Nature: ADMINISTRATIVE MATTERS in the Supreme Court. Facts: "On January 16, 1985, the complainant Enrico Cabrera gave a sworn statement to the National Bureau of Investigation

26

in Naga City, denouncing the respondent Judge James B. Pajares for having allegedly asked money from him in connection with his case. Cabrera said that in September, 1984 Judge Pajares intimated to him that he needed money. Cabrera said he gave P1, 000.00 to the respondent judge because the latter had been unduly strict, preventing him from making statements during the trial of his case. It appears that the complainant is the defendant in Civil Case No. R-751 which the respondent judge was trying. The case was filed by the complainant's father, Juan Cabrera, and by his half brothers and sisters, for the annulment of the sale made to the complainant of about 28 hectares of land in San Juan, Canaman, Camarines Sur. (See Exhs. 6 and 7-B) Cabrera said he had been advised by his counsel, Atty. Roberto Verdadero, to accommodate any request for money from the respondent so that he would not be unduly hard on the complainant. In September, 1984, according to the complainant, Judge Pajares intimated to him that he needed money. Following his counsel's advice, Cabrera said he expressed willingness to help the judge financially and, the following day after their meeting, gave him P1, 000.00. However, according to Cabrera, after two months (i.e., before Christmas of 1984), Judge Pajares again told him that he needed money. Cabrera said the judge saw him in front of the Hall of Justice in Naga City and called him. It was then, according to him, that he decided to denounce the judge to the authorities; Cabrera asked the assistance of the NBI in entrapping Judge Pajares. Issue: Whether or not the accused I guilty of the crime of indirect bribery. Held: ACCORDINGLY, respondent Judge is hereby dismissed from the service, with forfeiture of all retirement benefits and pay and with prejudice to reinstatement in any branch of the government or any of its agencies or instrumentalities. The Clerk of Court is hereby ordered to return the ten P100, 00 bills (Exhibits D-1 to D-10) to the complainant Atty. Enrico M. Cabrera. This decision is immediately executory. 60. PEOPLE VS. VENERACION (ACTS NOT COVERED BY LAW) 249 SCRA 244 (1995) Nature: Petition for certiorari to review a decision of the Regional Trial Court of Manila, Br. 47 Facts: Two criminal cases were consolidated to Branch 47 of RTC of Manila and presided over by respondent Judge Lorenzo Veneracion regarding the brutal rape and killing of Angel Alquiza, a 7 year old girl on August 2, 1994. After trial and presentation of the evidence of the prosecution and defense, the trial court rendered the decision on January 31, 1995 finding the defendants Henry Lagarto and Ernesto Cordero guilty beyond reasonable doubt of the crime of Rape with Homicide and sentenced both accused with the penalty of reclusion perpetua instead of imposing the penalty of death as provided for in RA 7659, Sec. 11. Issue: W o N the respondent judge acted with grave abuse of discretion and in excess of jurisdiction when he failed and/or refused to impose the mandatory penalty of death after finding the accused guilty of the crime rape with homicide. Held: The petition is granted and the case is remanded to the RTC for the imposition of the penalty of death. The Rules of Court mandates that after an adjudication of guilt, the judge should impose the proper penalty and civil liability provided for by the law on the accused. In the case at bar, a judge fully aware of the appropriate provisions of the law, refuses to impose a penalty to which he disagrees. In so doing, he acted without or in excess of his jurisdiction with grave abuse of discretion amounting to a lack of jurisdiction. The law plainly and unequivocably provides that when by reason or on the occasion of rape, a homicide is committed, the penalty shall be death. The provision leaves no room for the exercise of discretion on the part of the trial judge to impose a penalty under the circumstances described, other than the

penalty of death. As long as the death penalty remains in the statute books, and as long as our criminal law provides for its imposition in certain cases, it is the duty of judicial officers to respect and apply the law regardless of their private opinions. The only function of the judiciary is to interpret the laws and, if not in disharmony with the Constitution, to apply them. 61. PEOPLE VS. NUNAG 173 SCRA 274 (1989) Nature: An appeal from the judgment of the Court of First Instance of Pampanga, Branch I Facts: On May 1978 at about 7:30 pm, Lorenza Lopez, then about fifteen and a half years old, was watching a television program in the house of her neighbor, Carmen Laxaniana. She stood outside the house and peeped through the open window. As she was standing there, she saw the accused Mario Nunag, one of her neighbors, coming towards her Mario Nunag was staggering and appeared to be drunk. The moon was bright and she really recognized him. Mario Nunag came to her and asked her to go with him. But she refused, so that Nunag held her by the hand and poked a knife at her stomach and threatened to kill her. Nunag then placed something in her month and led her to a nearby rice field, about 15 meters behind the house of Carmen Laxamana. 1 Very soon thereafter, they were joined by the other accused Arnel Mandap, Efren Salangsang, Danio Carpio and Diosdado Manalili, who were also very well known to her. After conferring in whispers, Arnel Mandap and Efren Salangsang held her hands, while Danilo Carpio and Diosdado Manalili held her feet, and forced her to be on the ground. She struggled to free herself, but the accused held her tightly. Mario Nunag then undressed her and had sexual intercourse with her, at the same time fondling her breasts. She felt pain in her vagina. After Mario Nunag had finished, Arnel Mandap followed. After Arnel Mandap had finished, she lost consciousness and regained it while Diosdado Manalili was abusing her. Then, the five accused left, after warning her not to report the incident to anybody. Otherwise, they would kill her, her parents and brothers. The complainant felt pains and aches all over her body, especially in her breasts and vagina. She rested for a while and when the pains had somewhat subsided, she went home. She did not report the incident to anybody for fear of what the accused might do to her and her family. Issue: W o N the accused should be guilty of 5 counts of rape by virtue of conspiracy existing among them. Held: Judgment modified. It would appear, however, that there is no conclusive evidence that the accused-appellants Danilo Carpio and Efren Salangsang had sexual intercourse with the complainant, since the complainant said that she lost consciousness after the second man (Arnel Mandap) the first being Mario Nunag-had sexually abused her and she regained consciousness while Diosdado Manalili was abusing her sexually, and that she merely assumed that Danilo Carpio, and Efren Salangsang had also sexually abused her. Consequently, each of the five (5) accusedappellants must be found guilty of three (3) district and separate crimes of rape, the first three, namely, Mario Nunag, Arnel Mandap and Diosdado Manalili, by direct act and participation and the other two, namely Danilo Carpio and Efren Salangsang, by indispensable cooperation. 62. PEOPLE VS. DELA CERNA 21 SCRA 569 (1967) Nature: An appeal from a decision of the Court of First Instance of Cotabato finding the accused guilty for double murder. FACTS: Early in the morning of February 3, 1958, Rafael Cabizares, accompanied by his wife, Hospicia, his brothers Margarito and Romualdo, and his sons Gumercindo, Marcelo, Casiano, Juan and Lamberto, left Barrio Cebuano headed for the poblacion of Tupi, Cotabato, bringing five sacks of corn loaded on a bull cart to be milled in Tupi, Juan, Marcelo and Lamberto, who were all minors, were then going to school. Upon

27

approaching a hilly part, they had to stop since the carabao could not pull the bull cart uphill. Rafael then requested his two brothers and his son Gumercindo to accompany him up the hill and carry on their backs the sacks of corn. With Rafael leading, the four proceeded uphill. As the four approached Sulpicio de la Cerna's house oiltop of the hill and were about to put down the sacks of corn, appellant Sulpicio, who was in the house, fired at and hit Rafael, who fell down. Sulpicio then ordered his companions to burn his house so that they would have an excuse. Meanwhile, Casiano, Gumercindo, Marcelo and Romualdo brought the wounded Rafael Cabizares to the house of the latter's father, Demetrio, 100 meters away. Felisa Bastismo, Rafael's mother, Ursula Cabizares and Segundino Cabizares were there at the time. After the group reached the house, Rafael's wounds were washed with hot water and then he was brought inside the third room of the house. Subsequently, appellant Sulpicio and the other accused arrived at the premises, armed with firearms, bolos and canes. They stoned the house and thrust their bolos thru the bamboo walls and flooring. Finding that there were women inside the house, the accused ordered them to get out or else they would be killed also. As Felisa Bastismo and Ursula Cabizares alighted from the besieged house, Marcelo Cabizares followed them, and although held by accused Conrado Pardillo and boxed by Serapio Maquiling, he was able to escape to the nearby forest. Serapio Maquiling then climbed up the window of the kitchen, and with the carbine which he got from appellant Sulpicio de la Cerna, shot at Rafael Cabizares who was sitting in the third room. At this moment, Casiano Cabizares jumped down from the house thru the kitchen door and ran away. Serapio Maquiling followed him and shot the latter at the back, killing him a few meters away from Demetrio's house. Appellant Sulpicio de la Cerna then got back the carbine, climbed up the house and fired once more at Rafael, who was now lying down on the floor, killing him finally. Thereafter, the cadaver of Casiano Cabizares was tied to a bamboo pole, carried by accused Ramon Alquizar and one Wilfredo Malias (at large) and placed near the burned house of Sulpicio de la Cerna, as some of the accused followed while the rest proceeded to Rafael's house. ISSUE: Whether the five appellants are all guilty as principals? Held: The five appellants guilty as co-principals in the murder of Rafael Cabizares. The positive identification of the several prosecution witnesses must prevail over the alibis proffered by these appellants. Their presence and active participation in the meeting in Abapo's house make them actual conspirators in the killing of Rafael. They were also present and zealously participating in the execution of their criminal design, giving a carbine magazine and instructionns to appellant Rotor, threatening Rafael and giving encouragement to Sulpicio to shoot at the latter. They were among those who laid siege to Demetrio's house and left together with the others after finally accomplishing their criminal deeds as agreed upon. Appellants Bautista and Matchoca, are therefore also liable as co-principals in Rafael's murder. Regarding motive, it was proved that both were among those involved in the land conflict with Rafael Cabizares and were among the respondents in the case before the Agrarian Court The aggravating circumstance of treachery, applicable against appellant Sulpicio de la Cerna only, is offset by his voluntary surrender after the incident. This mitigating circumstance however cannot benefit the remaining appellant who did not voluntary surrender. For all the appellants, therefore, the penalty for Rafael Cabizares' murder must be imposed in the medium period. For the killing of Casiano Cabizares appellant Sulpicio de la Cerna must be acquitted. 63. PEOPLE VS. DELA CRUZ 97 SCRA 385 (1980) Nature: Automatic review of the decision of the Court of First Instance of Basilan City. Facts: Antonio Yu owned 200 hectares of rubber and coconut land in Lantawan, Isabela, Basilan City. The victim, Yu Chi

Chong, is his brother. The accused Agapito dela Cruz was an overseen of Antonio Yu for no less than 10 years. Eleven were charged for kidnapping and slaying of Yu Chi Chong but only Agapito dela Cruz, Jamas Jumaidi and Oyong Asidin were apprehended. The rest have remained at large. On September 24, 1968, the city fiscal asked for the discharged of Jamas Jumaidi and Oyong Asidin to be utilized as state witness. The Trial Court granted the motion. The evidence of the prosecution mainly rested on the testimonies of the two discharged witnesses and that on Mohammad Sagip who all pointed to Agapito as mastermind in the kidnapping of Yu Chi Chong. Mohammad Sagip testified that sometimes in October, 1967, the accused Agapito met with him, Alih Itum and a certain Asmad at which he proposed to them the killing of Antonio Yu and the kidnapping of the younger brother. Yu Chi Chong for a ransom. Apparently, Asmad contacted some people in Jolo, Sulu, for the purpose the accused herein among them. The two discharged witness narrated what transpired thereafter. In Isabela, the group waited in the ambush spot for the truck that would carry sacks of copra and supposed to be the two brothers but only Yu Chi Chong was in the truck because Antonio Yu had to go Tairan on some other business Isabelo Mancenido accompanied YU Chi Chong in the truck. When the truck came to a half near the ambush spot, the ambushers approached it and dragged Yu Chi Chong and Isabelo Mancenido there from. Shortly after, the group released Mancenido upon the latter's please of mercy. Upon reaching Bancao Sapa they found that the tide was low, rendering impossible for them to reach their boat. While waiting, Yu Chi Chong, in an attempt to escape, struck Angih with a piece of wood and tried to grab the gun of the latter but failed. Angih, in anger, fired at Yu Chi Chong several times, killing him. They dumped the body in the middle of the sea and it was never recovered. As a defense. Agapito claimed that on March 5, 1968 he reported for work in the land of Antonio Yu and that in the evening he had dinner in the house of Alfonso Flores and slept there that night. He strongly asserted that he never left that house from 7:30 in the evening until 6:00 in the morning. Issue: Whether or not the accused be held principally liable for the death of Yu Chi Chong. Held: Appellant is guilty as principal by inducement due to the fact that he was the one who lays down the strategy of the crime, he knew the route that the truck would take and approximate time that it was to pass by. He even selected the ambush place. He also presented the strongest temptation, a pecuniary gain in the form ransom, which was the determining factor of the commission of the crime by his co-accused. Without him, the crime would not have been conceived, much less committed. 64. PEOPLE VS. MONTELEAGRE 161 SCRA 700 (1988) Nature: Appeal from judgment of the Court of First Instance of Cavite City. Facts: At about 11:30 in the evening of March 11, 1983, while Edmundo Abadilla was eating at the Meding's Restaurant in Cavite City, he detected the smell of marijuana smoke coming from the nearby table. He then went outside to report the matter of Pfc. Renato Camantigue a policeman Camantigue joined Abadilla in the restaurant. He approached the two and collard both of them, saying "namamarijuana kayo, ano?" While Camantigue was holding the two, Montealegre with his right hand and Capalad with his left hand, Capalad suddenly pulled out the knife tucked in his waist and started stabbing Camantigue. Camantigue let loose Montealegre to draw the gun from his holster but Montealegre thus released restrained Camantigue's hand to prevent the latter in defending himself Montealegre used both his hands for this purpose as Capalad continued stabbing the victim. Capalad fled into the dark alley, while the accused escaped during the confusion Capalad was late found slumped in an alley with a bullet wound in his chest. Neither Camantigue nor Capalad survived, both expiring the next day.

28

Issue: Whether or not Montealegre be held liable for the death of Pfc. Renato Camantigue Held: The accused-appellant was correctly considered a coprincipal for having collaborated- with Capalad in the killing of the police officer. The two acted in concert, with Capalad actually stabbing Camantigue seven times and the accusedappellant holding on to the victim's hands to prevent him from drawing his pistol and defending himself. While it is true that the accused-appellant did not himself commit the act of stabbing, he was nonetheless equally guilty thereof for having prevented Camantigue from resisting the attack against him. The accused-appellant was a principal by indispensable cooperation under Article 17, par. 3, of the Revised Penal Code. As correctly interpreted, the requisites of this provision are: "(1) participating in the criminal resolution, that is, there is either anterior conspiracy or unity of criminal purpose and intention immediately before the commission of the crime charged; and (2) cooperation in the commission of the offense by performing another act without which it would not have been accomplished." 65. PEOPLE VS. MANDOLADO 123 SCRA 133 91983)

participation. There being ample evidence of their criminal participation but a doubt exists on the nature of their liability and so the court should favor the milder form or responsibility, which is that of being a mere accomplice. *note: Virgilio died while case was pending appeal 67. PEOPLE VS. TALINGDAN 84 SCRA 19 (1978) NATURE: APPEAL from the judgment of the Court of First Instance of Abra. FACTS: Prior to the violent death of Bernardo Bagabag on the night of June 24, 1967, he and appellant Teresa Domogma and their children, lived together in their house at Sobosob, Salapadan. For sometime, however, their relationship had been strained and beset with troubles, for Teresa had deserted their family home a couple of tunes and each time Bernardo took time out to look for her. Bernardo had gotten wind that illicit relationship was going on between Talingdan and Teresa, and during a quarrel between him and Teresa, he directly charged the latter that should she get pregnant, the child would not be his. between 10:00 and 11:00 o'clock the following Friday morning, Bernardo's daughter, Corazon, who was then in a creek to wash clothes saw her mother, Teresa, meeting with Talingdan and their co-appellants Magellan Tobias, Augusto Berras and Pedro Bides in a small hut owned by Bernardo. Teresa Domogma noticed the presence of her daughter; she shoved her away saying "You tell your father that we will kill him". Saturday, June 24, 1967, while the same 12-year old daughter of Bernardo was cooking food for supper in the kitchen of their house, she saw her mother go down the house through the stairs and go to the yard where she again met with the other appellants. She noted that the appellants had long guns at the time. His time, she informed her father about the presence of persons downstairs, but Bernardo paid no head to what she said. At that moment, he was suddenly fired upon from below the stairs of the "batalan". The four accused then climbed the stairs of the "batalan" carrying their long guns and seeing that Bernardo was still alive, Talingdan and Tobias fired at him again. Issue: Whether or not Teresita is guilty for being an accessory to the offense committed by her co-accused. HELD: True it is that the proof of her direct participation in the conspiracy is not beyond reasonable doubt, for which reason, she cannot have the same liability as her co-appellants. Indeed, she had no hand at all in the actual shooting of her husband. Neither is it clear that she helped directly in the planning and preparation thereof, albeit the SC was convinced that she knew it was going to be done and did not object. But this is not saying that she is entirely free from criminal liability. There is in the record morally convincing proof that she is at the very least an accessory to the offense committed by her co-accused. She was inside the room when her husband was shot. As she came out after the shooting, she inquired from Corazon if she was able to recognize the assailants of her father. When Corazon identified appellants Talingdan, Tobias, Berras and Bides as the culprits, Teresa did not only enjoin her daughter not to reveal what she knew to anyone, she went to the extent of warning her, "Don't tell it to anyone. I will kill you if you tell this to somebody." Later, when the peace officers who repaired to their house to investigate what happened, instead of helping them with the information given to her by Corazon, she claimed she had no suspects in mind. In other words, whereas, before the actual shooting of her husband, she was more or less passive in her attitude regarding her coappellants' conspiracy, known to her, to do away with him, after Bernardo was killed, she became active in her cooperation with them. These subsequent acts of her constitute "Concealing or assisting in the escape of the principal in the crime" which makes her liable as an accessory after the fact under paragraph 3 of Article 19 of the Revised Penal Code.

66. PEOPLE VS. DOCTOLERO 193 SCRA 632 (1991) Nature: Appeal from the decision of the then Court of First Instance of Lingayen, Pangasinan, Br. 2, convicting Ludovico Doctolero, Conrado Doctolero and Virgilio Doctolero of the crime multiple murder and unspecified physical injuries. Facts: At about 6:30 pm of Nov. 8, 1970, Marcial Sagun and his wife, Maria Oviedo-Sagun and Lolita de Guzman-Oviedo were on their way home to Barrio Binday. They came from the field where they bundled their harvest. Upon reaching a crossing of the road in bo. Binday they met the accused Ludovico Doctolero who, without warning and without cause or reason, held the left shoulder of Marcial Sagun with his left hand and struck her with a bolo. Paciencia Sagun-Diamoy testified that while she was cleaning palay in the yard of her uncle, the deceased Marcelo Doctolero, she saw the accused, Ludovico. Conrado and Virgilio throw stones at the house of Marcial Sagun. While throwing stones, Ludovico allegedly shouted for the men in the house to come out. Paciencia Sagun-Diamoy went towards the house of Marcial Sagun and saw the three accused, Ludovico, Conrado, and virgilio, coming down from the house going towards her. At about that time, Marcelo Doctolero, the half-brother of Antonio, and the uncle of the three accused was going towards the house of Marcial. The accused struck Marcelo several times with their bolo. Maria Oviedo-Sagun corroborated the testimony of Paciencia. As she was about to go to their house to get their children, she saw the three accused going up the house then she heard Epifania, her adopted mother, shouting at her Enieng, your children! then she saw the three accused coming down the house. The lower court held that Conrado and his brother Viregilio participated as accomplices in the slaying of the women and the infliction of injuries on the child. Held: The decision of the trial court is modified regarding the penalties rendered. The Trial Court correctly ruled that Conrado participated as an accomplice. It is impossible that the appellant did not know or were not aware that his brother, Ludovico, was brutally killing the two women and wounding the child. It is, therefore, reasonable to believe that Conrado merely stood by as his brother was murdering deceased women, ready to lend assistance. Indeed, there is no question that his presence gave his brother the encouragement and the reliance to proceed as he did proceed in committing the heinous crimes. Whatever doubt the court a quo entertained in the criminal responsibility of Conrado did not refer to whether or not he was liable but only with regard to the extent of his

29

68. PEOPLE VS. FERRER 48 SCRA 382 (1972) NATURE: Special civil action in the Supreme Court to review on certiorari the constitutionality of the Anti-Subversion Act (RA 1700) FACTS: On March 5, 1970, a criminal complaint for violation of section 4 of RA 1700 was filed against Felicaino Co in the CFI of Tarlac. Co moved to quash on the ground that the said RA was a bill of attainder. Meanwhile, Nilo Tagay, et al. had been charged for the violation of RA 1700. On July 21, 1970, he moved to quash the charges on the grounds that the said RA was a bill of attainder, and that it was vague and embraced more than one subject not expressed in its title. Resolving the constitutional issues raised, the trial court declared the stature void on the grounds that it is vague and overboard, and dismissed the informations against the two accused. Issue: W o N RA 1700 is a bill of attainder. Held: The questioned resolution of the lower court is set aside, and the two cases are remanded to the court for trial on the merits. It is not a bill of attainder. A bill of attainder is a legislative act which inflicts punishment without trial. Its essence is the substitution of a legislative for a judicial determination of guilt. When the act is viewed in its actual operation, it will be seen that it does not specify the Communist Party of the Philippines or the members thereof for the purpose of punishment. The term Communist Party of the Philippines is used solely for definitional purposes. Its focus is not on the individuals but on conduct. As to the membership, the act does not punish nominal membership but membership that is knowing or active, with specific intent to further the illegal objectives of the Party. It is only when a statute applies either to named individuals or to easily ascertainable members of a group in such a way as to inflict punishment on them without judicial trial does it become a bill of attainder. A bill of attainder is similar to an ex post facto law. RA 1700 is not such because it punishes only acts committed after the approval of such act. Only those who knowingly, willfully and by over acts affiliate themselves with, become or remain members after June 20, 1957 are punished. 69. GUMABON, et al. VS. DIRECTOR OF BUREAU OF PRISONS (PROSPECTIVITY) Nature: Original petition in the Supreme Court for Habeas Corpus Facts: The CFI of Manila sentenced Mario Gumabon on May 5, 1953 to reclusion perpetua for the complex crime of rebellion with multiple murder, robbery, arson and kidnapping. Gumabon, together with Agapito, Palmares, Padua and Bagobagol are invoking habeas corpus, citing the case of People vs. Hernandez that no such complex crime of rebellion exist under Art. 13 of RPC. According to the case of Hernandez, rebellion cannot be complexed with other common crimes since such common crimes assume political complexion of the main crime of which they are mere ingredients and consequently cannot be punished separately from the principal offense. They cite the ruling of Hernandez to be retroactively applied to them by virtue of Art. 22 of RPC. (People vs. Lava- the leaders of the rebellion have been freed while their followers, the petitioners, suffer life imprisonment) Issue: W o N the ruling in the Hernandez case should retroact to the case at bar. Held: Petition for habeas corpus granted and the petitioners are set free. The actual case of the petitioners is that at the time of their conviction, it was believed that the crime committed by them was punishable by life imprisonment, but the court has subsequently judicially determined it not be so and that the maximum penalty imposable is prision mayor or 12 years.

Petitioners-convicts are entitled to the benefits of this later judicial declaration, just as if a statutory amendment had been enacted. The writ prayed for should be issued since it is the only means of giving retroactive effect to a penal provision favorable to the accused where the trial judge has lost jurisdiction over the case. Furthermore art. 22 of the RPC extends its benefits even to convicts serving sentence, and the only legal remedy open to them is the writ of habeas corpus. 70. PEOPLE VS. GATWARD 267 SCRA 785 (1997) Nature: Appeal from a decision of the Regional Trial Court of Pasay City, Branch. Facts: Gatward was charged with violating Section 4 of the Dangerous Drugs Act of 1972 for having transported hereon contained in separate carton envelopes with a total weight of 5237 70 grams which is legally considered as a prohibited drug on or about the 31st day of August 1994, in the vicinity of the Ninoy Aquino International Airport, Pasay City. Meanwhile, U Aung Win was indicted for transgressing Section 3 of the Dangerous Drug Act 1972 for having imported and brought in the Philippines 5570 80 grams of herein which is legally considered as a prohibited drug. Gatward pleaded not guilty of the charge when arranged while U Aung Win pleaded guilty. Both were convicted of the offense charged. The penalty to de imposed under the Dangerous Drug Act shall range from reclusion perpetua to death. In imposing the proper penalty, the trial court declared that the penalty of "reclusion perpetua to death" shall have the following periods Death as the maximum thirty (30) years and one (1) day to forty (40) years as the medium and twenty (20) years and one (1) day to thirty (30) years as the maximum it regarded reclusion perpetua as an indivisible penalty. Thus Gatward was sentenced to suffer the penalty of imprisonment for thirty five (35) years of reclusion perpetua and to pay a fine of Five Million Pesos (5,000,000.00) in view of the presence of one (1) mutilating circumstance of voluntary plea of guilty, without any aggravating circumstance. Issue: Whether or not reclusion perpetua a divisible penalty. Held: The trial court had by considering reclusion perpetua as a divisible penalty imposed an unauthorized penalty on both accused which would remain uncorrected if the appeal had been allowed to be withdrawn. In fact it would stamp a mlul obstantient on a penalty that in law does not exist and which error initially committed by the Court in another case on which the trial court held had already been set aright by these. The penalty of reclusion perpetua is now accorded a "defined duration" ranging from twenty (20) years and one (1) day to forty (40) years, through the amendment introduced by RA 7659 to Article 27 of the Revised Penal Code. The Court held that in spite of the amendment putting the duration of reclusion perpetua at 20 years and 1 day to 40 years; it should remain as an indivisible penalty since there was never any intent on the part of Congress to reclassify it into a division penalty. The judgment of the court a quo, specifically with regard to the penalty imposed on accused-appellant Nigel Richard Gatward and that of accused U Aung Win was MODIFIED in the sense that both accused were sentenced to serve the penalty of reclusion perpetua in its entire duration and full extent. 71. PEOPLE VS. FORMIGONES 87 PHIL 658 (1950) Nature: An appeal from the decision of the CFI Camarines Sur finding the appellant guilty of parricide, sentencing him to reclusion perpetua. Facts: On Nov. 1946, defendant Abelardo Formigones lived on his farm in Bahao, Libmanan, Sipocot, Camarines Sur w/ wife Julia Agricola and 5 children. Sometime later they went to Binahian, Sipocot to seek employment as harvesters of palay, lived w/ his half-brother, Zacarias Formigones

30

On Dec. 28, 1946, while Julia was sitting on top of the stairs of the house, w/o any previous quarrel or provocation, Abelardo stabbed his wife w/ a bolo blade punctured Julia's back and right lung, caused a severe hemorrhage resulting in her death. Julia toppled down the stairs. Abelardo followed and carried and laid her on the floor of the living room and he had down beside her. Their eldest daughter Irene, who witnesses the incident, shouted for help, people came in response. The accused suspected them that they are maintaining illicit relations with his half-brother. Held: Appellant is guilty of parricide. Judgment of the lower court affirmed w/ modification, appellant will be credited with of any preventive imprisonment he has undergone. The SC convinced that the appellant is not imbecile. During his marriage of 16 years, he had not done anything to warrant an opinion that he was an imbecile. Killing his wife whom he suspect of being unfaithful to him in the belief that he was vindicating his honor could not be regarded as an imbecile. His feeling of jealousy may or may not be true bee in is statement, observed that the his half brother who was living in his grandmother not only frequented his house after they have transferred, but also sleep there during the night this aroused or partly confirmed Abelardo's suspicions. It was an act of remorse: lying beside his wife for hours after he killing her, he made no effect to flee and compel the police to hunt him down and arrest him, he readily admitted that he killed his wife in his statement 72. LACANILAO VS. CA 62 SCRA 563 (1988) Nature: A petition which calls for our exercise of the power of Judicial Review- question of Law. Facts: The court Of First Instance of Manila finding the petitioner guilty of homicide for the death of one Ceferino Erese, and was sentenced to an indeterminate penalty of six years and one day of prision mayor as minimum to fourteen years and one day of reclusion temporal as the maximum. The petitioner appealed to the CA which modified the decision of the CFI changing the maximum penalty from fourteen years and one day to twelve years and one day respectively. The court of appeals that Bernardo Lacanilao acted in the performance of his duty but that the shooting of the victim was not the necessary consequence of the due performance thereof, therefore, crediting to him the mitigating circumstance of incomplete fulfillment of duty. In the words of the respondent court saying while the appellant should be commended for responding to the call of duty when he tried to stop the victim and the latters companions from their drunken and disorderly conduct, nevertheless he cannot be exonerated from overdoing his fulfillment of duty to the extent of admittedly shooting and thereby killing said victim. Thus, the respondent court lowered the penalty merely by one period applying Art. 64 par. 2 of the RPC. The two conditions that must be met to justify fulfillment of duty are the ff: 1) that the accused acted in the performance of a duty or in the lawful exercise of a right or office, and 2) that the injury or offense committed be necessary consequence of the due performance of such duty or the lawful exercise of such right of office. In the case at bar, only the first condition was met. Issue: W o N art. 69 of the RPC is applicable to the case at bar. Held: Yes. Art. 69 is applicable, for the requirement of having a majority of the conditions be present to lower the penalty by one or two degrees is immaterial because there are only two conditions in order that the circumstance in no. 5 of art. 11 may be taken into account. The petition is granted in so far as it seeks to the modification of the penalty pursuant to art. 69 of the RPC and the ruling in Oanis. The petitioner is hereby sentenced to an indeterminate penalty of from two years, four months and one day of prision correctional to eight years and one day of prision mayor.

73. PEOPLE VS. GERONIMO 100 PHIL 99 (1956) Nature: Appeal from a judgment of the Court of First Instance of Camarines Sur Facts: In information filed by the provincial Fiscal in the CFI of Camarines Sur, appellant Federico Geronimo and many others were charged with the complex crime of rebellion with murder, robberies and kidnapping. Accused Federico Geronimo first entered a plea of not guilty to the information. When the case was called for trial, he asked the information of the court to substitute his original plea with one of guilty, and was allowed to change his plea. On the basis of the plea of guilt, the fiscal recommended that the penalty of life imprisonment be imposed upon the accused, his voluntary plea of guilt being considered as a mitigating circumstance. Geronimos counsel argued that the penalty imposable upon the accused was only prision mayor, for the reason that there is no such complex crime as rebellion with murder, robberies and kidnapping, because the crimes of murder, robbery and kidnapping being the natural consequences of the crime of rebellion, the crime charged against the accused should be considered only as simple rebellion. The trial court rendered judgment finding the accused guilty of the complex crime of rebellion with murder, robbery and kidnapping and giving him the benefit of the mitigating circumstance of voluntary plea of guilt, sentenced him to suffer the penalty of Reclusion Perpetua. Issue: Whether the crime committed is the complex crime of rebellion with murder, robbery and kidnapping, or simple rebellion. Ruling: The decision appealed from was modified and the accused convicted for the simple crime of rebellion and considering the mitigating effect of his plea of guilt, accusedappellant Federico Geronimo was sentenced to suffer 8 years of prision mayor. The acts when committed as a means to or in furtherance of the subversive ends become absorbed in the crime of rebellion, and cannot be regarded or penalized as distinct crimes in themselves. In law they are part and parcel of the rebellion itself, and cannot be considered as giving rise to a separate crime that would constitute a complex one with that of rebellion. If the killing, robbing and kidnapping were done for private purposes or profit, without any political motivation, the crime would be separately punishable and would not be absorbed by the rebellion. But even then, the individual misdeed could not be taken with the rebellion to constitute a complex crime, for the constitutive acts and intent would be unrelated to each other; and the individual crime would be a means necessary for committing the rebellion as it would not be done in preparation or in furtherance of the latter. 74. PONCE ENRILE VS. SALAZAR 86 SCRA 217 (1990) Nature: An appeal from a judgment of the Court of First Instance of Camarines Sur. Facts: In the afternoon of February 27, 1990, Senate Minority Floor Leader Juan Ponce Enrile was arrested by law enforcement officers led by Director Alfredo Lim of the National Bureau of Investigation on the strength of a warrant issued by Hon. Jaime Salazar of the Regional Trial Court of Quezon City Branch 103, in Criminal Case No. 9010941. The warrant had issued on an information signed and earlier that day filed by a panel of prosecutors composed of Senior State Prosecutor Aurelio C. Trampe, State Prosecutor Ferdinand R. Abesamis and Assistant City Prosecutor Eulogio Mananquil, Jr., charging Senator Enrile, the spouses Rebecco and Erlinda Panlilio, and Gregorio Honasan with the crime of rebellion with murder and multiple frustrated murder allegedly committed during the period of the failed coup attempt from November 29 to December 10, 1990. Senator Enrile was taken to and held overnight at the NBI headquarters on Taft Avenue, Manila, without bail, none having been recommended in the information

31

and none fixed in the arrest warrant. The following morning, February 28, 1990, he was brought to Camp Tomas Karingal in Quezon City where he was given over to the custody of the Superintendent of the Northern Police District, Brig. Gen. Edgardo Dula Torres. On the same date of February 28, 1990, Senator Enrile, through counsel, filed the petition for habeas corpus herein (which was followed by a supplemental petition filed on March 2, 1990), alleging that he was deprived of his constitutional rights. Issue: Whether or not there is a complex crime of rebellion Held: The petitioners' case does not fall within the Hernandez ruling because-and this is putting it very simply-the information in Hernandez charged murders and other common crimes committed as a necessary means for the commission of rebellion, whereas the information against Sen. Enrile et al. charged murder and frustrated murder committed on the occasion, but not in furtherance, of rebellion. Stated otherwise, the Solicitor General would distinguish between the complex crime ("delito complejo") arising from an offense being a necessary means for committing another, which is referred to in the second clause of Article 48, Revised Penal Code, and is the subject of the Hernandez ruling, and the compound crime ("delito compuesto") arising from a single act constituting two or more grave or less grave offenses referred to in the first clause of the same paragraph, with which Hernandez was not concerned and to which, therefore, it should not apply. 75. PEOPLE VS. ESCOBAR 157 SCRA 541 (1988) Nature: APPEAL from the decision of the Regional Trial Court of Quezon City, Br. 97. Leviste, J. Facts: One of the alleged co-conspirator Amadeo Abuyen alias Roberto Alorte, was formerly a co-security guard of appellant Juan Escober at the Bee Seng Electrical Supply, Inc., a family corporation owned by the couple Vicente Chua and Lina Chua. About 4 months prior to the incident, Abuyen was relieved by Domingo Rocero for being always absent and found sleeping while on duty. "At the time of the incident on December 3, 1982, Rocero's tour of duty was from 7:00 in the morning to 7:00 in the evening. He left his post at about 7:30 P.M. that evening after he was relieved by appellant Juan Escober. On his way home, he passed by Barangay Balingasa in Balintawak, where he saw Amadeo Abuyen in the store of Colonel Samson drinking beer with three companions, one of whom he later identified as the appellant Macario Punzalan, Jr. "After Rocero had left his point, Vicente Chua went to his office at the Bee Seng Electrical Supply as he usually does after office hours, accompanied by his 13-year old son Irvin and 6-year old daughter Tiffany. On their way, he saw appellant Escober at his post. At the office, the two children watched a television program, as their father proceeded to the bathroom to take a bath "Meanwhile, Abuyen and his three companions rode a tricycle and proceeded to the Bee Seng Electrical Supply. Upon alighting thereat, Abuyen knocked at the little door of the gate. Appellant Escober peeped thru the hole and opened the door. Then after Abuyen had talked with Escober, the former asked Punzalan to wait outside, while he (Abuyen) and his two other companions went inside "At this juncture, the victims 'mother, Mrs. Lina B. Chua, left their residence to join her husband and two children. On her way, she noticed that the pedestrian gate was wide open with the appellant Punzalan standing there. She shouted why the gate was opened, but nobody answered. Suddenly, she heard of shot coming from the direction of the garage; and when she looked thereat, she saw Abuyen and the appellant Escober walking towards the gate. So, she rushed back inside the house to contact her husband through the intercom. But since the intercom was out of order, she hurriedly went outside and met appellant Escober who volunteered the information that he was not hit' 'Upon the other hand, Vicente Chua was inside the bathroom, when he heard the gunshot. He hurriedly went out and saw her son Irvin lying on the sofa while Tiffany was lying on the floor, both mortally wounded. Beside her daughter, he

saw a scissor blade [full of blood. He also observed that everything was scattered in his office, with all his drawers opened. Later, he found out that the P5, 000.00 cash he kept in one of the drawers was lost Juan Escober, together with four unidentified persons designated as John Doe, Peter Doe, Richard Doe and Juan Doe, were charged with the crime of Robbery with Homicide before the Regional Trial Court of Quezon City in Information dated December 9, 1982. He entered a plea of "Not Guilty" with the assistance of counsel Atty. Hipolito de Peralta upon arraignment on March 2, 1983. Punzalan was later included in the charges. Issue: Whether or not the accused participated in a complex crime. Held: Decision reversed and set aside. The accused is acquitted. The act of opening a gate upon hearing a knock is by itself an innocent gesture. One who imputes an evil motive or purpose thereto must prove his allegations convincingly. In the case at bar, even if the version of Macario Punzalan, Jr, that Escober opened the gate at the knock of the alleged mastermind Amadeo Abuyen/Roberto Alorte were to be believed, the same would not constitute sufficient and convincing proof that Escober had knowledge of the nefarious plan. The worse that could be attributed to him is lack of better judgment or laxity in the performance of his duties as a security guard in having failed to exercise the minimum precaution dictated by his occupation to exclude from the premises being guarded persons who have not demonstrated any legitimate reason for getting in. 76. MEJORADA VS. SANDIGANBAYAN 51 SCRA 339 (1987) Nature: A petition for certiorari seeks to reverse the May 23, 1979 decision of the Sandiganbayan finding the accused Arturo A. Mejorada in Criminal Cases Nos. 002-009 guilty beyond reasonable doubt of violating Section 3(E) of Republic Act No. 3019, otherwise known as the Anti-Graft and Corrupt Practices Act Facts: Arturo A. Mejorada was a public officer who was first employed as a temporary skilled laborer in the Bureau of Public Works on March 16, 1947, and then as right-of-way agent in the Office of the Highway District Engineer, Pasig, Metro Manila, from February, 1974 up to December 31, 1978. As a right-ofway agent, his main duty was to negotiate with property owners affected by highway constructions or improvements for the purpose of compensating them for the damages incurred by said owners. Among those whose lots and improvements were affected by the widening of the proposed Pasig-Sta. CruzCalamba Road. 2nd IBRD Project, at Binangonan, Rizal was Isagani de Leon, Isaac Carlos, Napoleon Maybituin, Dominga Villaroza, Florentino de la Cruz, Cipriano Aran, Celestina S. Mallari and Rodolfo Rivera, all residents of Mambog, Binangonan, Rizal. Sometime in October or November 1977, petitioner contacted the forenamed persons and informed them that he could work out their claims for payment of the values of their lots and/or improvements affected by the widening of said highway. In the process, Mejorada required the claimants to sign blank copies of the "Sworn Statement on the Correct and Fair Market Value of Real Properties" and "Agreement to Demolish, Remove and Reconstruct improvements" pertinent to their claims. The claimants complied without bothering to find out what the documents were all about as they were only interested in the payment of damages. In said "Sworn Statements" and "Agreements to Demolish", the value of the respective properties of the claimants was made to appear very much higher than the actual value claimed by them. Likewise, the said "Agreements to Demolish" reflected the value of the improvements as per assessor" which on the average was only P2, 000.00 lower than the value declared by the owners in their sworn statements. The value as per assessor was, in turn, supported by the Declarations of Real Property in the names of the claimants containing an assessed value exactly the same as that stated in the Agreements to Demolish "as per assessor", except the claims of De la Cruz and Aran where there is only a difference of P400.00 and

32

P200.00, respectively. It turned out, however, that said Declarations of Property are not really intended for the claimants as they were registered in the names of other persons, thus showing that they were all falsified. A few months after processing the claims, accused accompanied the claimants to the Office of the Highway District Engineer at the provincial capitol of Pasig, Metro Manila, to receive payments and personally assisted the claimants in signing the vouchers and encashing the checks by certifying as to their Identities and guaranteeing payment. Right after the claimants had received the proceeds of their checks, accused accompanied them to his car which was parked nearby where they were divested of the amounts paid to them leaving only the sum of P1, 000.00 to each, except Isaac Carlos to whom P5, 000.00 was left, explaining to them that there were many who would share in said amounts. All the claimants were helpless to complaint because they were afraid of the accused and his armed companion. Issue: I. Whether or not the essential elements constituting the offense penalized by section 3(e) of Republic Act No. 3019, otherwise known as the Anti-Graft and Corrupt Practices Act have been clearly and convincingly proven by the prosecution; II. Whether or not the Sandiganbayan is a court of competent jurisdiction duly constituted in accordance with Pres. Dec. No. 1606; III. Whether or not the penalty imposed upon the petitioner is excessive and contrary to the three-fold rule as provided for by Article 70 of the Revised Penal Code; IV. Whether or not there is a variance between the offense charged in the information and the offense proved; V. Whether or not the conclusion drawn from the record of the Sandiganbayan in arriving at a verdict of conviction of petitioner is correct is a question of law which this Honorable Court is authorized to pass upon. Held: The argument is devoid of merit. The Sandiganbayan established the fact that the petitioner took advantage of his position as a right-of-way-agent by making the claimants sign the aforementioned agreements to demolish and sworn statements which contained falsified declarations of the value of the improvements and lots. There was evident bad faith on the part of the petitioner when he inflated the values of the true claims and when he divested the claimants of a large share of the amounts due them. We also dispose of the fourth issue which relates to the allegation that petitioner cannot be convicted for a violation of the Anti-Graft Law because the evidence adduced by the prosecution is not the violation of Section 3 (e) but the crime of robbery. Contrary to the petitioner averment. We find no variance between the offense charged in the information and the offense proved. The prosecution was able to establish through the corroborating testimonies of the witnesses presented how through evident bad faith, petitioner caused damage to the claimants and the Government. The manner by which the petitioner divested the private parties of the compensation they received was part of' the scheme which commenced when the petitioner approached the claimants and informed them that he could work out their claims for payment of the values of their lots and/or improvements affected by the widening of the PasigSta. Cruz-Calamba Road. The evidence presented by the prosecution clearly establishes a violation of Section 3(e). The judgment convicting petitioner was a unanimous Decision of the First Division duly constituted. It thus met the requirement for the pronouncement of a judgment as required by Section 5 of P.D. 1606 supra. Petitioner is mistaken in his application of the three-fold rule as set forth in Article 70 of the Revised Penal Code. This article is to be taken into account not in the imposition of the penalty but in connection with the service of the sentence imposed (People v. Escares, 102 Phil. 677 [1957]). Article 70 speaks of "service" of sentence, "duration" of penalty and penalty "to be inflicted". Nowhere in the article is anything mentioned about the "imposition of penalty". It merely provides that the prisoner cannot be made to serve more than three times the most severe of these penalties the maximum of which is 40 yrs.

77. BALA VS. MARTINEZ 181 SCRA 459 (1990) Nature: This is petition for certiorari and prohibition with preliminary injunction and/or temporary restraining order seeks reversal of the order dated April 2, 1984 of the CFI of the RTC of Manila Branch XX Facts: The petitioner had been indicted for removing and substituting the picture of Maria Eloisa Criss Drazen which had been attached to her United States of America passport, with that of Florencia Notarte, in effect falsifying a genuine public or official document. On January 3, 1978, the trial court adjudged petitioner Manuel Bata in Criminal Case No. 24443, guilty of the crime of falsification of a public document. The petitioner seasonably appealed, but the Court of Appeals, on April 9, 1980, affirmed in toto the lower court's decision. The petitioner applied for and was granted probation by the respondent judge in his order dated August 11, 1982. The petitioner was then placed under probation for a period of one (1) year, subject to the terms and conditions enumerated therein. By the terms of the petitioner's probation, it should have expired on August 10, 1983; one year after the order granting the same was issued. But, the order of final discharge could not be issued because the respondent probation officer had not yet submitted his final report on the conduct of his charge. On December 8, 1983, the respondent People of the Philippines, through Assistant City Fiscal Jose D. Cajucom of Manila, filed a motion to revoke the probation of the petitioner before Branch XX of the Regional Trial court (RTC) of Manila, presided over by the respondent judge. The motion alleged that the petitioner had violated the terms and conditions of his probation. On January 4, 1984, the petitioner filed his opposition to the motion on the ground that he was no longer under probation, his probation period having terminated on August 10, 1983, as previously adverted to. As such, no valid reason existed to revoke the same, he contended. As if to confirm the Manila Assistant City Fiscals motion to revoke the petitioner's probation, the respondent probation, the respondent probation officer filed on January 5, 1984. the same motion, however, became the subject of a "Manifestation," dated January 10, 1984, which stated that the probation officer was not pursuing the motion to terminate dated January 6, 1984, instead, he was submitting a supplemental report which recommended the revocation of probation "in the light of new facts, information, and evidences," As stated at the outset, the respondent judge denied the motion to dismiss for lack of merit. Held: The Court finds no merit in the petition. Probation is revocable before the final discharge of the probationer by the court, contrary to the petitioner's submission. It is worthy to note, that what was actually resolved and denied was the motion to dismiss and/or strike out the motion to revoke probation, which disposed of only the issue of the petitioner's transfer of residence. The motion did not touch on the issue of the timeliness to revoke probation. The respondent judge has not yet heard and received evidence, much less acted on the matter. Rule of Law: PD 968 is clear on this score: See, 16, Termination of Probation After the period of probation and upon consideration of the report and recommendation of the probation officer, the court may order the final discharge of the probationer upon findings that he had fulfilled the terms and conditions of his probation and thereupon the case in deemed terminated. Thus, the expiration of the probation period along does not automatically terminate probation. Nowhere is the ipso facto termination of probation found in the previsions of the probation law. Probation is not coterminous with its period. There must first be issued by the court of an order of an order of final discharge based on the report and recommendation of the probation officer. Only from such issuance can the case of the probationer be deemed terminated. 78. SALGADO VS. CA 189 SCRA 304 (1990) Nature: This petition for review on certiorari seeks to set aside the decision of the Court of Appeals in CA-G.R. SP No. 15493

33

entitled, "Agustin Salgado v. Hon. Antonio P. Solano, et. Al," who affirmed the Order dated December 22, 1987 of the Regional Trial Court of Quezon City (Branch 86) sustaining its previous order dated November 18, 1987 directing the issuance of a writ execution to enforce the civil liability of herein petitioner in Criminal Case No. 0-33798. Facts: Petitioner was charged with the crime of serious physical injuries in Criminal Case No. 0-33798 entitled. "People of the Philippines v. Agustin Salgado," before the Regional Trial Court of Quezon City (Branch 86). After trial, judgment was rendered on October 16, 1987 finding him guilty beyond reasonable doubt of the crime charged. On October 17, 1986, petitioner filed an application was granted in an Order dated April 15, 1987. For the months of May, June, July, August, September and October, 1987, petitioner complied with the above condition by paying in checks the said sum of P2, 000.00 monthly, through the City Probation Officer, Perla Diaz Alonzo. Private respondent Francisco Lukban, Jr. voluntarily accepted the checks and subsequently encashed them. On September 19, 1987, private respondents Francisco Lukban, Jr. filed a motion for the issuance of a write of execution for the enforcement of the civil liability adjudged in his favor in the criminal case. The motion was opposed by the petitioner. On November 18, 1987, the trial court issued an order granting the motion for issuance of a write of execution. A motion for reconsideration was filed by petitioner but it was denied on December 22, 1987. After the denial of his motion for reconsideration, the petitioner filed directly with this Court a petition for review of the trial court's order granting the motion for issuance of a writ of execution. We referred the petition to the Court of Appeals in the resolution dated April 13, 1988. On March 16, 1989 the petitioner went to this Court via a petition for review which was filed on September 26, 1989. Issue: Whether or not the probation extinguishes civil liability. Whether or not the trial court may impose as a condition of probation the manner in which a probationer may settle his civil liability against the offended party during the period of probation. Held: The decision of respondent Court of Appeals affirming the order of the trial court granting the motion for the issuance of a writ of execution as well as the resolution dated August 3, 1989 of the same court are hereby REVERSED and SET ASIDE. Probation affects only the criminal aspect of the case. Interpreting the phrase within the context of that case, it means that although the execution of sentence is suspended by the grant of probation, it does not follow that the civil liability of the offender, if any, is extinguished. The conditions which trial courts may impose on a probationer may be classified into general or mandatory and special or discretionary. The mandatory conditions, enumerated in Section 10 of the Probation Law, require that probationer should a) present himself to the probation officer designated to undertake his supervision at such place as may be specified in the order within 72 hours from receipt of said order, and b) report to the probation officer at least once a month at such time and place as specified by said officer. Special or discretionary conditions are those additional conditions, listed in the same Section 10 of the Probation Law, which the courts may additionally impose on the probationer towards his correction and rehabilitation outside of prison. The enumeration, however, is not inclusive. Probation statutes are liberal in character and enable courts to designate practically any term it chooses as long as the probationer's constitutional rights are not jeopardized. There are innumerable conditions, which may be relevant to the rehabilitation of the probationer when viewed in their specific individual context. It should, however, be borne in mind that the special or discretionary conditions of probation should be realistic, purposive and geared to help the probationer develop into a law-abiding and self-respecting individual, Conditions should be interpreted with flexibility in their application, and each case should be judged on its own merits-on the basis of the problems, needs and capacity of the probationer.

*This is an exception because there was a CLEAR condition in the probation, therefore, when the decision of the SC came out, the period had already lapsed. 79. MONSANTO VS. FACTORAN 170 SCRA 190 (1989) Facts: March 25, 1983, the Sandiganbayan convicted petitioner Salvacion A. Monsanto (then assistant treasurer of Calbayog City) and three other accused, of the complex crime of estafa thru falsification of public documents and sentenced them t imprisonment of four (4) years, two (2) months and one (1) day of prison correctional as minimum, to ten (10) years and one (1) day of prison mayor as maximum, and to pay a fine of P3, 500. They were further ordered to jointly and severally indemnify the government in the sum of P4, 892.50 representing the balance of the amount defrauded and to pay the costs proportionately Petitioner Monsanto appealed her conviction to this Court which subsequently affirmed the same. She then filed a motion for reconsideration but while said motion was pending, she was extended on December 17, 1984 by then President Marcos absolute pardon which she accepted on December 21, 1984. By reason of said pardon petitioner wrote the Calbayog City treasurer requesting that she be restored to her former post as assistant city treasurer since the same was still vacant. Issue: Whether or not a public officer, who has been granted an absolute pardon by the Chief Executive, is entitled to reinstatement to her former position without need of a new appointment. Held: The absolute disqualification or ineligibility from public office forms part of the punishment prescribed by the Revised Penal Code for estafa thru falsification of public documents. It is clear from authorities referred to that when her guilt and punishment were expunged by her pardon; this particular disability was likewise removed. Henceforth, petitioner may apply for reappointment to the office, which was forfeited by reason of her conviction. And in considering her qualifications and suitability for the public post, the facts constituting her offense must be and should be evaluated and taken the account with public funds. Stated differently, the pardon to petitioner has resulted in removing her disqualification from holding public employment but it cannot go beyond that, to regain her former pose as assistant city treasurer, she must reapply and undergo the usual procedure required for a new appointment. Ruling: It is well too remember that petitioner had been convicted of the complex crime of estafa thru falsification of public documents and sentenced to imprisonment of four years, two months and one day of prision correccional as minimum, to ten years and one day of prision mayor as maximum. The penalty of prision mayor carries the accessory penalties of temporary absolute disqualification and perpetual special disqualification from the right of suffrage, enforceable during the term of the principal penalty, the accessory penalties remain unless the same have been expressly remitted by the pardon. The penalty of prision correccional carried, as one of its accessory penalties, suspension from public office. The propositions earlier advanced by petitioner reveal her inadequate understanding of the nature of pardon and its legal consequences. This is not totally unexpected considering that the authorities on the subject have not been wholly consistent particularly in describing the effects of pardon. A pardon looks to the future, it is not retrospective. It makes no amends for the past, it affords no relief for what has been suffered by the offender, it does not impose upon the government any obligation to make reparation for what has been suffered since the offense has been established by judicial proceedings, that which has been rightfully done and justly suffered, and no satisfaction for it can be required. This would explain why petitioner, though pardoned cannot be entitled to receive back pay for lost earnings and benefits. The better considered cases regard full pardon (at least one not based on the offenders innocence) as relieving the party from all the punitive consequences of his criminal act, including the disqualifications or disabilities based on the of guilt. But it relieves him from nothing more. To say, however, that the

34

offender is a new man and as innocent as if he had never committed the offense. Is to ignore the difference between as if he had never committed the offense. is to ignore the difference between the crime and the criminal. A person adjudged guilty of an offense is a convicted criminal, though pardoned; he may be deserving of punishment, though left unpunished; and the law may regard him as more dangerous to society than one never guilty of crime, though it places no restraints upon him following his conviction. Pardons cannot mask the acts constituting the crime. These are historical facts which despite the public manifestation of mercy and forgiveness implicit in pardon, ordinary, prudent men will take into account in their subsequent dealings with the actor. 80. DOROJA VS. CARPIO Nature: A petition by certiorari the decision of the Municipal Trial Court of Zamboanga City, Branch IV, which denied petitioners motion for subsidiary writ of execution against the owner of the vehicle which figured in the accident. Facts: October 23, 1985, accused-respondent Edwin Ramirez, while driving a passenger Fuso Jeepney owned and operated by Eduardo Toribio, bumped Dionisio Carpio, a pedestrian crossing the street, as a consequence of which the latter suffered from fractured left clavicle as reflected in the medico-legal certificate and sustained injuries which required medical attention for a period of (3) three months. Finding the accused Edwin Ramirez y Wee guilty as a principal beyond reasonable doubt of the Amended |Information to which he voluntarily pleaded guilty and appreciating the mitigating circumstance in his favor, hereby sentences him to suffer the penalty of one (1) month and one (1) day to two (2) months of arresto mayor in its minimum period. The accused filed an application for probation. A writ of execution dated Mach 10, 1988 was duly served upon the accused but was, however, returned unsatisfied due to the insolvency of the accused as shown by the sheriffs return. Thus, complaint moved for a subsidiary writ of execution against the subsidiary liability of the owner-operator of the vehicle. The same was denied by the trail court. A motion for reconsideration of the said order was disallowed for the reason that complaint having failed to raise the matter of subsidiary liability with the appellate court, said court rendered its decision which has become final and executory and the trial court has no power to alter or modify such decision. Issue: Whether or not the subsidiary liability of the owneroperator may be enforced in the same criminal proceeding against the driver where the award was given or in a separate civil action? Held: compelling the owner-operator to pay on the basis of his subsidiary liability does not constitute an amendment of the judgment because in an action under Art, 103 of the Revised Penal Code, once all the requisites as earlier discussed are met, the employer becomes ipso facto subsidiary liable, without need of a separate action. Such being the case, the subsidiary liability can be enforced in the same case where the award was given, and this does not constitute an act of amending the decision. It becomes incumbent upon the court to grant a motion for subsidiary writ of execution (but only after the employer has been heard), upon conviction of the employee and after execution is returned unsatisfied due to the employees insolvency. Wherefore, the order of the respondent court disallowing the motion for subsidiary writ of execution is hereby set aside, the Court a qou is directed to hear and decide in the same proceeding the subsidiary liability of the alleged owneroperator of the passenger jeepney. Cost against private respondent. Ruling: Art. 103. Subsidiary civil liability of other persons. The subsidiary liability established in the next preceding article shall apply to employers, teacher, persons, and corporations engaged in any kind of industry for felonies committed by their servants, pupil, workmen, apprentices, or employees in t. In order that an employer may be held subsidiarily liable for the employees civil liability in the criminal action, it should be shown (1) that

the employer, etc. is engaged in any kind of industry, (2) that the employee committed the offense in the discharge of his duties and (3) that he is insolvent (Basa Marketing Corp. vs. Bolinao, 117 SCRA 156). The subsidiary liability of the employer, however, arises only after conviction of the employee in the criminal action. All these requisite present the employer becomes ipso facto subsidiarily liable upon the employees conviction and upon proof of the latters insolvency. Needles to say, the case at bar satisfies all these requirements.

35

Potrebbero piacerti anche